You are on page 1of 228

Vasile Cîrtoaje

DISCRETE
INEQUALITIES

VOLUME 3

SYMMETRIC
NONRATIONAL INEQUALITIES

ART OF PROBLEM SOLVING


2015
Contents

1 Symmetric Nonrational Inequalities 1


1.1 Applications . . . . . . . . . . . . . . . . . . . . . . . . . . . . . . . . . . . . . . 1
1.2 Solutions . . . . . . . . . . . . . . . . . . . . . . . . . . . . . . . . . . . . . . . . 21

A Glossary 211

i
ii Vasile Cîrtoaje
Chapter 1

Symmetric Nonrational Inequalities

1.1 Applications

1.1. If a, b, c are nonnegative real numbers, then


Xp Æ
a2 − a b + b2 ≤ 6(a2 + b2 + c 2 ) − 3(a b + bc + ca).

1.2. If a, b, c are nonnegative real numbers, then


v
p p p t a2 + b2 + c 2
a2 − a b + b2 + b2 − bc + c 2 + c 2 − ca + a2 ≤ 3 .
2

1.3. If a, b, c are nonnegative real numbers, then


v v v
t 2 t 2 t 2 p
a2 + b2 − a b + b2 + c 2 − bc + c 2 + a2 − ca ≥ 2 a2 + b2 + c 2 .
3 3 3

1.4. If a, b, c are nonnegative real numbers, then


Xp Æ
a2 + a b + b2 ≥ 4(a2 + b2 + c 2 ) + 5(a b + bc + ca).

1.5. If a, b, c are nonnegative real numbers, then


Xp Æ
a2 + a b + b2 ≤ 5(a2 + b2 + c 2 ) + 4(a b + bc + ca).

1
2 Vasile Cîrtoaje

1.6. If a, b, c are nonnegative real numbers, then


Xp p p
a2 + a b + b2 ≤ 2 a2 + b2 + c 2 + a b + bc + ca.

1.7. If a, b, c are nonnegative real numbers, then


p p p p p
a2 + 2bc + b2 + 2ca + c 2 + 2a b ≤ a2 + b2 + c 2 + 2 a b + bc + ca.

1.8. If a, b, c are nonnegative real numbers, then

1 1 1 1 2
p +p +p ≥p +p .
a2 + 2bc b2 + 2ca c 2 + 2a b a2 + b2 + c 2 a b + bc + ca

1.9. If a, b, c are positive real numbers, then


p p p p p
2a2 + bc + 2b2 + ca + 2c 2 + a b ≤ 2 a2 + b2 + c 2 + a b + bc + ca.

p
1.10. Let a, b, c be nonnegative real numbers such that a + b + c = 3. If k = 3 − 1,
then XÆ p
a(a + k b)(a + kc) ≤ 3 3.

1.11. If a, b, c are nonnegative real numbers such that a + b + c = 3, then



a(2a + b)(2a + c) ≥ 9.

1.12. Let a, b, c be nonnegative real numbers such that a + b + c = 3. Prove that


Æ Æ Æ
b2 + c 2 + a(b + c) + c 2 + a2 + b(c + a) + a2 + b2 + c(a + b) ≥ 6.

1.13. Let a, b, c be positive real numbers such that a + b + c = 3. Prove that


Æ Æ Æ
a (a + b)(a + c) + b (b + c)(b + a) + c (c + a)(c + b) ≥ 6.
Symmetric Nonrational Inequalities 3

1.14. Let a, b, c be positive real numbers such that a b + bc + ca = 3. Prove that


Æ Æ Æ
a (a + 2b)(a + 2c) + b (b + 2c)(b + 2a) + c (c + 2a)(c + 2b) ≥ 9.

1.15. Let a, b, c be nonnegative real numbers such that a + b + c = 1. Prove that


Æ Æ Æ p
a + (b − c)2 + b + (c − a)2 + c + (a − b)2 ≥ 3.

1.16. Let a, b, c be nonnegative real numbers, no two of which are zero. Prove that
v v v
t a(b + c) t b(c + a) t c(a + b)
+ + ≥ 2.
a2 + bc b2 + ca c2 + a b

1.17. Let a, b, c be positive real numbers such that a bc = 1. Prove that


1 1 1
p
3
+p
3
+p
3
≥ 1.
a2 + 25a + 1 b2 + 25b + 1 c2 + 25c + 1

1.18. If a, b, c are nonnegative real numbers, then


p p p 3
a2 + bc + b2 + ca + c2 + a b ≤ (a + b + c).
2

1.19. If a, b, c are nonnegative real numbers, then


p p p p
a2 + 9bc + b2 + 9ca + c 2 + 9a b ≥ 5 a b + bc + ca.

1.20. If a, b, c are nonnegative real numbers, then



(a2 + 4bc)(b2 + 4ca) ≥ 5(a b + ac + bc).

1.21. If a, b, c are nonnegative real numbers, then



(a2 + 9bc)(b2 + 9ca) ≥ 7(a b + ac + bc).
4 Vasile Cîrtoaje

1.22. If a, b, c are nonnegative real numbers, then



(a2 + b2 )(b2 + c 2 ) ≤ (a + b + c)2 .

1.23. If a, b, c are nonnegative real numbers, then



(a2 + a b + b2 )(b2 + bc + c 2 ) ≥ (a + b + c)2 .

1.24. If a, b, c are nonnegative real numbers, then



(a2 + 7a b + b2 )(b2 + 7bc + c 2 ) ≥ 7(a b + ac + bc).

1.25. If a, b, c are nonnegative real numbers, then


v
X t 7 7 13
‹ ‹
a + ab + b
2 2 b + bc + c ≤
2 2 (a + b + c)2 .
9 9 12

1.26. If a, b, c are nonnegative real numbers, then


v
X t 1 1 61
‹ ‹
a + ab + b
2 2 b + bc + c ≤
2 2 (a + b + c)2 .
3 3 60

1.27. If a, b, c are nonnegative real numbers, then


a b c
p +p +p ≥ 1.
4b + bc + 4c
2 2 4c + ca + 4a
2 2 4a + a b + 4b2
2

1.28. If a, b, c are nonnegative real numbers, then


a b c a+b+c
p +p +p ≥p .
b2 + bc + c2 c2 + ca + a2 a2 + ab + b2 a b + bc + ca

1.29. If a, b, c are nonnegative real numbers, then


a b c a+b+c
p +p +p ≤p .
a2 + 2bc b2 + 2ca c2 + 2a b a b + bc + ca
Symmetric Nonrational Inequalities 5

1.30. If a, b, c are nonnegative real numbers, then


p p p
a3 + b3 + c 3 + 3a bc ≥ a2 a2 + 3bc + b2 b2 + 3ca + c 2 c 2 + 3a b.

1.31. Let a, b, c be nonnegative real numbers, no two of which are zero. Prove that
a b c
p +p +p ≤ 1.
4a2 + 5bc 4b2 + 5ca 4c 2 + 5a b

1.32. Let a, b, c be nonnegative real numbers. Prove that


p p p
a 4a2 + 5bc + b 4b2 + 5ca + c 4c 2 + 5a b ≥ (a + b + c)2 .

1.33. Let a, b, c be nonnegative real numbers. Prove that


p p p
a a2 + 3bc + b b2 + 3ca + c c 2 + 3a b ≥ 2(a b + bc + ca).

1.34. Let a, b, c be nonnegative real numbers. Prove that


p p p
a a2 + 8bc + b b2 + 8ca + c c 2 + 8a b ≤ (a + b + c)2 .

1.35. Let a, b, c be nonnegative real numbers, no two of which are zero. Prove that

a2 + 2bc b2 + 2ca c 2 + 2a b p
p +p +p ≥3 a b + bc + ca.
b2 + bc + c 2 c 2 + ca + a2 a2 + a b + b2

1.36. Let a, b, c be nonnegative real numbers, no two of which are zero. If k ≥ 1, then

a k+1 b k+1 c k+1 ak + bk + c k


+ + ≤ .
2a2 + bc 2b2 + ca 2c 2 + a b a+b+c

1.37. If a, b, c are positive real numbers, then


a2 − bc b2 − ca c2 − a b
(a) p +p +p ≥ 0;
3a2 + bc 3b2 + ca 3c 2 + a b
a2 − bc b2 − ca c2 − a b
(b) p +p +p ≥ 0.
8a2 + (b + c)2 8b2 + (c + a)2 8c 2 + (a + b)2
6 Vasile Cîrtoaje

p
1.38. Let a, b, c be positive real numbers. If 0 ≤ k ≤ 1 + 2 2, then

a2 − bc b2 − ca c2 − a b
p +p +p ≥ 0.
ka2 + b2 + c 2 k b2 + c 2 + a2 kc 2 + a2 + b2

1.39. If a, b, c are nonnegative real numbers, then


p p p
(a2 − bc) b + c + (b2 − ca) c + a + (c 2 − a b) a + b ≥ 0.

1.40. If a, b, c are nonnegative real numbers, then


p p p
(a2 − bc) a2 + 4bc + (b2 − ca) b2 + 4ca + (c 2 − a b) c 2 + 4a b ≥ 0.

1.41. If a, b, c are nonnegative real numbers, then


v v v
t a3 t b3 t c3
+ + ≥ 1.
a3 + (b + c)3 b3 + (c + a)3 c 3 + (a + b)3

1.42. If a, b, c are positive real numbers, then

v v v
u
1 1 1 1 1 1
t  ‹ t  ‹
(a + b + c) + + ≥1+ 1+ (a2 + b2 + c 2 ) + +
t
.
a b c a2 b2 c 2

1.43. If a, b, c are positive real numbers, then


v
1 1 1 1 1 1
t  ‹  ‹
5+ 2(a2 + b2 + c2) + + − 2 ≥ (a + b + c) + + .
a2 b2 c 2 a b c

1.44. If a, b, c are real numbers, then


Æ
2(1 + a bc) + 2(1 + a2 )(1 + b2 )(1 + c 2 ) ≥ (1 + a)(1 + b)(1 + c).
Symmetric Nonrational Inequalities 7

1.45. Let a, b, c be nonnegative real numbers, no two of which are zero. Prove that
v v v
t a2 + bc t b2 + ca t c2 + a b 1
+ + ≥2+ p .
b2 + c2 c2 + a2 a2 + b2 2

1.46. If a, b, c are nonnegative real numbers, then


Æ Æ Æ Æ
a(2a + b + c) + b(2b + c + a) + c(2c + a + b) ≥ 12(a b + bc + ca).

1.47. Let a, b, c be nonnegative real numbers such that a + b + c = 3. Prove that


Æ Æ Æ
a (4a + 5b)(4a + 5c) + b (4b + 5c)(4b + 5a) + c (4c + 5a)(4c + 5b) ≥ 27.

1.48. Let a, b, c be nonnegative real numbers such that a b + bc + ca = 3. Prove that


Æ Æ Æ
a (a + 3b)(a + 3c) + b (b + 3c)(b + 3a) + c (c + 3a)(c + 3b) ≥ 12.

1.49. Let a, b, c be nonnegative real numbers such that a2 + b2 + c 2 = 3. Prove that


p p p Æ
2 + 7a b + 2 + 7bc + 2 + 7ca ≥ 3 3(a b + bc + ca).

1.50. Let a, b, c be nonnegative real numbers such that a b + bc + ca = 3. Prove that


Pp
(a) a(b + c)(a2 + bc) ≥ 6;
p p
a(b + c) a2 + 2bc ≥ 6 3;
P
(b)
p
a(b + c) (a + 2b)(a + 2c) ≥ 18.
P
(c)

1.51. Let a, b, c be nonnegative real numbers such that a b + bc + ca = 3. Prove that


p p p
a bc + 3 + b ca + 3 + c a b + 3 ≥ 6.
8 Vasile Cîrtoaje

1.52. Let a, b, c be nonnegative real numbers such that a + b + c = 3. Prove that


p
(b + c) b2 + c 2 + 7bc ≥ 18;
P
(a)
p p
(b + c) b2 + c 2 + 10bc ≤ 12 3.
P
(b)

1.53. Let a, b, c be nonnegative real numbers such then a + b + c = 2. Prove that


p p p p
a + 4bc + b + 4ca + c + 4a b ≥ 4 a b + bc + ca.

1.54. If a, b, c are nonnegative real numbers, then


p p p p
a2 + b2 + 7a b + b2 + c 2 + 7bc + c 2 + a2 + 7ca ≥ 5 a b + bc + ca.

1.55. If a, b, c are nonnegative real numbers, then


p p p Æ
a2 + b2 + 5a b + b2 + c 2 + 5bc + c 2 + a2 + 5ca ≥ 21(a b + bc + ca).

1.56. Let a, b, c be nonnegative real numbers such that a b + bc + ca = 3. Prove that


v
p p p t2
a a2 + 5 + b b2 + 5 + c c 2 + 5 ≥ (a + b + c)2 .
3

1.57. Let a, b, c be nonnegative real numbers such that a2 + b2 + c 2 = 1. Prove that


p p p
a 2 + 3bc + b 2 + 3ca + c 2 + 3a b ≥ (a + b + c)2 .

1.58. Let a, b, c be nonnegative real numbers such that a + b + c = 3. Prove that


v v v
t 2a + bc t 2b + ca t 2c + a b
(a) a +b +c ≥ 3;
3 3 3
v v v
t a(1 + b + c) t b(1 + c + a) t c(1 + a + b)
(b) a +b +c ≥ 3.
3 3 3
Symmetric Nonrational Inequalities 9

1.59. If a, b, c are nonnegative real numbers such that a + b + c = 3, then


Æ Æ Æ
8(a2 + bc) + 9 + 8(b2 + ca) + 9 + 8(c 2 + a b) + 9 ≥ 15.

9
1.60. Let a, b, c be nonnegative real numbers such that a + b + c = 3. If k ≥ , then
8
p p p p
a2 + bc + k + b2 + ca + k + c 2 + a b + k ≥ 3 2 + k.

1.61. If a, b, c are nonnegative real numbers such that a + b + c = 3, then


p p p p
a3 + 2bc + b3 + 2ca + c 3 + 2a b ≥ 3 3.

1.62. If a, b, c are positive real numbers, then


p p p p
a2 + bc b2 + ca c2 + a b 3 2
+ + ≥ .
b+c c+a a+b 2

1.63. If a, b, c are nonnegative real numbers, no two of which are zero,then


p p p
bc + 4a(b + c) ca + 4b(c + a) a b + 4c(a + b) 9
+ + ≥ .
b+c c+a a+b 2

1.64. If a, b, c are nonnegative real numbers, no two of which are zero,then


p p p
a a2 + 3bc b b2 + 3ca c c 2 + 3a b
+ + ≥ a + b + c.
b+c c+a a+b

1.65. If a, b, c are nonnegative real numbers, no two of which are zero,then


v v v
t 2a(b + c) t 2b(c + a) t 2c(a + b)
+ + ≥ 2.
(2b + c)(b + 2c) (2c + a)(c + 2a) (2a + b)(a + 2b)
10 Vasile Cîrtoaje

1.66. If a, b, c are nonnegative real numbers such that a b + bc + ca = 3, then


v s v v s v
t bc ca t ab t bc ca t ab
+ + ≤ 1 ≤ + + .
3a2 + 6 3b2 + 6 3c 2 + 6 6a2 + 3 6b2 + 3 6c 2 + 3

1.67. Let a, b, c be nonnegative real numbers such that a b + bc + ca = 3. If k > 1, than

a k (b + c) + b k (c + a) + c k (a + b) ≥ 6.

1.68. Let a, b, c be nonnegative real numbers such that a + b + c = 2. If 2 ≤ k ≤ 3, than

a k (b + c) + b k (c + a) + c k (a + b) ≤ 2.

1.69. Let a, b, c be nonnegative real numbers, no two of which are zero. If m > n ≥ 0,
than
bm + c m c m + am am + bm
(b + c − 2a) + (c + a − 2b) + (a + b − 2c) ≥ 0.
bn + c n c n + an an + bn

1.70. Let a, b, c be positive real numbers such that a bc = 1. Prove that


p p p
a2 − a + 1 + a2 − a + 1 + a2 − a + 1 ≥ a + b + c.

1.71. Let a, b, c be positive real numbers such that a bc = 1. Prove that


p p p
16a2 + 9 + 16b2 + 9 + 16b2 + 9 ≥ 4(a + b + c) + 3.

1.72. Let a, b, c be positive real numbers such that a bc = 1. Prove that


p p p
25a2 + 144 + 25b2 + 144 + 25c 2 + 144 ≤ 5(a + b + c) + 24.

1.73. If a, b are positive real numbers such that a b + bc + ca = 3, then


p p p
(a) a2 + 3 + b2 + 3 +
b2 + 3 ≥ a + b + c + 3;
p p p p
(b) a + b + b + c + c + a ≥ 4(a + b + c) + 6.
Symmetric Nonrational Inequalities 11

1.74. If a, b, c are nonnegative real numbers such that a + b + c = 3, then


Æ Æ Æ
(5a2 + 3)(5b2 + 3) + (5b2 + 3)(5c 2 + 3) + (5c 2 + 3)(5a2 + 3) ≥ 24.

1.75. If a, b, c are nonnegative real numbers such that a + b + c = 3, then


v
p p p t 4(a2 + b2 + c 2 ) + 42
a2 + 1 + b2 + 1 + c 2 + 1 ≥ .
3

1.76. If a, b, c are nonnegative real numbers such that a + b + c = 3, then


p p p p
(a) a2 + 3 + b2 + 3 + c2 + 3 ≥ 2(a2 + b2 + c 2 ) + 30;
p p p p
(b) 3a2 + 1 + 3b2 + 1 + 3c 2 + 1 ≥ 2(a2 + b2 + c 2 ) + 30.

1.77. If a, b, c are nonnegative real numbers such that a + b + c = 3, then


Æ Æ Æ
(32a2 + 3)(32b2 + 3) + (32b2 + 3)(32c 2 + 3) + (32c 2 + 3)(32a2 + 3) ≤ 105.

1.78. If a, b, c are positive real numbers, then

b+c c + a a + b


a − 3 + b − 3 + c − 3 ≥ 2.

1.79. If a, b, c are real numbers such that a bc 6= 0, then



b + c c + a a + b

a + b + c ≥ 2.

1.80. Let a, b, c be nonnegative real numbers, no two of which are zero, and let

2a 2b 2c
x= , y= , z= .
b+c c+a a+b
Prove that
p p p
x + y +z+ xy+ yz + z x ≥ 6.
12 Vasile Cîrtoaje

1.81. Let a, b, c be nonnegative real numbers, no two of which are zero, and let

2a 2b 2c
x= , y= , z= .
b+c c+a a+b
Prove that p p p
1 + 24x + 1 + 24 y + 1 + 24z ≥ 15.

1.82. If a, b, c are positive real numbers, then


v v v
t 7a t 7b t 7c
+ + ≤ 3.
a + 3b + 3c b + 3c + 3a c + 3a + 3b

1.83. If a, b, c are positive real numbers such that a + b + c = 3, then


Æ3
Æ
3
Æ
3
p
3
a2 (b2 + c 2 ) + b2 (c 2 + a2 ) + c 2 (a2 + b2 ) ≤ 3 2.

1.84. If a, b, c are nonnegative real numbers, no two of which are zero, then
1 1 1 1 2
+ + ≥ +p .
a+b b+c c+a a+b+c a b + bc + ca

1.85. If a, b ≥ 1, then
1 1 1 1
p + ≥p +p .
3a b + 1 2 3a + 1 3b + 1

1.86. Let a, b, c be positive real numbers such that a ≥ 1 ≥ b ≥ c and a bc = 1. Prove


that
1 1 1 3
p +p +p ≥ .
3a + 1 3b + 1 3c + 1 2

1
1.87. Let a, b, c be positive real numbers such that a + b + c = 3. If k ≥ p , then
2

(a bc)k (a2 + b2 + c 2 ) ≤ 3.
Symmetric Nonrational Inequalities 13

1.88. Let p and q be nonnegative real numbers such that p2 ≥ 3q, and let
v v
t 2p − 2w t 2p + w
g(p, q) = +2 ,
3 3

2p + 2w
 s s
2p − w

 +2 , p2 ≤ 4q
3 3
h(p, q) = p p ,
 p + p + q, p2 ≥ 4q
p

where w =
p
p2 − 3q. If a, b, c are nonnegative real numbers such that

a + b + c = p, a b + bc + ca = q,

then
p p p
(a) a+b+ b+c+ c + a ≥ g(p, q),

p + 2w p−w
with equality for a = and b = c = (or any cyclic permutation);
3 3
p p p
(b) a + b + b + c + c + a ≤ h(p, q),

p − 2w p+w
with equality for a = and b = c = (or any cyclic permutation) - when
3 3
p2 ≤ 4q, and for a = 0, b+c = p and bc = q (or any cyclic permutation) - when p2 ≥ 4q.

1.89. If a, b are positive real numbers such that a + b = a4 + b4 , then


3 3
aa b b ≤ 1 ≤ aa b b .

1.90. If a, b are positive real numbers, then

a2a + b2b ≥ a a+b + b a+b .

1.91. If a, b are positive real numbers, then

aa + b b ≥ a b + ba .
14 Vasile Cîrtoaje

1.92. If a, b are positive real numbers, then

a2a + b2b ≥ a2b + b2a .

1.93. If a, b are nonnegative real numbers such that a + b = 2, then

(a) a b + b a ≤ 1 + a b;

(b) a2b + b2a ≤ 1 + a b.

1.94. If a, b are nonnegative real numbers such that a2 + b2 = 2, then

a2b + b2a ≤ 1 + a b.

1.95. If a, b are positive real numbers, then


a+b
a a b b ≤ (a2 − a b + b2 ) 2 .

1.96. If a, b ∈ (0, 1], then


a a b b ≤ 1 − a b + a2 b2 .

1.97. If a, b are positive real numbers such that a + b ≤ 2, then


 a  b  b ‹a
+ ≤ 2.
b a

1.98. If a, b are positive real numbers such that a + b = 2, then

1
2a a b b ≥ a2b + b2a + (a − b)2 .
2

1.99. If a, b ∈ (0, 1] or a, b ∈ [1, ∞), then

2a a b b ≥ a2 + b2 .
Symmetric Nonrational Inequalities 15

1.100. If a, b are positive real numbers, then

2a a b b ≥ a2 + b2 .

1.101. If a, b are positive real numbers, then


 a+b
a2 + b2
 2
a b
a b ≥ .
2

1.102. If a, b are positive real numbers such that a2 + b2 = 2, then


1
2a a b b ≥ a2b + b2a + (a − b)2 .
2

1.103. If a, b ∈ (0, 1], then


1 1
 ‹
(a2 + b2 ) + ≤ 4.
a2a b2b

1.104. If a, b are positive real numbers such that a + b = 2, then

a b b a + 2 ≥ 3a b.

1.105. If a, b ∈ [0, 1], then

a b−a + b a−b + (a − b)2 ≤ 2.

1.106. If a, b are nonnegative real numbers such that a + b ≤ 2, then


7
a b−a + b a−b + (a − b)2 ≤ 2.
16

1.107. If a, b are nonnegative real numbers such that a + b ≤ 4, then

a b−a + b a−b ≤ 2.
16 Vasile Cîrtoaje

1
1.108. Let a, b be positive real numbers such that a + b = 2. If k ≥ , then
2
kb ka
a a b b ≥ 1.

1.109. If a, b are positive real numbers such that a + b = 2, then


p p
a b
a b ≥ 1.

1.110. If a, b are positive real numbers such that a + b = 2, then


1
1 − a a+1 b b+1 ≥ (1 − a b)2 .
3

1.111. If a, b are positive real numbers such that a + b = 2, then

a−a + b−b ≤ 2.

1.112. If a, b are nonnegative real numbers such that a + b = 2, then

a2b + b2a ≥ a b + b a ≥ a2 b2 + 1.

1.113. If a, b are positive real numbers such that a + b = 2, then

a3b + b3a ≤ 2.

1.114. If a, b are nonnegative real numbers such that a + b = 2, then


‹4
a−b

3b 3a
a +b + ≤ 2.
2

1.115. If a, b are positive real numbers such that a + b = 2, then


3 3
a a + b b ≥ 2.
Symmetric Nonrational Inequalities 17

1.116. If a, b are positive real numbers such that a + b = 2, then


2 2
a5b + b5a ≤ 2.

1.117. If a, b are positive real numbers such that a + b = 2, then


p p
a2 b
+ b2 a
≤ 2.

1.118. If a, b are nonnegative real numbers such that a + b = 2, then

a b(1 − a b)2 a b(1 − a b)2


≤ a b+1 + b a+1 − 2 ≤ .
2 3

1.119. If a, b are nonnegative real numbers such that a + b = 1, then

a2b + b2a ≤ 1.

1.120. If a, b are positive real numbers such that a + b = 1, then

2a a b b ≥ a2b + b2a .

1.121. If a, b are positive real numbers such that a + b = 1, then

a−2a + b−2b ≤ 4.

1.122. Let a, b, c, d be nonnegative real numbers such that a2 + b2 + c 2 + d 2 = 1. Prove


that p p
p p p p p p
1 − a + 1 − b + 1 − c + 1 − d ≥ a + b + c + d.

1.123. Let a, b, c, d be positive real numbers. Prove that


p
A + 2 ≥ B + 4,

where
1 1 1 1
‹
A = (a + b + c + d) + + + − 16,
a b c d
1 1 1 1
 ‹
2 2 2 2
B = (a + b + c + d ) 2 + 2 + 2 + 2 − 16.
a b c d
18 Vasile Cîrtoaje

1.124. Let a1 , a2 , . . . , an be nonnegative real numbers such that a1 + a2 + · · · + an = 1.


Prove that
3a1 + 1 + 3a2 + 1 + · · · + 3an + 1 ≥ n + 1.
p p p

1.125. Let 0 ≤ a < b and a1 , a2 , . . . , an ∈ [a, b]. Prove that


€p p Š2
a1 + a2 · · · + an − n n a1 a2 · · · an ≤ (n − 1)
p
b− a .

1.126. Let a1 , a2 , . . . , an be positive real numbers such that a1 a2 · · · an = 1. Prove that


1 1 1
+p + ··· + p ≥ 1.
1 + (n2 1 + (n2 − 1)a2 1 + (n2 − 1)an
p
− 1)a1

1.127. Let a1 , a2 , . . . , an be positive real numbers such that a1 a2 · · · an = 1. Prove that


n
X 1 1
≥ .
1+ 1 + 4n(n − 1)ai
p
i=1
2

1.128. If f is a convex function on a real interval I and a1 , a2 , . . . , an ∈ I, then


a + a + ··· + a 
1 2 n
f (a1 ) + f (a2 ) + · · · + f (an ) + n(n − 2) f ≥
n
≥ (n − 1)[ f (b1 ) + f (b2 ) + · · · + f (bn )],
where
1 X
bi = aj, i = 1, 2, · · · , n.
n − 1 j6=i

1.129. Let a1 , a2 , . . . , an be positive real numbers such that a1 a2 · · · an = 1. Prove that


n
X 1 1
≤ .
n−1+ (n − 1)2 + 4nai
p
i=1
2

1.130. If a1 , a2 , . . . , an are positive real numbers such that a1 a2 · · · an = 1, then


v
t a1 + a22 + · · · + an2
u 2
a1 + a2 + · · · + an ≥ n − 1 + .
n
Symmetric Nonrational Inequalities 19

1.131. If a1 , a2 , . . . , an are positive real numbers such that a1 a2 · · · an = 1, then


q Æ
(n − 1)(a12 + a22 + · · · + an2 ) + n − n(n − 1) ≥ a1 + a2 + · · · + an .

1.132. Let a1 , a2 , . . . , an (n ≥ 3) be positive real numbers such that a1 a2 · · · an = 1. If


2n − 1
0<p≤ ,
(n − 1)2
then
1 1 1 n
+p +p ≤p .
1 + pa1 1 + pa2 1 + pan 1+p
p

1.133. If a1 , a2 , . . . , an (n ≥ 3) are positive real numbers such that a1 a2 · · · an = 1, then

n Ç
‚ n
Œ2
X X
(n − 1)2 ai4 + 2n − 1 ≥ ai ;
i=1 i=1
n
‚ n
Œ2
X q X
ai (n − 1)2 ai2 + 2n − 1 ≥ ai .
i=1 i=1

1.134. Let a1 , a2 , . . . , an be positive real numbers such that a1 a2 · · · an ≥ 1. If k > 1,


then
X a1k
≥ 1.
a1k + a2 + · · · + an

1.135. Let a1 , a2 , . . . , an be positive real numbers such that a1 a2 · · · an ≥ 1. If


−2
≤ k < 1,
n−2
then
X a1k
≤ 1.
a1k + a2 + · · · + an

1.136. Let a1 , a2 , . . . , an be positive real numbers such that a1 a2 · · · an ≥ 1. If k > 1,


then X a1
≤ 1.
a1 + a2 + · · · + an
k
20 Vasile Cîrtoaje

1.137. Let a1 , a2 , . . . , an be positive real numbers such that a1 a2 · · · an ≥ 1. If

2
−1 − ≤ k < 1,
n−2
then X a1
≥ 1.
a1k + a2 + · · · + an

1.138. Let a1 , a2 , . . . , an be positive real numbers such that a1 a2 · · · an = 1. If k ≥ 0,


then X 1
≤ 1.
a1 + a2 + · · · + an
k

1.139. Let a1 , a2 , . . . , an be nonnegative real numbers such that a1 + a2 + · · · + an ≥ n.


If 1 < k ≤ n + 1, then
a1 a2 an
+ + ··· + ≤ 1.
a1k + a2 + · · · + an a1 + a2k + · · · + an a1 + a2 + · · · + ank

1.140. Let a1 , a2 , . . . , an be nonnegative real numbers such that a1 + a2 + · · · + an ≤ n.


If 0 ≤ k < 1, then
1 1 1
+ + ··· + ≥ 1.
a1k + a2 + · · · + an a1 + a2k + · · · + an a1 + a2 + · · · + ank

1.141. Let a1 , a2 , . . . , an be positive real numbers. If k > 1, then

X ak + ak + · · · + ak n(a1k + a2k + · · · + ank )


2 3 n
≤ .
a2 + a3 + · · · + an a1 + a2 + · · · + an
Symmetric Nonrational Inequalities 21

1.2 Solutions

P 1.1. If a, b, c are nonnegative real numbers, then


Xp Æ
a2 − a b + b2 ≤ 6(a2 + b2 + c 2 ) − 3(a b + bc + ca).

Solution. By squaring, the inequality becomes as follows



2(a b + bc + ca) + 2 (a2 − a b + b2 )(a2 − ac + c 2 ) ≤ 4(a2 + b2 + c 2 ),
X €p p Š2
a2 − a b + b2 − a2 − ac + c 2 ≥ 0.

The equality holds for a = b = c, and also for a = 0 and b = c (or any cyclic permuta-
tion).

P 1.2. If a, b, c are nonnegative real numbers, then


v
p p p t a2 + b2 + c 2
a2 − a b + b2 + b2 − bc + c 2 + c 2 − ca + a2 ≤ 3 .
2

Solution (by Nguyen Van Quy). Assume that c = min{a, b, c}. Since

b2 − bc + c 2 ≤ b2

and
c 2 − ca + a2 ≤ a2 ,
it suffices to show that
v
p t a2 + b2 + c 2
a2 − a b + b2 + a + b ≤ 3 .
2

Using the Cauchy-Schwarz inequality, we have



(a + b)2
p t ˜
a2 − ab + b2 +a+b≤ (a2 − ab + b2 ) + (1 + k)
k
v
t (1 + k)[(1 + k)(a2 + b2 ) + (2 − k)a b]
= .
k
22 Vasile Cîrtoaje

Choosing k = 2, we get
v v
p t a2 + b2 t a2 + b2 + c 2
a2 − a b + b2 + a + b ≤ 3 ≤3 = 3.
2 2

The equality holds for a = b and c = 0 (or any cyclic permutation).

P 1.3. If a, b, c are nonnegative real numbers, then


v v v
t 2 t 2 t 2 p
a2 + b2 − a b + b + c − bc + c 2 + a2 − ca ≥ 2 a2 + b2 + c 2 .
2 2
3 3 3

(Vasile Cîrtoaje, 2012)

First Solution. By squaring, the inequality becomes



2 (3a2 + 3b2 − 2a b)(3a2 + 3c 2 − 2ac) ≥ 6(a2 + b2 + c 2 ) + 2(a b + bc + ca),

X €p p Š2
6(a2 + b2 + c 2 − a b − bc − ca) ≥ 3a2 + 3b2 − 2a b − 3a2 + 3c 2 − 2ac ,
X X (b − c)2 (3b + 3c − 2a)2
3 (b − c)2 ≥ p p 2 ,
3a2 + 3b2 − 2a b + 3a2 + 3c 2 − 2ac
 
X (3b + 3c − 2a)2
(b − c)2 1 − €p p Š2  .
9a + 9b − 6a b + 9a + 9c − 6ac
2 2 2 2

Since p Æ
9a2 + 9b2 − 6a b = (3b − a)2 + 8a2 ≥ |3b − a|,
p Æ
9a2 + 9c 2 − 6ac = (3c − a)2 + 8a2 ≥ |3c − a|,
it suffices to show that
‹2 
|3b + 3c − 2a|
X  
2
(b − c) 1 − ≥ 0.
|3b − a| + |3c − a|

This is true since

|3b + 3c − 2a| = |(3b − a) + (3c − a)| ≤ |3b − a| + |3c − a|.

The equality holds for a = b = c, and also for b = c = 0 (or any cyclic permutation).
Symmetric Nonrational Inequalities 23

Second Solution. Assume that a ≥ b ≥ c. Write the inequality as


Æ Æ Æ
(a + b)2 + 2(a − b)2 + (b + c)2 + 2(b − c)2 + (a + c)2 + 2(a − c)2 ≥
Æ
≥ 2 3(a2 + b2 + c 2 ).
By Minkowski’s inequality, it suffices to show that
Æ Æ
[(a + b) + (b + c) + (a + c)]2 + 2[(a − b) + (b − c) + (a − c)]2 ≥ 2 3(a2 + b2 + c 2 ),

which is equivalent to
Æ Æ
(a + b + c)2 + 2(a − c)2 ≥ 3(a2 + b2 + c 2 ).

By squaring, the inequality turns into

(a − b)(b − c) ≥ 0.

P 1.4. If a, b, c are nonnegative real numbers, then


Xp Æ
a2 + a b + b2 ≥ 4(a2 + b2 + c 2 ) + 5(a b + bc + ca).

(Vasile Cîrtoaje, 2009)

First Solution. By squaring, the inequality becomes



(a2 + a b + b2 )(a2 + ac + c 2 ) ≥ (a + b + c)2 .

Using the Cauchy-Schwarz inequality, we get


v
b 2 3b2 

c 2 3c 2
XÆ Xu  ‹ ˜
(a + a b + b )(a + ac + c ) =
2 2 2 2 a+ + a+ +
t
2 4 2 4
X • c  3bc
‹ ˜
b 
≥ a+ a+ + = (a + b + c)2 .
2 2 4
The equality holds for a = b = c, and also for b = c = 0 (or any cyclic permutation).
Second Solution. Assume that a ≥ b ≥ c. By Minkowski’s inequality, we get
Xp XÆ
2 a2 + a b + b2 = 3(a + b)2 + (a − b)2
Æ
≥ 3[(a + b) + (b + c) + (c + a)]2 + [(a − b) + (b − c) + (a − c)]2
24 Vasile Cîrtoaje

Æ
=2 3(a + b + c)2 + (a − c)2 .
Therefore, it suffices to show that

3(a + b + c)2 + (a − c)2 ≥ 4(a2 + b2 + c 2 ) + 5(a b + bc + ca),

which is equivalent to the obvious inequality

(a − b)(b − c) ≥ 0.

Remark. Similarly, we can prove the following generalization.


• Let a, b, c be nonnegative real numbers such that a b + bc + ca = 4. If |k| ≤ 2, then
Xp p
a2 + ka b + b2 ≥ 2 a2 + b2 + c 2 + 3k + 2.

For k = −2/3 and k = 1, we get the inequalities in P 1.3 and P 1.4, respectively. For
k = −1 and k = 0, we get the inequalities
Xp p
a2 − a b + b2 ≥ 2 a2 + b2 + c 2 − 1,
Xp p
a2 + b2 ≥ 2 a2 + b2 + c 2 + 2.

P 1.5. If a, b, c are nonnegative real numbers, then


Xp Æ
a2 + a b + b2 ≤ 5(a2 + b2 + c 2 ) + 4(a b + bc + ca).

(Michael Rozenberg, 2008)

First Solution (by Vo Quoc Ba Can). Using the Cauchy-Schwarz inequality, we have
— X b2 + bc + c 2
 
€X p Š2 ”X
b + bc + c
2 2 ≤ (b + c)
b+c

b + bc + c 2
X 2  X a  2
= 2(a + b + c) =2 1+ (b + bc + c 2 )
b+c b+c
X 2a(b2 + bc + c 2 )
= 4(a2 + b2 + c 2 ) + 2(a b + bc + ca) +
b+c
 ‹
2 2 2
X bc
= 4(a + b + c ) + 2(a b + bc + ca) + 2a b + c −
b+c
X 1
= 4(a2 + b2 + c 2 ) + 6(a b + bc + ca) − 2a bc .
b+c
Symmetric Nonrational Inequalities 25

Thus, it suffices to prove that


X 1
4(a2 + b2 + c 2 ) + 6(a b + bc + ca) − 2a bc ≤ 5(a2 + b2 + c 2 ) + 4(a b + bc + ca),
b+c
which is equivalent to Schur’s inequality
X 1
2(a b + bc + ca) ≤ a2 + b2 + c 2 + 2a bc .
b+c
We can prove this inequality by writing it as follows:
X  bc
‹
(a + b + c)2 ≤ 2 a a+ ,
b+c
X a
(a + b + c)2 ≤ 2(a b + bc + ca) ,
b+c
”X —X a
(a + b + c)2 ≤ a(b + c) .
b+c
Clearly, the last inequality follows from the Cauchy-Schwarz inequality. The equality
holds for a = b = c.
Second Solution. Let us denote
p p p
A = b2 + bc + c 2 , B = c 2 + ca + a2 , C= a2 + a b + b2 .

Without loss of generality, assume that a ≥ b ≥ c. For b = c = 0, the inequality is clearly


true. Consider further b > 0. By squaring, the inequality becomes
X X X
2 BC ≤ 3 a2 + 3 a b,
X X X
a2 − ab ≤ (B − C)2 ,
X X (b − c)2
(b − c)2 ≤ 2(a + b + c)2 .
(B + C)2
Since
(B + C)2 ≤ 2(B 2 + C 2 ) = 2(2a2 + b2 + c 2 + ca + a b),
it suffices to show that
X X (b − c)2
(b − c)2 ≤ (a + b + c)2 ,
2a2 + b2 + c 2 + ca + a b
which is equivalent to X
(b − c)2 Sa ≥ 0,
26 Vasile Cîrtoaje

where
(a + b + c)2 −a2 + a b + 2bc + ca
Sa = − 1 = ,
2a2 + b2 + c 2 + ca + a b 2a2 + b2 + c 2 + ca + a b
−b2 + bc + 2ca + a b
Sb = ≥ 0,
2b2 + c 2 + a2 + a b + bc
−c 2 + ca + 2a b + bc
Sc = ≥ 0.
2c 2 + a2 + b2 + bc + ca
According to
X a2
(b − c)2 Sa ≥ (b − c)2 Sa + (a − c)2 S b ≥ (b − c)2 Sa + 2 (b − c)2 S b
b

2 Sa Sb
 ‹
2 a 2
≥ (b − c) Sa + (b − c) S b = a(b − c) + ,
b a b
it suffices to prove that
Sa S b
+ ≥ 0,
a b
which is equivalent to

−b2 + bc + 2ca + a b a2 − a b − 2bc − ca


≥ .
b(2b2 + c 2 + a2 + a b + bc) a(2a2 + b2 + c 2 + ca + a b)

Consider the non-trivial case a2 − a b − 2bc − ca ≥ 0. Since

(2a2 + b2 + c 2 + ca + a b) − (2b2 + c 2 + a2 + a b + bc) = (a − b)(a + b + c) ≥ 0,

it suffices to show that

−b2 + bc + 2ca + a b a2 − a b − 2bc − ca


≥ .
b a
Indeed,

a(−b2 + bc + 2ca + a b) − b(a2 − a b − 2bc − ca) = 2c(a2 + a b + b2 ) > 0.

P 1.6. If a, b, c are nonnegative real numbers, then


Xp p p
a2 + a b + b2 ≤ 2 a2 + b2 + c 2 + a b + bc + ca.

(Vasile Cîrtoaje, 2010)


Symmetric Nonrational Inequalities 27

First Solution (by Nguyen Van Quy). Assume that a = max{a, b, c}. Since
p p Æ
a2 + a b + b2 + c 2 + ca + a2 ≤ 2[(a2 + a b + b2 ) + (c 2 + ca + a2 )],

it suffices to show that


p p p p
2 A + b2 + bc + c 2 ≤ 2 X + Y ,

where
1
A = a2 + (b2 + c 2 + a b + ac), X = a2 + b2 + c 2 , Y = a b + bc + ca.
2
Write the desired inequality as follows
p p p p
2( A − X ) ≤ Y − b2 + bc + c 2 ,

2(A − X ) Y − (b2 + bc + c 2 )
p p ≤p p ,
A+ X Y + b2 + bc + c 2
b(a − b) + c(a − c) b(a − b) + c(a − c)
p p ≤p p .
A+ X Y + b2 + bc + c 2
Since b(a − b) + c(a − c) ≥ 0, we need to show that
p p p p
A + X ≥ Y + b2 + bc + c 2 .

This inequality is true because X ≥ Y and


p p
A ≥ b2 + bc + c 2 .

Indeed,

2(A − b2 − bc − c 2 ) = 2a2 + (b + c)a − (b + c)2 = (2a − b − c)(a + b + c) ≥ 0.

The equality holds for a = b = c, and also for b = c = 0 (or any cyclic permutation).
Second Solution. In the first solution of P 1.5, we have shown that
€X p Š2 X 1
b2 + bc + c 2 ≤ 4(a2 + b2 + c 2 ) + 6(a b + bc + ca) − 2a bc .
b+c
Thus, it suffices to prove that
X 1 € p p Š2
4(a2 + b2 +c 2 )+6(a b+ bc +ca)−2a bc ≤ 2 a2 + b2 + c 2 + a b + bc + ca ,
b+c
which is equivalent to
X 1 Æ
2a bc + 4 (a2 + b2 + c 2 )(a b + bc + ca) ≥ 5(a b + bc + ca).
b+c
28 Vasile Cîrtoaje

Since X 1 9 9
≥P = ,
b+c (b + c) 2(a + b + c)
it is enough to prove that

9a bc Æ
+ 4 (a2 + b2 + c 2 )(a b + bc + ca) ≥ 5(a b + bc + ca),
a+b+c
which can be written as
9a bc Æ
+ 4 q(p2 − 2q) ≥ 5q,
p
where
p = a + b + c, q = a b + bc + ca.
For p2 ≥ 4q, this inequality is true because 4 q(p2 − 2q) ≥ 5q. Consider further
p

3q ≤ p2 ≤ 4q.

By Schur’s inequality of third degree,

9a bc
≥ 4q − p2 .
p

Therefore, it suffices to show that


Æ
(4q − p2 ) + 4 q(p2 − 2q) ≥ 5q,

which is Æ
4 q(p2 − 2q) ≥ p2 + q.
Indeed,
16q(p2 − 2q) − (p2 + q)2 = (p2 − 3q)(11q − p2 ) ≥ 0.
Third Solution. Let us denote
p p p
A = b2 + bc + c 2 , B = c 2 + ca + a2 , C = a2 + a b + b2 ,
p p
X = a2 + b2 + c 2 , Y = a b + bc + ca.
By squaring, the inequality becomes
X X
2 BC ≤ 2 a2 + 4X Y,
X
(B − C)2 ≥ 2(X − Y )2 ,
P 2
2
X (b − c)2 (b − c)2
2(a + b + c) ≥ .
(B + C)2 (X + Y )2
Symmetric Nonrational Inequalities 29

Since
B + C ≤ (c + a) + (a + b) = 2a + b + c,
it suffices to show that
P 2
2
X (b − c)2 (b − c)2
2(a + b + c) ≥ .
(2a + b + c)2 (X + Y )2

According to the Cauchy-Schwarz inequality, we have


P 2
X (b − c)2 (b − c)2
≥P .
(2a + b + c)2 (b − c)2 (2a + b + c)2

Therefore, it is enough to prove that

2(a + b + c)2 1
≥ ,
(b − c)2 (2a + b + c)2 (X + Y )2
P

which is X
2(a + b + c)2 (X + Y )2 ≥ (b − c)2 (2a + b + c)2 .
We see that
€X X Š €X X Š
(a + b + c)2 (X + Y )2 ≥ a2 + 2 ab a2 + ab
€X Š2 €X Š €X Š €X Š2
= a2 + 3 ab a2 + 2 ab
X X X
≥ a4 + 3 a b(a2 + b2 ) + 4 a2 b2
and X X
(b − c)2 (2a + b + c)2 = (b − c)2 [4a2 + 4a(b + c) + (b + c)2 ]
X X X
=4 a2 (b − c)2 + 4 a(b − c)(b2 − c 2 ) + (b2 − c 2 )2
X X X
≤8 a2 b2 + 4 a(b3 + c 3 ) + 2 a4 .
Thus, it suffices to show that
X X X X X X
a4 + 3 a b(a2 + b2 ) + 4 a2 b2 ≥ 4 a2 b2 + 2 a(b3 + c 3 ) + a4 ,

which is equivalent to the obvious inequality


X
a b(a2 + b2 ) ≥ 0.
30 Vasile Cîrtoaje

P 1.7. If a, b, c are nonnegative real numbers, then


p p p p p
a2 + 2bc + b2 + 2ca + c 2 + 2a b ≤ a2 + b2 + c 2 + 2 a b + bc + ca.

(Vasile Cîrtoaje and Nguyen Van Quy, 1989)

Solution (by Nguyen Van Quy). Let


p p
X = a2 + b2 + c 2 , Y= a b + bc + ca.

Consider the nontrivial case when no two of a, b, c are zero (Y 6= 0) and write the
inequality as X€ p Š
X − a2 + 2bc ≥ 2(X − Y ),

(b − c)2
P
X (b − c)2
p ≥ .
X + a2 + 2bc X +Y
By the Cauchy-Schwarz inequality, we have
P 2
X (b − c)2 (b − c)2
p ≥P p .
X + a2 + 2bc (b − c)2 X + a2 + 2bc

Therefore, it suffices to show that

(b − c)2
P
1
p ≥ ,
(b − c) X + a + 2bc +
P
2 2 X Y

which is equivalent to X € p Š
(b − c)2 Y − a2 + 2bc ≥ 0.
From € p Š2
Y− a2 + 2bc ≥ 0.
we get
p Y 2 − (a2 + 2bc) (a − b)(c − a)
Y− a2 + 2bc ≥ = .
2Y 2Y
Thus,
X € p Š X (b − c)2 (a − b)(c − a)
(b − c)2 Y − a2 + 2bc ≥
2Y
(a − b)(b − c)(c − a) X
= (b − c) = 0.
2Y
The equality holds for a = b, or b = c, or c = a.
Symmetric Nonrational Inequalities 31

P 1.8. If a, b, c are nonnegative real numbers, then

1 1 1 1 2
p +p +p ≥p +p .
a2 + 2bc b2 + 2ca c 2 + 2a b a2 + b2 + c 2 a b + bc + ca
(Vasile Cîrtoaje, 1989)

Solution . Let p p
X= a2 + b2 + c 2 , Y= a b + bc + ca.
Consider the nontrivial case when Y > 0 and write the inequality as
X 1 1 1 1
‹  ‹
p − ≥2 − ,
a2 + 2bc X Y X

(b − c)2
P
X (b − c)2
p p ≥ .
Y (X + Y )

a2 + 2bc X + a2 + 2bc
By the Cauchy-Schwarz inequality, we have
P 2
X (b − c)2 (b − c)2
p p ≥P p p .
a2 + 2bc X + a2 + 2bc (b − c)2 a2 + 2bc X + a2 + 2bc

Therefore, it suffices to show that

(b − c)2
P
1
p p ≥ ,
(b − c) a + 2bc X + a + 2bc (X + Y)
P
2 2 2 Y

which is equivalent to
X p
(b − c)2 [X Y − X a2 + 2bc + (a − b)(c − a)] ≥ 0.

Since X X
(b − c)2 (a − b)(c − a) = (a − b)(b − c)(c − a) (b − c) = 0,
the inequality becomes
X € p Š
(b − c)2 X Y − a2 + 2bc ≥ 0,
X € p Š
(b − c)2 Y − a2 + 2bc ≥ 0.

We have proved this inequality at the preceding problem P 1.7. The equality holds for
a = b, or b = c, or c = a.
32 Vasile Cîrtoaje

P 1.9. If a, b, c are positive real numbers, then


p p p p p
2a2 + bc + 2b2 + ca + 2c 2 + a b ≤ 2 a2 + b2 + c 2 + a b + bc + ca.

Solution. We will apply Lemma below for

X = 2a2 + bc, Y = 2b2 + ca, Z = 2c 2 + a b

and
A = B = a2 + b2 + c 2 , C = a2 + b2 + c 2 .
We have
X + Y + Z = A+ B + C
and
A = B ≥ C.
Without loss of generality, assume that

a ≥ b ≥ c.

By Lemma, it suffices to show that

max{X , Y, Z} ≥ A, min{X , Y, Z} ≤ C.

Indeed, we have

max{X , Y, Z} − A ≥ X − A = (a2 − b2 ) + c(b − c) ≥ 0,

min{X , Y, Z} − C ≤ Z − C = c(2c − a − b) ≤ 0.
Equality holds for a = b = c.
Lemma. If X , Y, Z and A, B, C are positive real numbers such that

X + Y + Z = A + B + C,

max{X , Y, Z} ≥ max{A, B, C}, min{X , Y, Z} ≤ min{A, B, C},


then p p p p p p
X+ Y+ Z≤ A + B + C.
Proof. On the assumption that X ≥ Y ≥ Z and A ≥ B ≥ C, we have

X ≥ A, Z ≤ C,

and hence
p p p p p p p p p p p p
X + Y + Z − A− B − C = ( X − A ) + ( Y − B ) + ( Z − C )
Symmetric Nonrational Inequalities 33

X −A Y −B Z −C X −A Y −B Z −C
≤ p + p + p ≤ p + p + p
2 A 2 B 2 C 2 B 2 B 2 C
C −Z Z −C 1 1
 ‹
= p + p = (C − Z) p − p ≤ 0.
2 B 2 C 2 B 2 C

Remark. This Lemma is a particular case of Karamata’s inequality.

p
P 1.10. Let a, b, c be nonnegative real numbers such that a + b + c = 3. If k = 3 − 1,
then XÆ p
a(a + k b)(a + kc) ≤ 3 3.

Solution. By the Cauchy-Schwarz inequality, we have


XÆ r€X Š ”X —
a(a + k b)(a + kc) ≤ a (a + k b)(a + kc) .

Thus, it suffices to show that


rX
(a + k b)(a + kc) ≤ a + b + c,

which is an identity. The equality holds for a = b = c = 1, and also for a = 3 and
b = c = 0 (or any cyclic permutation).

P 1.11. If a, b, c are nonnegative real numbers such that a + b + c = 3, then



a(2a + b)(2a + c) ≥ 9.

Solution. Write the inequality as follows


X ”Æ Æ —
a(2a + b)(2a + c) − a 3(a + b + c) ≥ 0,
X
(a − b)(a − c)Ea ≥ 0,
where p
a
Ea = p p .
(2a + b)(2a + c) + 3a(a + b + c)
34 Vasile Cîrtoaje

Assume that a ≥ b ≥ c. Since (c − a)(c − b)Ec ≥ 0, it suffices to show that

(a − c)Ea ≥ (b − c)E b ,

which is equivalent to
Æ Æ Æ
(a − b) 3a b(a + b + c) + (a − c) a(2b + c)(2b + a) ≥ (b − c) b(2a + b)(2a + c).

This is true if
Æ Æ
(a − c) a(2b + c)(2b + a) ≥ (b − c) b(2a + b)(2a + c).

For the non-trivial case b > c, we have


p
a−c a a
≥ ≥p .
b−c b b

Therefore, it is enough to show that

a2 (2b + c)(2b + a) ≥ b2 (2a + b)(2a + c).

Write this inequality as

a2 (2a b + 2bc + ca) ≥ b2 (2a b + bc + 2ca).

It is true if
a(2a b + 2bc + ca) ≥ b(2a b + bc + 2ca).

Indeed,

a(2a b + 2bc + ca) − b(2a b + bc + 2ca) = (a − b)(2a b + bc + ca) ≥ 0.

The equality holds for a = b = c = 1, and also for a = 0 and b = c = 3/2 (or any cyclic
permutation).

P 1.12. Let a, b, c be nonnegative real numbers such that a + b + c = 3. Prove that


Æ Æ Æ
b2 + c 2 + a(b + c) + c 2 + a2 + b(c + a) + a2 + b2 + c(a + b) ≥ 6.
Symmetric Nonrational Inequalities 35

Solution. Denote

A = b2 + c 2 + a(b + c), B = c 2 + a2 + b(c + a), C = a2 + b2 + c(a + b).

First Solution. Write the inequality in the homogeneous form


p p p
A + B + C ≥ 2(a + b + c).

By squaring, the inequality becomes


Xp X X
2 BC ≥ 2 a2 + 6 bc,
X X p p 2
(a − b)2 ≥ B− C ,
X
(b − c)2 Sa ≥ 0,
where
(b + c − a)2
Sa = 1 − p p .
( B + C)2
Since
(b + c − a)2 a(a + 3b + 3c)
Sa ≥ 1 − = ≥ 0, S b ≥ 0, Sc ≥ 0,
B+C B+C
the conclusion follows. The equality holds for a = b = c = 1, and also for a = 3 and
b = c = 0 (or any cyclic permutation).
Second Solution. Write the original inequality as follows
Xp
( A − b − c) ≥ 0,

X c(a − b) + b(a − c)
p ≥ 0,
A+ b + c
X c(a − b) X c(b − a)
p + p ≥ 0,
A+ b + c B+c+a
X c(a − b)[a − b − (pA − pB)]
p p ≥ 0.
( A + b + c)( B + c + a)
It suffices to show that
p p
(a − b)[a − b + ( B − A)] ≥ 0.

Indeed,
p p a+b−c
 ‹
2
(a − b)[a − b + ( B − A)] = (a − b) 1 + p p ≥ 0,
B+ A
36 Vasile Cîrtoaje

because, for the nontrivial case a + b − c < 0, we have


a+b−c a+b−c
1+ p p >1+ > 0.
B+ A c+c
16
Open Generalization. Let a, b, c be nonnegative real numbers. If 0 < k ≤ , then
9

(b + c)2 + k(a b − 2bc + ca) ≥ 2(a + b + c).

16
Notice that if k = , then the equality holds for a = b = c = 1, for a = 0 and b = c (or
9
any cyclic permutation), and for b = c = 0 (or any cyclic permutation).

P 1.13. Let a, b, c be positive real numbers such that a + b + c = 3. Prove that


Æ Æ Æ
a (a + b)(a + c) + b (b + c)(b + a) + c (c + a)(c + b) ≥ 6.

First Solution. Write the inequality in the homogeneous form


X Æ
3 a (a + b)(a + c) ≥ 2(a + b + c)2 ,
X X 3 X €p p Š2
a2 − ab ≥ a a+b− a+c ,
2
X X a(b − c)2
(b − c)2 ≥ 3 p p 2 ,
a+b+ a+c
X
(b − c)2 Sa ≥ 0,
where
3a
Sa = 1 − p p 2 .
a+b+ a+c
Since
3a
Sa ≥ 1 − p p 2 > 0, S b > 0, Sc > 0,
a+ a
the inequality is true. The equality holds for a = b = c = 1.
Second Solution. By Hölder’s inequality, we have

( a)3
P
”X Æ —2 27
a (a + b)(a + c) ≥ P a =P a .
(a + b)(a + c) (a + b)(a + c)
Symmetric Nonrational Inequalities 37

Therefore, it suffices to show that


X a 3
≤ .
(a + b)(a + c) 4
This inequality has the homogeneous form
X a 9
≤ ,
(a + b)(a + c) 4(a + b + c)
which is equivalent to the obvious inequality
X
a(b − c)2 ≥ 0.

P 1.14. Let a, b, c be positive real numbers such that a b + bc + ca = 3. Prove that


Æ Æ Æ
a (a + 2b)(a + 2c) + b (b + 2c)(b + 2a) + c (c + 2a)(c + 2b) ≥ 9.

First Solution. Write the inequality as follows:


X Æ
a (a + 2b)(a + 2c) ≥ 3(a b + bc + ca),
X X 1 X €p p Š2
a2 − ab ≥ a a + 2b − a + 2c ,
2
X X a(b − c)2
(b − c)2 ≥ 4 p p 2 ,
a + 2b + a + 2c
X
(b − c)2 Sa ≥ 0,
where
4a
Sa = 1 − p p 2 .
a + 2b + a + 2c
Since
4a
Sa > 1 − p p 2 = 0, S b > 0, Sc > 0,
a+ a
the inequality is true. The equality holds for a = b = c = 1.
Second Solution. We use the AM-GM inequality to get
X Æ X 2a(a + 2b)(a + 2c) X 2a(a + 2b)(a + 2c)
a (a + 2b)(a + 2c) = ≥
(a + 2b) + (a + 2c)
p
2 (a + 2b)(a + 2c)
1 X
= a(a + 2b)(a + 2c).
a+b+c
38 Vasile Cîrtoaje

Thus, it suffices to show that


X
a(a + 2b)(a + 2c) ≥ 9(a + b + c).
Write this inequality in the homogeneous form
X
a(a + 2b)(a + 2c) ≥ 3(a + b + c)(a b + bc + ca),
which is equivalent to Schur’s inequality of degree three
a3 + b3 + c 3 + 3a bc ≥ a b(a + b) + bc(b + c) + ca(c + a).

P 1.15. Let a, b, c be nonnegative real numbers such that a + b + c = 1. Prove that


Æ Æ Æ p
a + (b − c)2 + b + (c − a)2 + c + (a − b)2 ≥ 3.
(Phan Thanh Nam, 2007)
Solution. By squaring, the inequality becomes

[a + (b − c)2 ][b + (a − c)2 ] ≥ 3(a b + bc + ca).
Applying the Cauchy-Schwarz inequality, it suffices to show that
Xp X
ab + (b − c)(a − c) ≥ 3(a b + bc + ca).
This is equivalent to the homogeneous inequality
€X Š €X p Š X
a ab + a2 ≥ 4(a b + bc + ca).
p p p
Making the substitution x = a, y = b, z = c, the inequality turns into
€X Š €X Š X X
x2 xy + x4 ≥ 4 x 2 y 2,
which is equivalent to
X X X X
x4 + x y(x 2 + y 2 ) + x yz x ≥4 x 2 y 2.
Since X X
4 x2 y2 ≤ 2 x y(x 2 + y 2 ),
it suffices to show that
X X X
x 4 + x yz x≥ x y(x 2 + y 2 ),
1
which is just Schur’s inequality of degree four. The equality holds for a = b = c = ,
3
1
and for a = 0 and b = c = (or any cyclic permutation).
2
Symmetric Nonrational Inequalities 39

P 1.16. Let a, b, c be nonnegative real numbers, no two of which are zero. Prove that
v v v
t a(b + c) t b(c + a) t c(a + b)
+ + ≥ 2.
a2 + bc b2 + ca c2 + a b
(Vasile Cîrtoaje, 2006)

Solution. Using the AM-GM inequality gives


v
t a(b + c) a(b + c) 2a(b + c) 2a(b + c)
=p ≥ 2 = .
a + bc
2
(a2 + bc)(ab + ac) (a + bc) + (a b + ac) (a + b)(a + c)

Therefore, it suffices to show that

a(b + c) b(c + a) c(a + b)


+ + ≥ 1,
(a + b)(a + c) (b + c)(b + a) (c + a)(c + b)
which is equivalent to

a(b + c)2 + b(c + a)2 + c(a + b)2 ≥ (a + b)(b + c)(c + a),

4a bc ≥ 0.
The equality holds for a = 0 and b = c (or any cyclic permutation).

P 1.17. Let a, b, c be positive real numbers such that a bc = 1. Prove that


1 1 1
p
3
+p
3
+p
3
≥ 1.
a2 + 25a + 1 b2 + 25b + 1 c 2 + 25c + 1

Solution. Replacing a, b, c by a3 , b3 , c 3 , respectively, we need to show that a bc = 1


yields
1 1 1
p
3
+p 3
+p 3
≥ 1.
a + 25a + 1
6 3 b + 25b + 1
6 3 c + 25c 3 + 1
6

We first show that


1 1
p ≥ 2 .
3
a + 25a + 1
6 3 a +a+1
This is equivalent to
(a2 + a + 1)3 ≥ a6 + 25a3 + 1,
which is true since

(a2 + a + 1)3 − (a6 + 25a3 + 1) = 3a(a − 1)2 (a2 + 4a + 1) ≥ 0.


40 Vasile Cîrtoaje

Therefore, it suffices to prove that


1 1 1
+ 2 + 2 ≥ 1.
a2 +a+1 b + b+1 b + b+1
Putting
yz zx xy
a= , b= , c= , x, y, z > 0
x2 y2 z2
we need to show that
X x4
≥ 1.
x 4 + x 2 yz + y 2 z 2
Indeed, the Cauchy-Schwarz inequality gives
P 2 2
x + 2 x2 y2
P 4 P
X x4 x
≥P =P
x 4 + x 2 yz + y 2 z 2 (x 4 + x 2 yz + y 2 z 2 ) x 4 + x yz x + x 2 y 2
P P
P 2 2 P
x y − x yz x
=1+ P ≥ 1.
x 4 + x yz x + x 2 y 2
P P

The equality holds for a = b = c = 1.

P 1.18. If a, b, c are nonnegative real numbers, then


p p p 3
a2 + bc + b2 + ca + c 2 + a b ≤ (a + b + c).
2
(Pham Kim Hung, 2005)
Solution. Without loss of generality, assume that a ≥ b ≥ c. Since the equality occurs
for a = b and c = 0, we use the inequalities
p c
a2 + bc ≤ a +
2
and p p Æ
b2 + ca + c 2 + a b ≤ 2(b2 + ca) + 2(c 2 + a b).
Thus, it suffices to prove that
Æ a + 3b + 2c
2(b2 + ca) + 2(c 2 + a b) ≤ .
2
By squaring, this inequality becomes
a2 + b2 − 4c 2 − 2a b + 12bc − 4ca ≥ 0,
(a − b − 2c)2 + 8c(b − c) ≥ 0.
The equality holds for a = b and c = 0 (or any cyclic permutation).
Symmetric Nonrational Inequalities 41

P 1.19. If a, b, c are nonnegative real numbers, then


p p p p
a2 + 9bc + b2 + 9ca + c 2 + 9a b ≥ 5 a b + bc + ca.

(Vasile Cîrtoaje, 2012)

First Solution (by Nguyen Van Quy). Assume that c = min{a, b, c}. Since the equality
occurs for a = b and c = 0, we use the inequality
p p
c 2 + 9a b ≥ 3 a b.

Also, by Minkowski’s inequality, we have


s
p p €p p Š2
a2 + 9bc + b2 + 9ca ≥ (a + b)2 + 9c a+ b .

Therefore, it suffices to show that


s
€p p Š2 p p
(a + b)2 + 9c a + b ≥ 5 a b + bc + ca − 3 a b.

By squaring, this inequality becomes


p Æ
(a + b)2 + 18c a b + 30 a b(a b + bc + ca) ≥ 34a b + 16c(a + b).

Since
c(a + b) 2 c(a + b)(3a b − ac − bc)
• ˜
a b(a b + bc + ca) − a b + = ≥ 0,
3 9
p
it suffices to show that f (c) ≥ 0 for 0 ≤ c ≤ a b, where
p
f (c) = (a + b)2 + 18c a b + [30a b + 10c(a + b)] − 34a b − 16c(a + b)
p
= (a + b)2 − 4a b + 6c(3 a b − a − b).
p
Since f (c) is a linear function, we only need to prove that f (0) ≥ 0 and f ( a b) ≥ 0.
We have
f (0) = (a − b)2 ≥ 0,
p p p
f ( a b) = (a + b)2 + 14a b − 6(a + b) a b ≥ (a + b)2 + 9a b − 6(a + b) a b
€ p Š2
= a + b − 3 a b ≥ 0.
The equality holds for a = b and c = 0 (or any cyclic permutation).
Second Solution. Assume that c = min{a, b, c}. By squaring, the inequality becomes
X XÆ X
a2 + 2 (a2 + 9bc)(b2 + 9ca) ≥ 16 a b,
42 Vasile Cîrtoaje

X Æ p €p p Š X
a2 + 2 (a2 + 9bc)(b2 + 9ca) + 2 c 2 + 9a b a2 + 9bc + b2 + 9ca ≥ 16 a b.
The Cauchy-Schwarz inequality gives
Æ p
(a2 + 9bc)(b2 + 9ca) ≥ a b + 9c a b.

In addition, Minkowski’s inequality gives


s
p p €p p Š2
a2 + 9bc + b2 + 9ca ≥ (a + b)2 + 9c a + b ≥ a + b + 4c,

hence p €p p Š p
c 2 + 9a b a2 + 9bc + b2 + 9ca ≥ 3 a b (a + b + 4c).
Therefore, it suffices to show that f (c) ≥ 0, where
p p X
f (c) = a2 + b2 + 2(a b + 9c a b) + 6 a b (a + b + 4c) − 16 ab
p p
= a2 + b2 − 14a b + 6(a + b) a b + c[42 a b − 16(a + b)].
p
Since fp(c) is a linear function and 0 ≤ c ≤ a b, it is sufficient to show that f (0) ≥ 0
and f ( a b) ≥ 0. We have
p €p p Š2
f (0) = (a − b)2 + 6 ab a− b ≥0

and
p p p
f ( a b) = a2 + b2 + 28a b − 10(a + b) a b ≥ (a + b)2 + 25a b − 10(a + b) a b
€ p Š2
= a + b − 5 a b ≥ 0.

P 1.20. If a, b, c are nonnegative real numbers, then



(a2 + 4bc)(b2 + 4ca) ≥ 5(a b + ac + bc).

(Vasile Cîrtoaje, 2012)

First Solution (by Michael Rozenberg). Assume that a ≥ b ≥ c. For b = c = 0, the


inequality is trivial. Assume further that b > 0 and write the inequality as follows:
X Æ
[ (b2 + 4ca)(c 2 + 4a b) − bc − 2a(b + c)] ≥ 0,

X (b2 + 4ca)(c 2 + 4a b) − (bc + 2a b + 2ac)2


p ≥ 0,
(b2 + 4ca)(c 2 + 4a b) + bc + 2a(b + c)
Symmetric Nonrational Inequalities 43

X
(b − c)2 Sa ≥ 0,
where
a(b + c − a) Æ
Sa = , A = (b2 + 4ca)(c 2 + 4a b) + bc + 2a(b + c),
A
b(c + a − b) Æ
Sb = , B= (c 2 + 4a b)(a2 + 4bc) + ca + 2b(c + a),
B
c(a + b − c) Æ
Sc = , C = (a2 + 4bc)(b2 + 4ac) + a b + 2c(a + b).
C
Since S b ≥ 0 and Sc ≥ 0, we have
X a2
(b − c)2 Sa ≥ (b − c)2 Sa + (a − c)2 S b ≥ (b − c)2 Sa + 2 (b − c)2 S b
b
bSa aS b
 ‹
a
= (b − c)2 + .
b a b
Thus, it suffices to prove that
bSa aS b
+ ≥ 0,
a b
which is equivalent to
b(b + c − a) a(c + a − b)
+ ≥ 0.
A B
Since
b(b + c − a) a(c + a − b) b(b − a) a(a − b) (a − b)(aA − bB)
+ ≥ + = ,
A B A B AB
it is enough to show that
aA − bB ≥ 0.
Indeed,
p ” p p —
aA − bB = c 2 + 4a b a b2 + 4ca − b a2 + 4bc + 2(a − b)(a b + bc + ca)
p
4c(a3 − b3 ) c 2 + 4a b
= p p + 2(a − b)(a b + bc + ca) ≥ 0.
a b2 + 4ca + b a2 + 4bc
The equality holds for a = b = c, and also for a = b and c = 0 (or any cyclic permuta-
tion).
Second Solution (by Nguyen Van Quy). Write the inequality as
€p p p Š2
a2 + 4bc + b2 + 4ca + c 2 + 4a b ≥ a2 + b2 + c 2 + 14(a b + bc + ca),
44 Vasile Cîrtoaje

p p p Æ
a2 + 4bc + b2 + 4ca + c 2 + 4a b ≥ a2 + b2 + c 2 + 14(a b + bc + ca).

Assume that c = min{a, b, c}. For t = 2c, the inequality (b) in Lemma below becomes
p p Æ
a2 + 4bc + b2 + 4ca ≥ (a + b)2 + 8(a + b)c.

Thus, it suffices to show that


Æ p Æ
(a + b)2 + 8(a + b)c + c 2 + 4a b ≥ a2 + b2 + c 2 + 14(a b + bc + ca).

By squaring, this inequality becomes


Æ
[(a + b)2 + 8(a + b)c] (c 2 + 4a b) ≥ 4a b + 3(a + b)c,

2(a + b)c 3 − 2(a + b)2 c 2 + 2a b(a + b)c + a b(a + b)2 − 4a2 b2 ≥ 0,

2(a + b)(a − c)(b − c)c + a b(a − b)2 ≥ 0.

Lemma. Let a,b and t be nonnegative numbers such that

t ≤ 2(a + b).

Then,
p
(a) (a2 + 2bt)(b2 + 2at) ≥ a b + (a + b)t;
p p p
(b) a2 + 2bt + b2 + 2at ≥ (a + b)2 + 4(a + b)t.

Proof. (a) By squaring, the inequality becomes

(a − b)2 t[2(a + b) − t] ≥ 0,

which is clearly true.

(b) By squaring, this inequality turns into the inequality in (a).

P 1.21. If a, b, c are nonnegative real numbers, then



(a2 + 9bc)(b2 + 9ca) ≥ 7(a b + ac + bc).

(Vasile Cîrtoaje, 2012)


Symmetric Nonrational Inequalities 45

Solution (by Nguyen Van Quy). We see that the equality holds for a = b and c = 0.
Without loss of generality, assume that c = min{a, b, c}. For t = 4c, the inequality (a)
in Lemma from the preceding P 1.20 becomes
Æ
(a2 + 8bc)(b2 + 8ca) ≥ a b + 4(a + b)c.
Thus, we have Æ
(a2 + 9bc)(b2 + 9ca) ≥ a b + 4(a + b)c,
and also,
p €p p Š p Æ
4
c 2 + 9a b a2 + 9bc + b2 + 9ca ≥ 3 a b · 2 (a2 + 9bc)(b2 + 9ca)
p Æ Æ
≥ 6 a b · a b + 4(a + b)c = 3 4a2 b2 + 16a bc(a + b)
Æ
≥ 3 4a2 b2 + 4a bc(a + b) + c 2 (a + b)2 = 3(2a b + bc + ca).
Therefore,

(a2 + 9bc)(b2 + 9ca) ≥ (a b + 4bc + 4ca) + 3(2a b + bc + ca)
= 7(a b + bc + ca).
The equality holds for a = b and c = 0 (or any cyclic permutation).

P 1.22. If a, b, c are nonnegative real numbers, then


Æ Æ Æ
(a2 + b2 )(b2 + c 2 ) + (b2 + c 2 )(c 2 + a2 ) + (c 2 + a2 )(a2 + b2 ) ≤ (a + b + c)2 .
(Vasile Cîrtoaje, 2007)
Solution. Without loss of generality, assume that a = min{a, b, c}. Let us denote
a a
y= + b, z= + c.
2 2
Since
a2 + b2 ≤ y 2 , b2 + c 2 ≤ y 2 + z 2 , c 2 + a2 ≤ z 2 ,
it suffices to prove that
Æ
yz + ( y + z) y 2 + z 2 ≤ ( y + z)2 .
This is true since
Æ y 2z2
y 2 + yz + z 2 − ( y + z) y 2 + z 2 = ≥ 0.
y 2 + yz + z 2 + ( y + z) y 2 + z 2
p

The equality holds for a = b = 0 (or any cyclic permutation).


46 Vasile Cîrtoaje

P 1.23. If a, b, c are nonnegative real numbers, then



(a2 + a b + b2 )(b2 + bc + c 2 ) ≥ (a + b + c)2 .

Solution. By the Cauchy-Schwarz inequality, we have

b 2 3b2  c 2 3c 2
 ‹  
2 2 2 2
(a + a b + b )(a + ac + c ) = a + + a+ +
2 4 2 4

c  3bc a(b + c)
 ‹
b
≥ a+ a+ + = a2 + + bc.
2 2 4 2
Then,
X• a(b + c)
XÆ ˜
2
(a + a b + b )(a + ac + c ) ≥
2 2 2 2 a + + bc = (a + b + c)2 .
2
The equality holds for a = b = c, and also for b = c = 0 (or any cyclic permutation).

P 1.24. If a, b, c are nonnegative real numbers, then



(a2 + 7a b + b2 )(b2 + 7bc + c 2 ) ≥ 7(a b + ac + bc).

(Vasile Cîrtoaje, 2012)

First Solution. Without loss of generality, assume that c = min{a, b, c}. We see that the
equality holds for a = b and c = 0. Since
Æ
(a2 + 7ac + c 2 )(b2 + 7bc + c 2 ) ≥ (a + 2c)(b + 2c) ≥ a b + 2c(a + b),

it suffices to show that


p €p p Š
a2 + 7a b + b2 a2 + 7ac + b2 + 7bc ≥ 6a b + 5c(a + b).

By Minkowski’s inequality, we have


s
p p €p p Š2
a2 + 7ac + b2 + 7bc ≥ (a + b)2 + 7c a+ b
v
t 28a bc
≥ (a + b)2 + 7c(a + b) + .
a+b
Therefore, it suffices to show that
28a bc
• ˜
(a2 + 7a b + b2 ) (a + b)2 + 7c(a + b) + ≥ (6a b + 5bc + 5ca)2 .
a+b
Symmetric Nonrational Inequalities 47

1
Due to homogeneity, we may assume that a + b = 1. Let us denote d = a b, d ≤ .
4
Since
2a b
c≤ = 2d,
a+b
1
we need to show that f (c) ≥ 0 for 0 ≤ c ≤ 2d ≤ , where
2
f (c) = (1 + 5d)(1 + 7c + 28cd) − (6d + 5c)2 .

Since f (c) is concave, it suffices to show that f (0) ≥ 0 and f (2d) ≥ 0. Indeed,

f (0) = 1 + 5d − 36d 2 = (1 − 4d)(1 + 9d) ≥ 0

and

f (2d) = (1 + 5d)(1 + 14d + 56d 2 ) − 256d 2 ≥ (1 + 4d)(1 + 14d + 56d 2 ) − 256d 2

= (1 − 4d)(1 + 22d − 56d 2 ) ≥ d(1 − 4d)(22 − 56d) ≥ 0.


The equality holds for a = b and c = 0 (or any cyclic permutation).
Second Solution. We will use the inequality
Æ 2x y
x 2 + 7x y + y 2 ≥ x + y + , x, y ≥ 0,
x+y
which, by squaring, reduces to
x y(x − y)2 ≥ 0.
We have
X 2a b 2ac
XÆ ‹ ‹
(a2 + 7a b + b2 )(a2 + 7ac + c2) ≥ a+b+ a+c+
a+b a+c
X X X 2a2 b X 2a2 c X 2a bc
≥ a2 + 3 ab + + + .
a+b a+c a+b
Since
X 2a2 b X 2a2 c X 2a2 b X 2b2 a X
+ = + =2 ab
a+b a+c a+b b+a
and X 2a bc 18a bc 9a bc
≥P = ,
a+b (a + b) a+b+c
it suffices to show that X 9a bc X
a2 + ≥2 a b,
a+b+c
which is just Schur’s inequality of degree three.
48 Vasile Cîrtoaje

P 1.25. If a, b, c are nonnegative real numbers, then


v
X t 7 7 13
‹ ‹
a2 + a b + b2 b2 + bc + c 2 ≤ (a + b + c)2 .
9 9 12
(Vasile Cîrtoaje, 2012)
Solution (by Nguyen Van Quy). Without loss of generality, assume that c = min{a, b, c}.
It is easy to see that the equality holds for a = b = 1 and c = 0. By the AM-GM inequality,
the following inequality holds for any k > 0:
v ‚v v Œ
t 7 t 7 t 7
12 a + a b + b
2 2 a + ac + c + b + bc + c ≤
2 2 2 2
9 9 9
‚v v Œ2
36 2 7 7 7
 ‹ t t
≤ a + a b + b2 + k a2 + ac + c 2 + b2 + bc + c 2 .
k 9 9 9
We can use this inequality to prove the original inequality only if
‚v v Œ2
36 2 7 7 7
 ‹ t t
2
a + ab + b = k a2 + ac + c 2 + b2 + bc + c 2
k 9 9 9

for a = b = 1 and c = 0. This necessary condition if satisfied for k = 5. Therefore, it


suffices to show that

7 7 36 2 7
t ‹ ‹  ‹
2
12 a + ac + c
2 2 b + bc + c +
2 2 a + ab + b
9 9 5 9
‚v v Œ2
t 7 t 7
+5 a2 + ac + c 2 + b2 + bc + c 2 ≤ 13(a + b + c)2 .
9 9
which is equivalent to

7 7 4(a + b)2 + 94a b 199c(a + b)
t ‹ ‹
22 a + ac + c
2 2 b + bc + c ≤
2 2 + 3c 2 + .
9 9 5 9
Since
v v
7 7 16 16
t ‹ ‹ t ‹ ‹
2 a2 + ac + c 2 b + bc + c ≤ 2
2 2 a +
2 ac b +
2 bc
9 9 9 9
v 
16 16
t ‹  ‹
=2 a b+ c ·b a+ c
9 9
16 16
 ‹  ‹
≤a b+ c +b a+ c
9 9
16c(a + b)
= 2a b + ,
9
Symmetric Nonrational Inequalities 49

we only need to prove that

8c(a + b) 4(a2 + b2 ) + 102a b 199c(a + b)


• ˜
22 a b + ≤ + 3c 2 + .
9 5 9
This reduces to the obvious inequality

4(a − b)2 23c(a + b)


+ + 3c 2 ≥ 0.
5 9
Thus, the proof is completed. The equality holds for a = b and c = 0 (or any cyclic
permutation).

P 1.26. If a, b, c are nonnegative real numbers, then


v
X t 1 1 61
‹ ‹
a + ab + b
2 2 b + bc + c ≤
2 2 (a + b + c)2 .
3 3 60
(Vasile Cîrtoaje, 2012)

Solution (by Nguyen Van Quy). Without loss of generality, assume that c = min{a, b, c}.
It is easy to see that the equality holds for c = 0 and 11(a2 + b2 ) = 38a b. By the AM-GM
inequality, the following inequality holds for any k > 0:
v ‚v v Œ
t 1 t 1 t 1
60 a2 + a b + b2 a2 + ac + c 2 + b2 + bc + c 2 ≤
3 3 3
‚v v Œ2
36 2 1 1 1
 ‹ t t
2
≤ a + a b + b + 25k a + ac + c + b + bc + c
2 2 2 2 .
k 3 3 3
We can use this inequality to prove the original inequality only if the equality
‚v v Œ2
36 2 1 1 1
 ‹ t t
2
a + a b + b = 25k a2 + ac + c 2 + b2 + bc + c 2
k 3 3 3

holds for c = 0 and 11(a2 + b2 ) = 38a b. This necessary condition if satisfied for k = 1.
Therefore, it suffices to show that

1 1 1
t ‹ ‹  ‹
2 2
60 a + ab + b
2 2 b + bc + c + 36 a + a b + b
2 2
3 3 3
‚v v Œ2
t 1 t 1
+25 a2 + ac + c 2 + b2 + bc + c 2 ≤ 61(a + b + c)2 ,
3 3
50 Vasile Cîrtoaje

which is equivalent to

1 1 31c(a + b)
t ‹ ‹
10 a + ac + c
2 2 b + bc + c ≤ 10a b + c 2 +
2 2 .
3 3 3
Since
v v
1 1 4 4
t ‹ ‹ t ‹ ‹
2 a2 + ac + c 2 b2 + bc + c 2 ≤ 2 a2 + ac b2 + bc
3 3 3 3
v 
4 4
t ‹  ‹
=2 a b+ c ·b a+ c
3 3
4 4
 ‹  ‹
≤a b+ c +b a+ c
3 3
4c(a + b)
= 2a b + ,
3
we only need to prove that
2c(a + b) 31c(a + b)
• ˜
10 a b + ≤ 10a b + c 2 + .
3 3
This reduces to the obvious inequality
3c 2 + 11c(a + b) ≥ 0.
Thus, the proof is completed. The equality holds for 11(a2 + b2 ) = 38a b and c = 0 (or
any cyclic permutation).

P 1.27. If a, b, c are nonnegative real numbers, then


a b c
p +p +p ≥ 1.
4b + bc + 4c
2 2 4c + ca + 4a
2 2 4a + a b + 4b2
2

(Pham Kim Hung, 2006)


Solution. By Hölder’s inequality, we have
P 3
a + 3 a b(a + b) + 6a bc
‹2 P 3 P
a
X
a
p ≥P = .
a(4b2 + bc + 4c 2 ) 4 a b(a + b) + 3a bc
P
4b2 + bc + 4c 2
Thus, it suffices to show that
X X
a3 + 3a bc ≥ a b(a + b),
which is Schur’s inequality of degree three. The equality holds for a = b = c, and also
for a = 0 and b = c (or any cyclic permutation).
Symmetric Nonrational Inequalities 51

P 1.28. If a, b, c are nonnegative real numbers, then

a b c a+b+c
p +p +p ≥p .
b2 + bc + c2 c2 + ca + a2 a2 + ab + b2 a b + bc + ca

Solution. By Hölder’s inequality, we have


‹2 P 3 P 2
a a
X
a
p ≥P = P ,
b2 + bc + c 2 a(b2 + bc + c 2 ) ab

from which the desired inequality follows. The equality holds for a = b = c, and also
for a = 0 and b = c (or any cyclic permutation).

P 1.29. If a, b, c are nonnegative real numbers, then

a b c a+b+c
p +p +p ≤p .
a2 + 2bc b2 + 2ca c2 + 2a b a b + bc + ca
(Ho Phu Thai, 2007)

Solution. Without loss of generality, assume that a ≥ b ≥ c.


First Solution. Since
c c
p ≤p ,
c2 + 2a b a b + bc + ca
it suffices to show that
a b a+b
p +p ≤p ,
a2 + 2bc b2 + 2ca a b + bc + ca
which is equivalent to
p p p p
a( a2 + 2bc − a b + bc + ca) b( a b + bc + ca − b2 + 2ca)
p ≥ p .
a2 + 2bc b2 + 2ca
Since p p
a2 + 2bc − a b + bc + ca ≥ 0
and
a b
p ≥p ,
a2 + 2bc b2 + 2ca
it suffices to show that
p p p p
a2 + 2bc − a b + bc + ca ≥ a b + bc + ca − b2 + 2ca,
52 Vasile Cîrtoaje

which is equivalent to
p p p
a2 + 2bc + b2 + 2ca ≥ 2 a b + bc + ca.

Using the AM-GM inequality, it suffices to show that

(a2 + 2bc)(b2 + 2ca) ≥ (a b + bc + ca)2 ,

which is equivalent to the obvious inequality

c(a − b)2 (2a + 2b − c) ≥ 0.

The equality holds for a = b = c, and also for a = b and c = 0 (or any cyclic permuta-
tion).
Second Solution. By the Cauchy-Schwarz inequality, we have
X ‹2
a €X Š X a 
p ≤ a .
a2 + 2bc a2 + 2bc
Thus, it suffices to prove that
X a a+b+c
≤ .
a2 + 2bc a b + bc + ca
This is equivalent to
X  1 1
‹
a − 2 ≥ 0,
a b + bc + ca a + 2bc
X a(a − b)(a − c)
≥ 0.
a2 + 2bc
Assuming that a ≥ b ≥ c, we get
X a(a − b)(a − c) a(a − b)(a − c) b(b − c)(b − a)
≥ +
a2 + 2bc a2 + 2bc b2 + 2ca
c(a − b)2 [2a(a − c) + 2b(b − c) + 3a b]
= ≥ 0.
(a2 + 2bc)(b2 + 2ca)

P 1.30. If a, b, c are nonnegative real numbers, then


p p p
a3 + b3 + c 3 + 3a bc ≥ a2 a2 + 3bc + b2 b2 + 3ca + c 2 c 2 + 3a b.

(Vo Quoc Ba Can, 2008)


Symmetric Nonrational Inequalities 53

Solution. If a = 0, then the inequality is an identity. Consider further that a, b, c > 0,


and write the inequality as follows:
X p
a2 ( a2 + 3bc − a) ≤ 3a bc,
X 3a2 bc
p ≤ 3a bc,
a2 + 3bc + a
X 1
Ç ≤ 1.
1 + 3bc
a 2 + 1
Let us denote
1 1 1
x=Ç , y=q , z=Ç ,
3ca
1+ 3bc
a2
+1 1+ b2
+1 1+ 3ab
c2
+1
which implies
bc 1 − 2x ca 1 − 2y ab 1 − 2z 1
2
= , = , = , 0 < x, y, z < ,
a 3x 2 b 2 3 y2 c 2 3z 2 2
and
(1 − 2x)(1 − 2 y)(1 − 2z) = 27x 2 y 2 z 2 .
We need to prove that
x + y +z ≤1
1
for 0 < x, y, z < such that (1 − 2x)(1 − 2 y)(1 − 2z) = 27x 2 y 2 z 2 . We will use the
2
1
contradiction method. Assume that x + y + z > 1 for 0 < x, y, z < , and show that
2
(1 − 2x)(1 − 2 y)(1 − 2z) < 27x 2 y 2 z 2 . We have
(1 − 2x)(1 − 2 y)(1 − 2z) < (x + y + z − 2x)(x + y + z − 2 y)(x + y + z − 2z)
< ( y + z − x)(z + x − y)(x + y − z)(x + y + z)3
≤ 3( y + z − x)(z + x − y)(x + y − z)(x + y + z)(x 2 + y 2 + z 2 )
= 3(2x 2 y 2 + 2 y 2 z 2 + 2z 2 x 2 − x 4 − y 4 − z 4 )(x 2 + y 2 + z 2 ).
Therefore, it suffices to show that
(2x 2 y 2 + 2 y 2 z 2 + 2z 2 x 2 − x 4 − y 4 − z 4 )(x 2 + y 2 + z 2 ) ≤ 9x 2 y 2 z 2 ,
which is equivalent to
X
x 6 + y 6 + z 5 + 9x 2 y 2 z 2 ≥ y 2 z 2 ( y 2 + z 2 ).

Clearly, this is just Schur’s inequality of degree three applied to x 2 , y 2 , z 2 . So, the proof
is completed. The equality holds for a = b = c, and also for a = 0 or b = 0 or c = 0.
54 Vasile Cîrtoaje

P 1.31. Let a, b, c be nonnegative real numbers, no two of which are zero. Prove that

a b c
p +p +p ≤ 1.
4a2 + 5bc 4b2 + 5ca 4c 2 + 5a b
(Vasile Cîrtoaje, 2004)

First Solution (by Vo Quoc Ba Can). If one of a, b, c is zero, then the inequality becomes
an equality. Consider next that a, b, c > 0 and denote

1
 ‹
a b c
x=p , y=p , z=p , x, y, z ∈ 0, .
4a2 + 5bc 4b2 + 5ca 4c 2 + 5a b 2

We have
bc 1 − 4x 2 ca 1 − 4 y2 ab 1 − 4z 2
= , = , = ,
a2 5x 2 b2 5 y2 c2 5z 2
and
(1 − 4x 2 )(1 − 4 y 2 )(1 − 4z 2 ) = 125x 2 y 2 z 2 .
For the sake of contradiction, assume that x + y + z > 1. Using the AM-GM inequality
and the Cauchy-Schwarz inequality, we have

1 Y 1 Y
x 2 y 2z2 = (1 − 4x 2 ) < [(x + y + z)2 − 4x 2 ]
125 125
1 Y Y
= (3x + y + z) · ( y + z − x)
125
 x + y + z 3 Y
≤ ( y + z − x)
3
1 Y
≤ (x 2 + y 2 + z 2 )(x + y + z) ( y + z − x)
9
1
= (x 2 + y 2 + z 2 )[2(x 2 y 2 + y 2 z 2 + z 2 x 2 ) − x 4 − y 4 − z 4 ],
9
and hence

9x 2 y 2 z 2 < (x 2 + y 2 + z 2 )[2(x 2 y 2 + y 2 z 2 + z 2 x 2 ) − x 4 − y 4 − z 4 ],
X
x 6 + y 6 + z 6 + 3x 2 y 2 z 2 < x 2 y 2 (x 2 + y 2 ).
The last inequality contradicts Schur’s inequality
X
x 6 + y 6 + z 6 + 3x 2 y 2 z 2 ≥ x 2 y 2 (x 2 + y 2 ).

Thus, the proof is completed. The equality holds for a = b = c, and also for a = 0 and
b = c (or any cyclic permutation).
Symmetric Nonrational Inequalities 55

bc ca
Second Solution. In the nontrivial case when a, b, c > 0, setting x = 2
, y = 2 and
a b
ab
z= (x yz = 1), the desired inequality becomes E(x, y, z) ≤ 1, where
c2
1 1 1
E(x, y, z) = p +p +p .
4 + 5x 4 + 5y 4 + 5z

Without loss of generality, we may assume that x ≥ y ≥ z, x ≥ 1, yz ≤ 1. We will prove


that
p p
E(x, y, z) ≤ E(x, yz, yz) ≤ 1.
The left inequality has the form
1 1 1
+p ≤p p .
4 + 5y 4 + 5z 4 + 5 yz
p

For the nontrivial case y 6= z, consider y > z and denote


y +z p
s= , p = yz,
2
Æ
q = (4 + 5 y)(4 + 5z).
We have s > p, p ≤ 1 and
Æ Æ
q = 16 + 40s + 25p2 > 16 + 40p + 25p2 = 4 + 5p.

By squaring, the desired inequality becomes in succession as follows:


1 1 2 4
+ + ≤ ,
4 + 5 y 4 + 5z q 4 + 5p
1 1 2 2 2
+ − ≤ − ,
4 + 5 y 4 + 5z 4 + 5p 4 + 5p q
8 + 10s 2 2(q − 4 − 5p)
− ≤ ,
q 2 4 + 5p q(4 + 5p)
(s − p)(5p − 4) 8(s − p)
≤ ,
q (4 + 5p)
2 q(4 + 5p)(q + 4 + 5p)
5p − 4 8
≤ ,
q q + 4 + 5p
25p2 − 16 ≤ (12 − 5p)q.
The last inequality is true since

(12 − 5p)q − 25p2 + 16 > (12 − 5p)(4 + 5p) − 25p2 + 16 = 2(8 − 5p)(4 + 5p) > 0.
56 Vasile Cîrtoaje

In order to prove the right inequality, namely


1 2
p +p p ≤ 1,
4 + 5x 4 + 5 yz
p
4 + 5 yz = 3t, t ∈ (2/3, 1]. Since
p
let us denote

1 25
x= = ,
yz (9t − 4)2
2

the inequality becomes


9t 2 − 4 2
p + ≤ 1,
3 36t 4 − 32t 2 + 21 3t
€p Š
(2 − 3t) 36t 4 − 32t 2 + 21 − 3t 2 − 2t ≤ 0.
Since 2 − 3t < 0, we still have to show that
p
36t 4 − 32t 2 + 21 ≥ 3t 2 + 2t.

Indeed,
p 3(t − 1)2 (9t 2 + 14t + 7)
36t 4 − 32t 2 + 21 − 3t 2 − 2t = p ≥ 0.
36t 4 − 32t 2 + 21 + 3t 2 + 2t

P 1.32. Let a, b, c be nonnegative real numbers. Prove that


p p p
a 4a2 + 5bc + b 4b2 + 5ca + c 4c 2 + 5a b ≥ (a + b + c)2 .

(Vasile Cîrtoaje, 2004)

First Solution. Write the inequality as


X €p Š
a 4a2 + 5bc − 2a ≥ 2(a b + bc + ca) − a2 − b2 − c 2 ,
X 1
5a bc p ≥ 2(a b + bc + ca) − a2 − b2 − c 2 .
4a + 5bc + 2a
2

Writing Schur’s inequality


X
a3 + b3 + c 3 + 3a bc ≥ a b(a2 + b2 )

in the form
9a bc
≥ 2(a b + bc + ca) − a2 − b2 − c 2 ,
a+b+c
Symmetric Nonrational Inequalities 57

it suffices to prove that


X 5 9
p ≥ .
4a + 5bc + 2a
2 a+b+c

Let p = a + b + c and q = a b + bc + ca. By the AM-GM inequality, we have


p
p 2 (16a2 + 20bc)(3b + 3c)2 (16a2 + 20bc) + (3b + 3c)2
4a2 + 5bc = ≤
12(b + c) 12(b + c)

16a2 + 16bc + 10(b + c)2 8a2 + 5b2 + 5c 2 + 18bc


≤ = ,
12(b + c) 6(b + c)
hence X 5 X 5
p ≥
4a + 5bc + 2a
2 8a + 5b + 5c 2 + 18bc
2 2
+ 2a
6(b + c)
X 30(b + c) X 30(b + c)
= = .
8a + 5b + 5c + 12a b + 18bc + 12ac
2 2 2 5p + 2q + 3a2 + 6bc
2

Thus, it suffices to show that


X 30(b + c) 9
≥ .
5p2 + 2q + 3a2 + 6bc p

By the Cauchy-Schwarz inequality, we get

30[ (b + c)]2
P
X 30(b + c)
≥P
5p2 + 2q + 3a2 + 6bc (b + c)(5p2 + 2q + 3a2 + 6bc)

120p2 120p2
= = .
10p3 + 4pq + 9 bc(b + c) 10p3 + 13pq − 27a bc
P

Therefore, it is enough to show that

120p2 9
≥ ,
10p + 13pq − 27a bc
3 p

which is equivalent to
10p3 + 81a bc ≥ 39pq.
From Schur’s inequality p3 + 9a bc ≥ 4pq and the known inequality pq ≥ 9a bc, we have

10p3 + 81a bc − 39pq = 10(p3 + 9a bc − 4pq) + pq − 9a bc ≥ 0.

This completes the proof. The equality holds for a = b = c, and also for a = 0 and b = c
(or any cyclic permutation).
58 Vasile Cîrtoaje

Second Solution. By the Cauchy-Schwarz inequality, we have


€X p Š X a
‹
a 4a + 5bc
2 p ≥ (a + b + c)2 .
4a2 + 5bc
From this inequality and the inequality in P 1.31, namely
X a
p ≤ 1,
4a + 5bc
2

the desired inequality follows.

Remark. Using the same way as in the second solution, we can prove the following
inequalities for a, b, c > 0 satisfying a bc = 1:
p p p
a 4a2 + 5 + b 4b2 + 5 + c 4c 2 + 5 ≥ (a + b + c)2 ;
p p p
4a4 + 5 + 4b4 + 5 + 4c 4 + 5 ≥ (a + b + c)2 .
The first inequality follows from the the Cauchy-Schwarz inequality
€X p Š X a
‹
a 4a + 5
2 p ≥ (a + b + c)2
4a2 + 5
and the inequality X a
p ≤ 1, a bc = 1,
4a2 + 5
which follows from the inequality in P 1.31 by replacing bc/a2 , ca/b2 , a b/c 2 with 1/x 2 ,
1/ y 2 , 1/z 2 , respectively.
The second inequality follows from the the Cauchy-Schwarz inequality
2
 
€X p Š X a
4a4 + 5 p ≥ (a + b + c)2
4a + 5
4

and the inequality


X a2
p ≤ 1, a bc = 1.
4a4 + 5

P 1.33. Let a, b, c be nonnegative real numbers. Prove that


p p p
a a2 + 3bc + b b2 + 3ca + c c 2 + 3a b ≥ 2(a b + bc + ca).

(Vasile Cîrtoaje, 2005)


Symmetric Nonrational Inequalities 59

First Solution (by Vo Quoc Ba Can). Using the AM-GM inequality yields
X p X a(b + c)(a2 + 3bc)
a a2 + 3bc = p
(b + c)2 (a2 + 3bc)
X 2a(b + c)(a2 + 3bc)
≥ .
(b + c)2 + (a2 + 3bc)

Thus, it suffices to prove the inequality


X 2a(b + c)(a2 + 3bc) X
≥ a(b + c),
a2 + b2 + c 2 + 5bc
which can be written as follows
X a(b + c)(a2 − b2 − c 2 + bc)
≥ 0,
a2 + b2 + c 2 + 5bc
X a3 (b + c) − a(b3 + c 3 )
≥ 0,
a2 + b2 + c 2 + 5bc
X a b(a2 − b2 ) − ac(c 2 − a2 )
≥ 0,
a2 + b2 + c 2 + 5bc
X a b(a2 − b2 ) X ba(a2 − b2 )
− ≥ 0,
a2 + b2 + c 2 + 5bc b2 + c 2 + a2 + 5ca
X 5a bc(a2 − b2 )(a − b)
≥ 0.
(a2 + b2 + c 2 + 5bc)(a2 + b2 + c 2 + 5ac)
Since the last inequality is clearly true, the proof is completed. The equality holds a =
b = c, and also for a = 0 and b = c (or any cyclic permutation).
Second Solution. Write the inequality as
X p
(a a2 + 3bc − a2 ) ≥ 2(a b + bc + ca) − a2 − b2 − c 2 .

Due to homogeneity, we may assume that a + b + c = 3. By the AM-GM inequality, we


have
p 3a bc 12a bc
a a2 + 3bc − a2 = p = p
a2 + 3bc + a 2 4(a2 + 3bc) + 4a
12a bc
≥ .
4 + a2 + 3bc + 4a
Thus, it suffices to show that
X 1
12a bc ≥ 2(a b + bc + ca) − a2 − b2 − c 2 .
4 + a2 + 3bc + 4a
60 Vasile Cîrtoaje

By the Cauchy-Schwarz inequality, we have


X 1 9
≥P
4 + a + 3bc + 4a
2 (4 + a + 3bc + 4a)
2

9
=
24 + a + 3 a b
P P
2

27
= P
8( a) + 3 a2 + 9 a b
P P
2
P
9 a 3
= ≥ P .
11( a) + 3 a b
P P
2 4 a
Then, it remains to show that
9a bc
≥ 2(a b + bc + ca) − a2 − b2 − c 2 ,
a+b+c
which is equivalent to Schur’s inequality of degree three
X X
a3 + 3a bc ≥ a b(a + b).

P 1.34. Let a, b, c be nonnegative real numbers. Prove that


p p p
a a2 + 8bc + b b2 + 8ca + c c 2 + 8a b ≤ (a + b + c)2 .

Solution. Multiplying by a + b + c, the inequality becomes


X Æ
a (a + b + c)2 (a2 + 8bc) ≤ (a + b + c)3 .

Since Æ
2 (a + b + c)2 (a2 + 8bc) ≤ (a + b + c)2 + (a2 + 8bc),
it suffices to show that
X
a[(a + b + c)2 + (a2 + 8bc)] ≤ 2(a + b + c)3 ,

which can be written as

a3 + b3 + c 3 + 24a bc ≤ (a + b + c)3 .

This inequality is equivalent to

a(b − c)2 + b(c − a)2 + c(a − b)2 ≥ 0.

The equality holds for a = b = c, and also for b = c = 0 (or any cyclic permutation).
Symmetric Nonrational Inequalities 61

P 1.35. Let a, b, c be nonnegative real numbers, no two of which are zero. Prove that

a2 + 2bc b2 + 2ca c 2 + 2a b p
p +p +p ≥3 a b + bc + ca.
b2 + bc + c 2 c 2 + ca + a2 a2 + a b + b2
(Michael Rozenberg and Marius Stanean, 2011)

Solution. By the AM-GM inequality, we have


p
X a2 + 2bc X2(a2 + 2bc) a b + bc + ca
p =
b2 + bc + c 2
p
2 (b2 + bc + c 2 )(a b + bc + ca)
p X 2(a2 + 2bc)
≥ ab + bc + ca
(b2 + bc + c 2 ) + (a b + bc + ca)
p X 2(a2 + 2bc)
= ab + bc + ca .
(b + c)(a + b + c)

Thus, it suffices to show that

a2 + 2bc b2 + 2ca c 2 + 2a b 3
+ + ≥ (a + b + c).
b+c c+a a+b 2
This inequality is equivalent to

1X
a4 + b4 + c 4 + a bc(a + b + c) ≥ a b(a + b)2 .
2
We can prove this inequality by summing Schur’s inequality of fourth degree
X
a4 + b4 + c 4 + a bc(a + b + c) ≥ a b(a2 + b2 )

and the obvious inequality


X 1X
a b(a2 + b2 ) ≥ a b(a + b)2 .
2
The equality holds for a = b = c.

P 1.36. Let a, b, c be nonnegative real numbers, no two of which are zero. If k ≥ 1, then

a k+1 b k+1 c k+1 ak + bk + c k


+ + ≤ .
2a2 + bc 2b2 + ca 2c 2 + a b a+b+c
(Vasile Cîrtoaje and Vo Quoc Ba Can, 2011)
62 Vasile Cîrtoaje

Solution. Write the inequality as follows

X ak a k+1

− 2 ≥ 0,
a + b + c 2a + bc

X a k (a − b)(a − c)
≥ 0.
2a2 + bc
Assume that a ≥ b ≥ c. Since (c − a)(c − b) ≥ 0, it suffices to show that

a k (a − b)(a − c) b k (b − a)(b − c)
+ ≥ 0.
2a2 + bc 2b2 + ca

This is true if
a k (a − c) b k (b − c)
− ≥ 0,
2a2 + bc 2b2 + ca
which is equivalent to

a k (a − c)(2b2 + ca) ≥ b k (b − c)(2a2 + bc).

Since a k /b k ≥ a/b, it remains to show that

a(a − c)(2b2 + ca) ≥ b(b − c)(2a2 + bc),

which is equivalent to the obvious inequality

(a − b)c[a2 + 3a b + b2 − c(a + b)] ≥ 0.

The equality holds for a = b = c, and also for a = b and c = 0 (or any cyclic permuta-
tion).

P 1.37. If a, b, c are positive real numbers, then

a2 − bc b2 − ca c2 − a b
(a) p +p +p ≥ 0;
3a2 + bc 3b2 + ca 3c 2 + a b
a2 − bc b2 − ca c2 − a b
(b) p +p +p ≥ 0.
8a2 + (b + c)2 8b2 + (c + a)2 8c 2 + (a + b)2
(Vasile Cîrtoaje, 2006)
Symmetric Nonrational Inequalities 63

Solution. (a) Let


p p p
A= 3a2 + bc, B= 3b2 + ca, C= 3c 2 + a b.

We have
X a2 − bc X (a − b)(a + c) + (a − c)(a + b)
2 =
A A
X (a − b)(a + c) X (b − a)(b + c)
= +
A B
a+c b+c
X ‹
= (a − b) −
A B
X a − b (a + c)2 B 2 − (b + c)2 A2
= ·
AB (a + c)B + (b + c)A
X c(a − b)2 2(a − b)2 + c(a + b + 2c)
= · ≥ 0.
AB (a + c)B + (b + c)A
The equality holds for a = b = c.
(b) Let
Æ Æ Æ
A= 8a2 + (b + c)2 , B= 8b2 + (c + a)2 , C= 8c 2 + (a + b)2 b.

As we have shown before,


X a2 − bc X a − b (a + c)2 B 2 − (b + c)2 A2
2 = · ,
A AB (a + c)B + (b + c)A
hence
X a2 − bc X (a − b)2 C1
2 = · ≥ 0,
A AB (a + c)B + (b + c)A
since

C1 = [(a + c) + (b + c)][(a + c)2 + (b + c)2 ] − 8ac(b + c) − 8bc(a + c)


≥ [(a + c) + (b + c)](4ac + 4bc) − 8ac(b + c) − 8bc(a + c)
= 4c(a − b)2 ≥ 0.

The equality holds for a = b = c.

p
P 1.38. Let a, b, c be positive real numbers. If 0 ≤ k ≤ 1 + 2 2, then

a2 − bc b2 − ca c2 − a b
p +p +p ≥ 0.
ka2 + b2 + c 2 k b2 + c 2 + a2 kc 2 + a2 + b2
64 Vasile Cîrtoaje

Solution. Let
p p p
A= ka2 + b2 + c 2 , B= k b2 + c 2 + a2 , C= kc 2 + a2 + b2 .

As we have shown at the preceding problem,


X a2 − bc X a − b (a + c)2 B 2 − (b + c)2 A2
2 = · ,
A AB (a + c)B + (b + c)A

therefore
X a2 − bc X (a − b)2 C1
2 = · ≥ 0,
A AB (a + c)B + (b + c)A
where

C1 = (a2 + b2 + c 2 )(a + b + 2c) − (k − 1)c(2a b + bc + ca)


p
≥ (a2 + b2 + c 2 )(a + b + 2c) − 2 2 c(2a b + bc + ca).

Putting a + b = 2x, we have a2 + b2 ≥ 2x 2 , a b ≤ x 2 , and hence


p p
C1 ≥ (2x 2 + c 2 )(2x + 2c) − 2 2 c(2x 2 + 2c x) = 2(x + c)(x 2 − c)2 ≥ 0.

The equality holds for a = b = c.

P 1.39. If a, b, c are nonnegative real numbers, then


p p p
(a2 − bc) b + c + (b2 − ca) c + a + (c 2 − a b) a + b ≥ 0.

First Solution. Let us denote


v v
tb+c s
c+a ta + b
x= , y= , z= ,
2 2 2
hence
a = y 2 + z2 − x 2, b = z2 + x 2 − y 2, c = x 2 + y 2 − z2.
The inequality turns into

x y(x 3 + y 3 ) + yz( y 3 + z 3 ) + z x(z 3 + x 3 ) ≥ x 2 y 2 (x + y) + y 2 z 2 ( y + z) + z 2 x 2 (z + x),

which is equivalent to the obvious inequality

x y(x + y)(x − y)2 + yz( y + z)( y − z)2 + z x(z + x)(z − x)2 ≥ 0.


Symmetric Nonrational Inequalities 65

The equality holds for a = b = c, and also for b = c = 0 (or any cyclic permutation).
Second Solution. Write the inequality as

A(a2 − bc) + B(b2 − ca) + C(c 2 − a b) ≥ 0,

where p p p
A= b + c, B= c + a, C= a + b.
We have
X X
2 A(a2 − bc) = A[(a − b)(a + c) + (a − c)(a + b)]
X X
= A(a − b)(a + c) + B(b − a)(b + c)
X
= (a − b)[A(a + c) − B(b + c)]
X A2 (a + c)2 − B 2 (b + c)2
= (a − b) ·
A(a + c) + B(b + c)
X (a − b)2 (a + c)(b + c)
= ≥ 0.
A(a + c) + B(b + c)

P 1.40. If a, b, c are nonnegative real numbers, then


p p p
(a2 − bc) a2 + 4bc + (b2 − ca) b2 + 4ca + (c 2 − a b) c 2 + 4a b ≥ 0.

(Vasile Cîrtoaje, 2005)

Solution. If two of a, b, c are zero, then the inequality is clearly true. Otherwise, write
the inequality as
AX + BY + C Z ≥ 0,
where p p p
a2 + 4bc b2 + 4ca c 2 + 4a b
A= , B= , C= ,
b+c c+a a+b
X = (a2 − bc)(b + c), Y = (b2 − bc)(b + c), X = (c 2 − a b)(a + b).
Without loss of generality, assume that a ≥ b ≥ c. We have X ≥ 0, Z ≤ 0 and

X + Y + Z = 0.

In addition,
X − Y = a b(a − b) + 2(a2 − b2 )c + (a − b)c 2 ≥ 0
66 Vasile Cîrtoaje

and

a4 − b4 + 2(a3 − c 3 )c + (a2 − c 2 )c 2 + 4a bc(a − b) − 4(a − b)c 3


A2 − B 2 =
(b + c)2 (c + a)2

4a bc(a − b) − 4(a − b)c 3 4c(a − b)(a b − c 2 )


≥ = ≥ 0.
(b + c)2 (c + a)2 (b + c)2 (c + a)2
Since
2(AX + BY + C Z) = (A − B)(X − Y ) − (A + B − 2C)Z,

it suffices to show that


A + B − 2C ≥ 0.

This is true if AB ≥ C 2 . Using the Cauchy-Schwarz inequality gives


p p
a b + 4c a b a b + 2c a b + 2c 2
AB ≥ ≥ .
(b + c)(c + a) (b + c)(c + a)

Thus, we need to show that


p
(a + b)2 (a b + 2c a b + 2c 2 ) ≥ (b + c)(c + a)(c 2 + 4a b).

Write this inequality as


p €p p Š2
a b(a − b)2 + 2c a b(a + b) a− b + c 2 [2(a + b)2 − 5a b − c(a + b) − c 2 ] ≥ 0.

It is true since

2(a + b)2 − 5a b − c(a + b) − c 2 = a(2a − b − c) + b(b − c) + b2 − c 2 ≥ 0.

The equality holds for a = b = c, and also for a = b and c = 0 (or any cyclic permuta-
tion).

P 1.41. If a, b, c are nonnegative real numbers, then


v v v
t a3 t b3 t c3
+ + ≥ 1.
a3 + (b + c)3 b3 + (c + a)3 c 3 + (a + b)3
Symmetric Nonrational Inequalities 67

Solution. For a = 0, the inequality reduces to the obvious inequality


p p p
b3 + c 3 ≥ b3 + c 3 .

For a, b, c > 0, write the inequality as


v
Xu 1
‹3 ≥ 1.
u
b+c
u 
1+
t
a

For any x ≥ 0, we have


p Æ (1 + x) + (1 − x + x 2 ) 1
1 + x3 = (1 + x)(1 − x + x 2 ≤ = 1 + x 2.
2 2
Therefore, we get v
Xu 1 X 1
‹3 ≥
u
b+c 1 b+c 2
u   ‹
1+ 1+
t
a 2 a
X 1 X a2
≥ = = 1.
b +c 2 2 a2 + b2 + c 2
1+
a2
The equality holds for a = b = c, and also for b = c = 0 (or any cyclic permutation).

P 1.42. If a, b, c are positive real numbers, then


v v v
u
1 1 1 1 1 1
t  ‹ t  ‹
(a + b + c) + + ≥ 1 + 1 + (a2 + b2 + c 2 ) 2 + 2 + 2 .
t
a b c a b c

(Vasile Cîrtoaje, 2002)

Solution. Using the Cauchy-Schwarz inequality, we have


v
€X Š X 1 ‹ t€X X Š X 1 X 1‹
a = a2 + 2 bc + 2
a a2 bc
v v
t€X Š X 1  ‹ t€X Š X 1 ‹

≥ a2 + 2 bc
a2 bc
v v
t€X Š X 1  ‹ t€X Š X 1 ‹

= a 2 +2 a ,
a2 a
68 Vasile Cîrtoaje

hence
v v
€X Š X 1 ‹ t€X Š X 1 ‹ t€X Š X 1 ‹
a −2 a +1=1+ a2 ,
a a a2
–v ™2 v
t€X Š X 1 ‹ t€X Š X 1 ‹
a −1 ≥1+ a2 ,
a a2
v v v
t€X Š X 1 ‹ u t€X Š X 1 ‹
−1≥ 1+
t
a a2 .
a a2
The equality holds if and only if
€X Š X 1 ‹ X 1 ‹ €X Š
a2 = bc ,
bc a2
which is equivalent to
(a2 − bc)(b2 − ca)(c 2 − a b) = 0.
Consequently, the equality occurs for a2 = bc, or b2 = ca, or c 2 = a b.

P 1.43. If a, b, c are positive real numbers, then


v
1 1 1 1 1 1
t  ‹  ‹
5 + 2(a + b + c ) 2 + 2 + 2 − 2 ≥ (a + b + c)
2 2 2 + + .
a b c a b c

(Vasile Cîrtoaje, 2004)

Solution. Let us denote


a b c b c a
x= + + , y= + + .
b c a a b c
We have
1 1 1
 ‹
(a + b + c) + + = x + y +3
a b c
and
1 1 1
 ‹
2 2 2
2(a + b + c ) 2 + 2 + 2 − 2 =
a b c
 2 2 2
 2
c 2 a2
 
a b c b
=2 + + +2 + + +4
b2 c 2 a2 a2 b2 c 2
= 2(x 2 − 2 y) + 2( y 2 − 2x) + 4 = (x + y − 2)2 + (x − y)2 ≥ (x + y − 2)2 .
Symmetric Nonrational Inequalities 69

Therefore, v
1 1 1
t  ‹
2(a2+ b2 + c2) + + −2 ≥ x + y −2
a2 b2 c 2
1 1 1
 ‹
= (a + b + c) + + − 5.
a b c
The equality occurs for a = b, or b = c, or c = a.

P 1.44. If a, b, c are real numbers, then


Æ
2(1 + a bc) + 2(1 + a2 )(1 + b2 )(1 + c 2 ) ≥ (1 + a)(1 + b)(1 + c).

(Wolfgang Berndt, 2006)


First Solution. Denoting

p = a + b + c, q = a b + bc + ca, r = a bc,

the inequality becomes


Æ
2(p2 + q2 + r 2 − 2pr − 2q + 1) ≥ p + q − r − 1.

It suffices to show that

2(p2 + q2 + r 2 − 2pr − 2q + 1) ≥ (p + q − r − 1)2 ,

which is equivalent to

p2 + q2 + r 2 − 2pq + 2qr − 2pr + 2p − 2q − 2r + 1 ≥ 0,

(p − q − r + 1)2 ≥ 0.
The equality holds for p + 1 = q + r and q ≥ 1. The last condition follows from p + q −
r − 1 ≥ 0.
Second Solution. Since

2(1 + a2 ) = (1 + a)2 + (1 − a)2

and
(1 + b2 )(1 + c 2 ) = (b + c)2 + (bc − 1)2 ,
by the Cauchy-Schwarz inequality, we get
Æ
2(1 + a2 )(1 + b2 )(1 + c 2 ) ≥ (1 + a)(b + c) + (1 − a)(bc − 1)

= (1 + a)(1 + b)(1 + c) − 2(1 + a bc).


70 Vasile Cîrtoaje

P 1.45. Let a, b, c be nonnegative real numbers, no two of which are zero. Prove that
v v v
t a2 + bc t b2 + ca t c 2 + a b 1
+ + ≥2+ p .
b +c
2 2 c +a
2 2 a +b
2 2
2
(Vo Quoc Ba Can, 2006)
Solution. We may assume that a ≥ b ≥ c. Then, it suffices to show that
v v v
t a2 + c 2 t b2 + c 2 t a b 1
+ + ≥2+ p .
b +c
2 2 c +a
2 2 a +b
2 2
2
Let us denote v
t a2 + c 2 s
a
x= , y= .
b2 + c2 b
Since
(a − b)(a b − c 2 )
x2 − y2 = ≥ 0,
b(b2 + c 2 )
it follows that
x ≥ y ≥ 1.
From
1 1 (x − y)(x y − 1)
 ‹
x+ − y+ = ≥ 0,
x y xy
we have v v v
t a2 + c 2 t b2 + c 2 s
a tb
+ ≥ + .
b2 + c 2 c 2 + a2 b a
Therefore, it is enough to show that
s v v
a t b t ab 1
+ + ≥2+ p .
b a a +b
2 2
2
s
a
Putting t = , the inequality becomes
b
s
1 1 t
t + −2≥ p − .
t 2 t +1
2

We have
(t − 1)2 (t − 1)2
s
1 t
p − = ‹≤ 2
t +1
2 t +1
s
1

2 t
2(t 2 + 1) p + 2+1
2 t
(t − 1) 2
1
≤ = t + − 2.
t t
The equality holds for a = b and c = 0 (or any cyclic permutation).
Symmetric Nonrational Inequalities 71

P 1.46. If a, b, c are nonnegative real numbers, then


Æ Æ Æ Æ
a(2a + b + c) + b(2b + c + a) + c(2c + a + b) ≥ 12(a b + bc + ca).

(Vasile Cîrtoaje, 2012)

Solution. By squaring, the inequality becomes



a2 + b2 + c 2 + bc(2b + c + a)(2c + a + b) ≥ 5(a b + bc + ca).

Using the Cauchy-Schwarz inequality yields


XÆ XÆ
bc(2b + c + a)(2c + a + b) = (2b2 + bc + a b)(2c 2 + bc + ac)

X p X p
≥ (2bc + bc + a bc) = 3(a b + bc + ca) + a bc.

Therefore, it suffices to show that


X p
a2 + b2 + c 2 + a bc ≥ 2(a b + bc + ca).

We can get this inequality by summing Schur’s inequality


X p Xp
a2 + b2 + c 2 + a bc ≥ a b(a + b)

and Xp
a b (a + b) ≥ 2(a b + bc + ca).

The last inequality is equivalent to the obvious inequality


Xp p p
a b ( a − b )2 ≥ 0.

The equality holds for a = b = c, and also for a = 0 and b = c (or any cyclic permuta-
tion).

P 1.47. Let a, b, c be nonnegative real numbers such that a + b + c = 3. Prove that


Æ Æ Æ
a (4a + 5b)(4a + 5c) + b (4b + 5c)(4b + 5a) + c (4c + 5a)(4c + 5b) ≥ 27.

(Vasile Cîrtoaje, 2010)


72 Vasile Cîrtoaje

Solution. Assume that a ≥ b ≥ c, consider the non-trivial case b > 0, and write the
inequality in the following equivalent homogeneous forms:
X Æ
a (4a + 5b)(4a + 5c) ≥ 3(a + b + c)2 ,
X X X €p p Š2
2( a2 − a b) = a 4a + 5b − 4a + 5c ,
X X 25a(b − c)2
(b − c)2 ≥ p p ,
( 4a + 5b + 4a + 5c)2
X
(b − c)2 Sa ≥ 0,
where
25a
Sa = 1 − p p .
( 4a + 5b + 4a + 5c)2
Since
25b 25b
Sb = 1 − p p ≥1− p p =0
( 4b + 5c + 4b + 5a)2 ( 4b + 9b)2
and
25c 25c 25
Sc = 1 − p p ≥1− p p =1− > 0,
( 4c + 5a + 4c + 5b)2 ( 9c + 9c)2 36
we have
X a2
(b − c)2 Sa ≥ (b − c)2 Sa + (a − c)2 S b ≥ (b − c)2 Sa + 2 (b − c)2 S b
b
 ‹
a b a
= (b − c)2 Sa + S b .
b a b
Thus, it suffices to prove that
b a
Sa + S b ≥ 0.
a b
We have p p
25a a( 4a + 5b − 4a)2
Sa ≥ 1 − p p =1− ,
( 4a + 5b + 4a)2 b2
p p
25b b( 4b + 5a − 4b)2
Sb ≥ 1 − p p =1− ,
( 4b + 4b + 5a)2 a2
and hence
p p p p
b a b ( 4a + 5b − 4a)2 a ( 4b + 5a − 4b)2
Sa + S b ≥ − + −
a b a b b a
‚v v Œ
t 4a2 5a t 4b2 5b 
a b
‹
=4 + + + − 7 + − 10
b2 b a2 a b a
q p
= 4 4x 2 + 5x − 8 + 2 20x + 41 − 7x − 10,
Symmetric Nonrational Inequalities 73

where
a b
x= + ≥ 2.
b a
To end the proof, we only need to show that x ≥ 2 yields
q p
4 4x 2 + 5x − 8 + 2 20x + 41 ≥ 7x + 10.

By squaring, this inequality becomes


p
15x 2 − 60x − 228 + 32 20x + 41 ≥ 0.

Indeed,
p p
15x 2 − 60x − 228 + 32 20x + 41 ≥ 15x 2 − 60x − 228 + 32 81 = 15(x − 2)2 ≥ 0.

3
The equality holds for a = b = c = 1, and also for c = 0 and a = b = (or any cyclic
2
permutation).

P 1.48. Let a, b, c be nonnegative real numbers such that a b + bc + ca = 3. Prove that


Æ Æ Æ
a (a + 3b)(a + 3c) + b (b + 3c)(b + 3a) + c (c + 3a)(c + 3b) ≥ 12.

(Vasile Cîrtoaje, 2010)

Solution. Assume that a ≥ b ≥ c (b > 0), and write the inequality as


X Æ
a (a + 3b)(a + 3c) ≥ 4(a b + bc + ca),
X X X €p p Š2
2( a2 − a b) = a a + 3b − a + 3c ,
X X 9a(b − c)2
(b − c)2 ≥ p p ,
( a + 3b + a + 3c)2
X
(b − c)2 Sa ≥ 0,
where
9a
Sa = 1 − p p .
( a + 3b + a + 3c)2
Since
9b 9b
Sb = 1 − p p ≥1− p p =0
( b + 3c + b + 3a)2 ( b + 4b)2
74 Vasile Cîrtoaje

and
9c 9c 9
Sc = 1 − p p ≥1− p p =1− > 0,
( c + 3a + c + 3b)2 ( 4c + 4c)2 16
we have
X a2
(b − c)2 Sa ≥ (b − c)2 Sa + (a − c)2 S b ≥ (b − c)2 Sa + 2 (b − c)2 S b
b
 ‹
a b a
= (b − c)2 Sa + S b .
b a b
Thus, it suffices to prove that
b a
Sa + S b ≥ 0.
a b
We have p p
9a a( a + 3b − a)2
Sa ≥ 1 − p p =1− ,
( a + 3b + a)2 b2
p p
9b b( b + 3a − b)2
Sb ≥ 1 − p p =1− ,
( b + b + 3a)2 a2
and hence
p p p p
b a b ( a + 3b − a)2 a ( b + 3a − b)2
Sa + S b ≥ − + −
a b a b b a
‚ v v Œ 
t a2 3a t b2 3b a b
‹
=2 + + + − + −6
b2 b a2 a b a
Æ p
= 2 x 2 + 3x − 2 + 2 3x + 10 − x − 6,

where
a b
x= + ≥ 2.
b a
To end the proof, it remains to show that
Æ p
2 x 2 + 35x − 2 + 2 3x + 10 ≥ x + 6

for x ≥ 2. By squaring, this inequality becomes


p
3x 2 − 44 + 8 3x + 10 ≥ 0.

Indeed, p
3x 2 − 44 + 8 3x + 10 ≥ 12 − 44 + 32 = 0.
p
The equality holds for a = b = c = 1, and also for c = 0 and a = b = 3 (or any cyclic
permutation).
Symmetric Nonrational Inequalities 75

P 1.49. Let a, b, c be nonnegative real numbers such that a2 + b2 + c 2 = 3. Prove that


p p p Æ
2 + 7a b + 2 + 7bc + 2 + 7ca ≥ 3 3(a b + bc + ca).
(Vasile Cîrtoaje, 2010)
Solution. Consider a ≥ b ≥ c. Since the inequality is trivial for b = c = 0, we may
assume that b > 0. By squaring, the desired inequality becomes

a2 + b2 + c 2 + (2 + 7a b)(2 + 7ac) ≥ 10(a b + bc + ca),
X €p p Š2
6(a2 + b2 + c 2 − a b − bc − ca) = 2 + 7a b − 2 + 7ac ,
X X 49a2 (b − c)2
3 (b − c)2 ≥ p p ,
( 2 + 7a b + 2 + 7ac )2
X
(b − c)2 Sa ≥ 0,
where
49a2
Sa = 1 − p p ,
( 6 + 21a b + 6 + 21ac)2
49b2
Sb = 1 − p p ,
( 6 + 21a b + 6 + 21bc)2
49c 2
Sc = 1 − p p .
( 6 + 21ac + 6 + 21bc)2
Since 6 ≥ 2(a2 + b2 ) ≥ 4a b, we have
49a2 49a2 a
Sa ≥ 1 − p p ≥1− p p =1− ,
( 4a b + 21a b + 4a b + 21ac)2 (5 a b + 2 a b)2 b
49b2 49b2 b
Sb ≥ 1 − p p ≥1− p p =1− ,
( 4a b + 21a b + 4a b + 21bc)2 (5 a b + 2 a b)2 a
49c 2 49c 2 49
Sc ≥ 1 − p p ≥1− =1− > 0.
( 4a b + 21ac + 4a b + 21bc) 2 (5c + 5c)2 100
Therefore,
X
(b − c)2 Sa ≥ (b − c)2 Sa + (c − a)2 S b
 ‹
2
 a 2 b
≥ (b − c) 1 − + (c − a) 1 −
b a
(a − b)2 (a b − c 2 )
= ≥ 0.
ab
p
The equality holds for a = b = c = 1, and also for c = 0 and a = b = 3 (or any cyclic
permutation).
76 Vasile Cîrtoaje

P 1.50. Let a, b, c be nonnegative real numbers such that a b + bc + ca = 3. Prove that


Pp
(a) a(b + c)(a2 + bc) ≥ 6;
p p
a(b + c) a2 + 2bc ≥ 6 3;
P
(b)
p
a(b + c) (a + 2b)(a + 2c) ≥ 18.
P
(c)
(Vasile Cîrtoaje, 2010)

Solution. Assume that a ≥ b ≥ c (b > 0).


(a) Write the inequality in the homogeneous form

a(b + c)(a2 + bc) ≥ 2(a b + bc + ca).

First Solution. Write the homogeneous inequality as


XÆ ”p Æ —
a(b + c) a2 + bc − a(b + c) ≥ 0,
p
X (a − b)(a − c) a(b + c)
p p ≥ 0.
a2 + bc + a(b + c)
Since (c − a)(c − b) ≥ 0, it suffices to show that
p p
(a − b)(a − c) a(b + c) (b − c)(b − a) b(c + a)
p p + p p ≥ 0.
a2 + bc + a(b + c) b2 + ca + b(c + a)

This is true if p p
(a − c) a(b + c) (b − c) b(c + a)
p p ≥p p .
a2 + bc + a(b + c) b2 + ca + b(c + a)
Since Æ Æ
a(b + c) ≥ b(c + a),
it suffices to show that
a−c b−c
p p ≥p p .
a2 + bc + a(b + c) b2 + ca + b(c + a)

Moreover, since
p Æ p Æ
a2 + bc ≥ a(b + c), b2 + ca ≤ b(c + a),

it is enough to show that


a−c b−c
p ≥p .
a2 + bc b2 + ca
Symmetric Nonrational Inequalities 77

Indeed, we have

(a − c)2 (b2 + ca) − (b − c)2 (a2 + bc) = (a − b)(a2 + b2 + c 2 + 3a b − 3bc − 3ca) ≥ 0,

because

a2 + b2 + c 2 + 3a b − 3bc − 3ca = (a2 − bc) + (b − c)2 + 3a(b − c) ≥ 0.


p
The equality holds for a = b = c = 1, and for c = 0 and a = b = 3 (or any cyclic
permutation).
Second Solution. By squaring, the homogeneous inequality becomes
X XÆ
a(b + c)(a2 + bc) + 2 bc(a + b)(a + c)(b2 + ca)(c 2 + a b) ≥ 4(a b + bc + ca)2 .

Since
(b2 + ca)(c 2 + a b) − bc(a + b)(a + c) = a(b + c)(b − c)2 ≥ 0,
it suffices to show that
X X
a(b + c)(a2 + bc) + 2 bc(a + b)(a + c) ≥ 4(a b + bc + ca)2 ,

which is equivalent to X
bc(b − c)2 ≥ 0.
(b) Write the inequality as
X p p
a(b + c) a2 + 2bc ≥ 2(a b + bc + ca) a b + bc + ca,
X ”p p —
a(b + c) a2 + 2bc − a b + bc + ca ≥ 0,
X a(b + c)(a − b)(a − c)
p p ≥ 0.
a2 + 2bc + a b + bc + ca
Since (c − a)(c − b) ≥ 0, it suffices to show that

a(b + c)(a − b)(a − c) b(c + a)(b − c)(b − a)


p p +p p ≥ 0.
a + 2bc + a b + bc + ca
2 b2 + 2ca + a b + bc + ca

This is true if
a(b + c)(a − c) b(c + a)(b − c)
p p ≥p p .
a2 + 2bc + a b + bc + ca b2 + 2ca + a b + bc + ca
Since
(b + c)(a − c) ≥ (c + a)(b − c),
78 Vasile Cîrtoaje

it suffices to show that


a b
p p ≥p p .
a2 + 2bc + a b + bc + ca b2 + 2ca + a b + bc + ca
Moreover, since
p p p p
a2 + 2bc ≥ a b + bc + ca, b2 + 2ca ≤ a b + bc + ca,

it is enough to show that


a b
p ≥p .
a2 + 2bc b2 + 2ca
Indeed, we have

a2 (b2 + 2ca) − b2 (a2 + 2bc) = 2c(a3 − b3 ) ≥ 0.


p
The equality holds for a = b = c = 1, and for c = 0 and a = b = 3 (or any cyclic
permutation).
(c) Write the inequality as follows:
X Æ Æ
a(b + c) (a + 2b)(a + 2c) ≥ 2(a b + bc + ca) 3(a b + bc + ca),
X ”Æ Æ —
a(b + c) (a + 2b)(a + 2c) − 3(a b + bc + ca) ≥ 0,
X a(b + c)(a − b)(a − c)
p p ≥ 0.
(a + 2b)(a + 2c) + 3(a b + bc + ca)
Since (c − a)(c − b) ≥ 0, it suffices to show that
a(b + c)(a − b)(a − c) b(c + a)(b − c)(b − a)
p p +p p ≥ 0.
(a + 2b)(a + 2c) + 3(a b + bc + ca) (b + 2c)(b + 2a) + 3(a b + bc + ca)
This is true if
a(b + c)(a − c) b(c + a)(b − c)
p p ≥p p .
(a + 2b)(a + 2c) + 3(a b + bc + ca) (b + 2c)(b + 2a) + 3(a b + bc + ca)
Since
(b + c)(a − c) ≥ (c + a)(b − c),
it suffices to show that
a b
p p ≥p p .
(a + 2b)(a + 2c) + 3(a b + bc + ca) (b + 2c)(b + 2a) + 3(a b + bc + ca)
Moreover, since
Æ Æ Æ Æ
(a + 2b)(a + 2c) ≥ 3(a b + bc + ca), (b + 2c)(b + 2a) ≤ 3(a b + bc + ca),
Symmetric Nonrational Inequalities 79

it is enough to show that

a b
p ≥p .
(a + 2b)(a + 2c) (b + 2c)(b + 2a)

This is true if p p
a b
p ≥p .
(a + 2b)(a + 2c) (b + 2c)(b + 2a)
Indeed, we have

a(b + 2c)(b + 2a) − b(a + 2b)(a + 2c) = (a − b)(a b + 4bc + 4ca) ≥ 0.


p
The equality holds for a = b = c = 1, and for c = 0 and a = b = 3 (or any cyclic
permutation).

P 1.51. Let a, b, c be nonnegative real numbers such that a b + bc + ca = 3. Prove that


p p p
a bc + 3 + b ca + 3 + c a b + 3 ≥ 6.

(Vasile Cîrtoaje, 2010)

First Solution. Write the inequality in the homogeneous form


X p
a a b + 2bc + ca ≥ 2(a b + bc + ca).

If a = 0, then the inequality turns into


p p p
bc( b − c)2 ≥ 0.

Consider further a, b, c > 0. By squaring, the inequality becomes


X X Æ X X
a b(a2 + b2 ) + 2 bc (bc + 2ca + a b)(ca + 2a b + bc) ≥ 4 a2 b2 + 6a bc a.

Using the Cauchy-Schwarz inequality, we have


Æ Æ
(bc + 2ca + a b)(ca + 2a b + bc) = (a b + bc + ca + ca)(a b + bc + ca + a b)
p
≥ a b + bc + ca + a bc,
and hence
X Æ Xp
bc (bc + 2ca + a b)(ca + 2a b + bc) ≥ (a b + bc + ca)2 + a bc bc.
80 Vasile Cîrtoaje

Thus, it suffices to show that


X Xp X X
a b(a2 + b2 ) + 2(a b + bc + ca)2 + 2a bc bc ≥ 4 a2 b2 + 6a bc a,

which is equivalent to
X Xp X X
a b(a2 + b2 ) + 2a bc bc ≥ 2 a2 b2 + 2a bc a,
X X Xp
a b(a − b)2 ≥ 2a bc( a− bc),
X (a − b)2 Xp p
≥ ( a − b)2 .
c
p p p
Using the substitution x = a, y = b, z = c, the last inequality becomes
X (x 2 − y 2 )2 X
≥ (x − y)2 ,
z2

( y − z)2 A + (z − x)2 B + (x − y)2 C ≥ 0,


where
y +z− x z+x− y x + y −z
A= , B= , C= .
x2 y2 z2
Without loss of generality, assume that x ≥ y ≥ z > 0. Since B > 0 and C > 0, we have

( y − z)2 A + (z − x)2 B + (x − y)2 C ≥ ( y − z)2 A + (z − x)2 B ≥ ( y − z)2 (A + B).

Thus, we only need to show that A + B ≥ 0. Indeed,

y +z− x z+ x − y y +z− x z+ x − y 2z
A+ B = + ≥ + = 2 > 0.
x2 y2 x2 x2 x
p
The equality holds for a = b = c = 1, and for a = 0 and b = c = 3 (or any cyclic
permutation).
Second Solution. Assume that a ≥ b ≥ c, and write the inequality as
X p
a a b + 2bc + ca ≥ 2(a b + bc + ca),
X €p Š
a a b + 2bc + ca − b − c ≥ 0,
X a(a b + ac − b2 − c 2 )
p ≥ 0,
a b + 2bc + ca + b + c
x y z
+ + ≥ 0, (*)
b+c+A c+a+B a+ b+C
Symmetric Nonrational Inequalities 81

where
p p p
A= a b + 2bc + ca, B= bc + 2ca + a b, C= ca + 2a b + bc,

x = a2 (b + c) − a(b2 + c 2 ), y = b2 (c + a) − b(c 2 + a2 ), z = c 2 (a + b) − c(a2 + b2 ).


Since x + y + z = 0, we can write the inequality as
1 1 1 1
 ‹  ‹
x − +z − ≥ 0.
b+c+A c+a+B a+b+C c+a+B
We have

x = a b(a − b) + ac(a − c) ≥ 0, z = ac(c − a) + bc(c − b) ≤ 0.

Therefore, it suffices to show that


1 1 1 1
− ≥ 0, − ≤ 0,
b+c+A c+a+B a+b+C c+a+B
that is,
a − b + B − A ≥ 0, b − c + C − B ≥ 0.
It is enough to prove that A ≤ B ≤ C. Indeed,

B 2 − A2 = c(a − b) ≥ 0, C 2 − B 2 = a(b − c) ≥ 0.

Remark. We can also prove the inequality (*) as follows


X a b(a − b) + ac(a − c)
≥ 0,
b+c+A
X a b(a − b) X ba(b − a)
+ ≥ 0,
b+c+A c+a+B
1 1
X  ‹
a b(a − b) − ≥ 0,
b+c+A c+a+B
X
a b(a + b + C)(a − b)(a − b + B − A) ≥ 0,
X
2
 c 
a b(a + b + C)(a − b) 1 + ≥ 0.
A+ B

P 1.52. Let a, b, c be nonnegative real numbers such that a + b + c = 3. Prove that


p
(b + c) b2 + c 2 + 7bc ≥ 18;
P
(a)
p p
(b + c) b2 + c 2 + 10bc ≤ 12 3.
P
(b)
(Vasile Cîrtoaje, 2010)
82 Vasile Cîrtoaje

Solution. (a) Write the inequality in the equivalent homogeneous forms


X p
(b + c) b2 + c 2 + 7bc ≥ 2(a + b + c)2 ,
X” p —
(b + c) b2 + c 2 + 7bc − b2 − c 2 − 4bc ≥ 0,
X (b + c)2 (b2 + c 2 + 7bc) − (b2 + c 2 + 4bc)2
p ≥ 0,
(b + c) b2 + c 2 + 7bc + b2 + c 2 + 4bc
X bc(b − c)2
p ≥ 0.
(b + c) b2 + c 2 + 7bc + b2 + c 2 + 4bc
3
The equality holds for a = b = c = 1, for a = 0 and b = c = (or any cyclic permuta-
2
tion), and for a = 3 and b = c = 0 (or any cyclic permutation).
(b) Write the inequality as
X Æ
(b + c) 3(b2 + c 2 + 10bc) ≤ 4(a + b + c)2 ,
X” Æ —
2b2 + 2c 2 + 8bc − (b + c) 3(b2 + c 2 + 10bc) ≥ 0,
X 4(b2 + c 2 + 4bc)2 − 3(b + c)2 (b2 + c 2 + 10bc)
p ≥ 0,
2b2 + 2c 2 + 8bc + (b + c) 3(b2 + c 2 + 10bc)
X (b − c)4
p ≥ 0.
2b2 + 2c 2 + 8bc + (b + c) 3(b2 + c 2 + 10bc)
The equality holds for a = b = c = 1.

P 1.53. Let a, b, c be nonnegative real numbers such then a + b + c = 2. Prove that


p p p p
a + 4bc + b + 4ca + c + 4a b ≥ 4 a b + bc + ca.

(Vasile Cîrtoaje, 2012)

Solution. Without loss of generality, assume that c = min{a, b, c}. Using Minkowski’s
inequality gives

p p p
p p Ç p p p p
a + 4bc + b + 4ca ≥ ( a + b)2 + 4c( a + b)2 = ( a + b) 1 + 4c.

Therefore, it suffices to show that


p p p p p
( a + b) 1 + 4c ≥ 4 a b + bc + ca − c + 4a b.
Symmetric Nonrational Inequalities 83

By squaring, this inequality becomes


p Æ
(a + b + 2 a b)(1 + 4c) + 8 (a b + bc + ca)(c + 4a b) ≥ 16(a b + bc + ca) + c + 4a b.
According to Lemma below, it suffices to show that
p
(a + b + 2 a b)(1 + 4c) + 8(2a b + bc + ca) ≥ 16(a b + bc + ca) + c + 4a b,
which is equivalent to
p p
a+ b−c+2 a b + 8c a b ≥ 4(a b + bc + ca).
Write this inequality in the homogeneous form
p p
(a + b + c)(a + b − c + 2 a b) + 16c a b ≥ 8(a b + bc + ca).
p 1
Due to homogeneity, we may assume that a + b = 1. Let us denote d = a b, 0 ≤ d ≤ .
2
We need to show that f (c) ≥ 0 for 0 ≤ c ≤ d, where
f (c) = (1 + c)(1 − c + 2d) + 16cd − 8d 2 − 8c
= (1 − 2d)(1 + 4d) + 2(9d − 4)c − c 2 .
Since f (c) is concave, it suffices to show that f (0) ≥ 0 and f (d) ≥ 0. Indeed,
f (0) = (1 − 2d)(1 + 4d) ≥ 0,
f (d) = (3d − 1)2 ≥ 0.
Thus, the proof is completed. The equality holds for a = b = 1 and c = 0 (or any cyclic
permutation).
Lemma (by Nguyen Van Quy). Let a, b, c be nonnegative real numbers such then
c = min{a, b, c}, a + b + c = 2.
Then, Æ
(a b + bc + ca)(c + 4a b) ≥ 2a b + bc + ca.
Proof. By squaring, the inequality becomes
c[a b + bc + ca − c(a + b)2 ] ≥ 0.
We need to show that
(a + b + c)(a b + bc + ca) − 2c(a + b)2 ≥ 0.
We have
(a + b + c)(a b + bc + ca) − 2c(a + b)2 ≥ (a + b)(b + c)(c + a) − 2c(a + b)2
= (a + b)(a − c)(b − c) ≥ 0.
84 Vasile Cîrtoaje

P 1.54. If a, b, c are nonnegative real numbers, then


p p p p
a2 + b2 + 7a b + b2 + c 2 + 7bc + c 2 + a2 + 7ca ≥ 5 a b + bc + ca.

(Vasile Cîrtoaje, 2012)

Solution (by Nguyen Van Quy). Assume that c = min{a, b, c}. Using Minkowski’s in-
equality yields
p
p p Ç p
b + c + 7bc + a + c + 7ca ≥ (a + b)2 + 4c 2 + 7c( a + b)2 .
2 2 2 2

Therefore, it suffices to show that


p
Ç p p p
(a + b)2 + 4c 2 + 7c( a + b)2 ≥ 5 a b + bc + ca − a2 + b2 + 7a b.

By squaring, this inequality becomes


p Æ
2c 2 + 7c a b + 5 (a2 + b2 + 7a b)(a b + bc + ca) ≥ 15a b + 9c(a + b).

Due to homogeneity, we may assume that a + b = 1. Let us denote x = a b. We need to


1
show that f (x) ≥ 0 for c 2 ≤ x ≤ , where
4
p Æ
f (x) = 2c 2 + 7c x + 5 (1 + 5x)(c + x) − 15x − 9c.

Since
−7c 5(5c − 1)2
f 00 (x) = p − p <0
4 x 3 4 [5x 2 + (5c + 1)x + c]3
1
 ‹
f (c) is concave. Thus, it suffices to show that f (c 2 ) ≥ 0 and f ≥ 0.
4
Write the inequality f (c 2 ) ≥ 0 as
Æ
5 (1 + 5c 2 )(c + c 2 ) ≥ 6c 2 + 9c.

By squaring, this inequality turns into

c(89c 3 + 17c 2 − 56c + 25) ≥ 0,

which is true since

89c 3 + 17c 2 − 56c + 25 ≥ 12c 2 − 56c + 25 = (1 − 2c)(25 − 6c) ≥ 0.


1
 ‹
Write the inequality f ≥ 0 as
4
p
8c 2 − 22c + 15( 4c + 1 − 1) ≥ 0.
Symmetric Nonrational Inequalities 85

p
Making the substitution t = 4c + 1, t ≥ 1, the inequality becomes

(t − 1)(t 3 + t 2 − 12t + 18) ≥ 0.

This is true since

t 3 + t 2 − 12t + 18 ≥ 2t 2 − 12t + 18 = 2(t − 3)2 ≥ 0.

Thus, the proof is completed. The equality holds for a = b and c = 0 (or any cyclic
permutation).

P 1.55. If a, b, c are nonnegative real numbers, then


p p p Æ
a2 + b2 + 5a b + b2 + c 2 + 5bc + c 2 + a2 + 5ca ≥ 21(a b + bc + ca).

(Nguyen Van Quy, 2012)

Solution. Without loss of generality, assume that c = min{a, b, c}. Using Minkowski’s
inequality, we have

p
Æ Æ Ç p
(a + c)2 + 3ac + (b + c)2 + 3bc ≥ (a + b + 2c)2 + 3c( a + b)2 .

Therefore, it suffices to show that

p
Ç p Æ p
(a + b + 2c)2 + 3c( a + b)2 ≥ 21(a b + bc + ca) − a2 + b2 + 5a b.

By squaring, this inequality becomes


p Æ
2c 2 + 3c ab + 21(a2 + b2 + 5a b)(a b + bc + ca) ≥ 12a b + 7c(a + b).

Due to homogeneity, we may assume that a + b = 1. Let us denote x = a b. We need to


1
show that f (x) ≥ 0 for c 2 ≤ x ≤ , where
4
p Æ
f (x) = 2c 2 + 3c x + 21(1 + 3x)(c + x) − 12x − 7c.

Since p
00 −3c 21(3c − 1)2
f (x) = p − p <0
4 x 3 4 [3x 2 + (3c + 1)x + c]3
1
 ‹
f (c) is concave. Thus, it suffices to show that f (c 2 ) ≥ 0 and f ≥ 0.
4
86 Vasile Cîrtoaje

Write the inequality f (c 2 ) ≥ 0 as


Æ
21(1 + 3c 2 )(c + c 2 ) ≥ 7(c + c 2 ).

By squaring, this inequality turns into

c(c + 1)(1 − 2c)(3 − c) ≥ 0,

which is clearly true.  ‹


1
Write the inequality f ≥ 0 as
4
Æ
8c 2 − 22c + 7 3(4c + 1) − 12 ≥ 0.

1
Using the substitution 3t 2 = 4c + 1, t ≥ p , the inequality becomes
3

(t − 1)2 (3t 2 + 6t − 4) ≥ 0.

This is true since p


3t 2 + 6t − 4 ≥ 1 + 2 3 − 4 > 0.
Thus, the proof is completed. The equality holds for a = b = c.

P 1.56. Let a, b, c be nonnegative real numbers such that a b + bc + ca = 3. Prove that


v
p p p t2
a a2 + 5 + b b2 + 5 + c c2 + 5 ≥ (a + b + c)2 .
3

(Vasile Cîrtoaje, 2010)

Solution. Write the inequality in the homogeneous form


X Æ p
a 3a2 + 5(a b + bc + ca) ≥ 2 (a + b + c)2 .

Due to homogeneity, we may assume that

a b + bc + ca = 1.

By squaring, the inequality becomes


X X Æ X X X
a4 + 2 bc (3b2 + 5)(3c 2 + 5) ≥ 12 a2 b2 + 19a bc a+3 a b(a2 + b2 ).
Symmetric Nonrational Inequalities 87

Applying Lemma below for x = 3b2 , y = 3c 2 and d = 5, we have


Æ 9 2
2 (3b2 + 5)(3c 2 + 5) ≥ 3(b2 + c 2 ) + 10 − (b − c 2 )2 ,
20
hence
Æ 9
2bc (3b2 + 5)(3c 2 + 5) ≥ 3bc(b2 + c 2 ) + 10bc − bc(b2 − c 2 )2 ,
20
X Æ X X 9 X
2 bc (3b2 + 5)(3c 2 + 5) ≥ 3 bc(b2 + c 2 ) + 10( bc)2 − bc(b2 − c 2 )2
20
X X X 9 X
= 10 a2 b2 + 20a bc a+3 a b(a2 + b2 ) − bc(b2 − c 2 )2 .
20
Therefore, it suffices to show that
X X X X 9 X
a4 + 10 a2 b2 + 20a bc a+3 a b(a2 + b2 ) − bc(b2 − c 2 )2 ≥
20
X X X
≥ 12 a2 b2 + 19a bc a+3 a b(a2 + b2 ),
which is equivalent to
X X X 9 X
a4 − 2 a2 b2 + a bc a− bc(b2 − c 2 )2 ≥ 0.
20
Since
€X X X Š €X X Š €X X Š
2 a4 − 2 a2 b2 + a bc a =2 a4 − a2 b2 − 2 a2 b2 − a bc a
X X
= (b2 − c 2 )2 − a2 (b − c)2 ,
we can write the inequality as
X
(b − c)2 Sa ≥ 0,

where
9
Sa = (b + c)2 − a2 − bc(b + c)2 .
10
In addition, since

bc(b + c)2
Sa ≥ (b + c)2 − a2 − bc(b + c)2 = (b + c)2 − a2 − ,
a b + bc + ca
a(b + c)3 − a2 (a b + bc + ca)
= ,
a b + bc + ca
it is enough to show that X
(b − c)2 Ea ≥ 0,
88 Vasile Cîrtoaje

where
Ea = a(b + c)3 − a2 (a b + bc + ca).

Assume that a ≥ b ≥ c, b > 0. Since

E b = b(c + a)3 − b2 (a b + bc + ca) ≥ b(c + a)3 − b2 (c + a)(c + b)

≥ b(c + a)3 − b2 (c + a)2 = b(c + a)2 (c + a − b) ≥ 0,

Ec = c(a + b)3 − c 2 (a b + bc + ca) ≥ c(a + b)3 − c 2 (a + b)(b + c)

≥ c(a + b)3 − c 2 (a + b)2 = c(a + b)2 (a + b − c) ≥ 0

and

Ea E b (b + c)3 (c + a)3
+ = + − 2(a b + bc + ca)
a2 b2 a b
b3 + 2b2 c a3 + 2a2 c
≥ + − 2(a b + bc + ca)
a b
(a2 − b2 )2 + 2c(a + b)(a − b)2
= ≥ 0,
ab
we get

2 Ea Eb
X  ‹
2 2 2 2
(b − c) Ea ≥ (b − c) Ea + (a − c) E b ≥ a (b − c) + ≥ 0.
a2 b2
p
The equality holds for a = b = c = 1, and also for a = b = 3 and c = 0 (or any cyclic
permutation).
Lemma. If x ≥ 0, y ≥ 0 and d > 0, then
Æ 1
2 (x + d)( y + d) ≥ x + y + 2d − (x − y)2 .
4d

Proof. We have
Æ 2(d x + d y + x y) 2(d x + d y + x y)
2 (x + d)( y + d) − 2d = p ≥ (x+d)+( y+d)
(x + d)( y + d) + d 2 +d

4(d x + d y + x y) (x − y)2 (x − y)2


= =x+y− ≥x+y− .
x + y + 4d x + y + 4d 4d
Symmetric Nonrational Inequalities 89

P 1.57. Let a, b, c be nonnegative real numbers such that a2 + b2 + c 2 = 1. Prove that


p p p
a 2 + 3bc + b 2 + 3ca + c 2 + 3a b ≥ (a + b + c)2 .

(Vasile Cîrtoaje, 2010)

Solution. Let q = a b + bc + ca. Write the inequality as


X p
a 2 + 3bc ≥ 1 + 2q.

By squaring, the inequality becomes


X X Æ
1 + 3a bc a+2 bc (2 + 3a b)(2 + 3ac) ≥ 4q + 4q2 .

Applying Lemma from the preceding P 1.56 for x = 3a b, y = 3ac2 and d = 2, we have
Æ 9
2 (2 + 3a b)(2 + 3ac) ≥ 3a(b + c) + 4 − a2 (b − c)2 ,
8
X Æ X X 9 X
2 bc (2 + 3a b)(2 + 3ac) ≥ 3a bc (b + c) + 4 bc − a bc a(b − c)2
8
X 9 X
= 6a bc a + 4q − a bc a(b − c)2 .
8
Therefore, it suffices to show that
X X 9 X
1 + 3a bc a + 4q + 6a bc a − a bc a(b − c)2 ≥ 4q + 4q2 ,
8
which is equivalent to
X 9 X
1 + 9a bc a − 4q2 ≥ a bc a(b − c)2 .
8
Since X
a4 + b4 + c 4 = 1 − 2(a2 b2 + b2 c 2 + c 2 a2 ) = 1 − 2q2 + 4a bc a,
from Schur’s inequality of fourth degree
X €X Š €X Š
a4 + b4 + c 4 + 2a bc a≥ a2 ab ,

we get X
1 ≥ 2q2 + q − 6a bc a.
Thus, it is enough to prove that
X X 9 X
(2q2 + q − 6a bc a) + 9a bc a − 4q2 ≥ a bc a(b − c)2 ;
8
90 Vasile Cîrtoaje

that is, X X
8(q − 2q2 + 3a bc a) ≥ 9a bc a(b − c)2 .

Since X €X Š €X Š €X Š2 X
q − 2q2 + 3a bc a= a2 ab − 2 a b + 3a bc a
X X X
= bc(b2 + c 2 ) − 2 b2 c 2 = bc(b − c)2 ,

we need to show that X


bc(8 − 9a2 )(b − c)2 ≥ 0.

Also, since
8 − 9a2 = 8(b2 + c 2 ) − a2 ≥ b2 + c 2 − a2 ,

it suffices to prove that


X
bc(b2 + c 2 − a2 )(b − c)2 ≥ 0.

Assume that a ≥ b ≥ c. It is enough to show that

bc(b2 + c 2 − a2 )(b − c)2 + ca(c 2 + a2 − b2 )(c − a)2 ≥ 0.

This is true if
a(c 2 + a2 − b2 )(a − c)2 ≥ b(a2 − b2 − c 2 )(b − c)2 .

For the non-trivial case a2 − b2 − c 2 ≥ 0, this inequality follows from

a ≥ b, c 2 + a2 − b2 ≥ a2 − b2 − c 2 , (a − c)2 ≥ (b − c)2 .

1 1
The equality holds for a = b = c = p , and for a = 0 and b = c = p (or any cyclic
3 2
permutation).

P 1.58. Let a, b, c be nonnegative real numbers such that a + b + c = 3. Prove that


v v v
t 2a + bc t 2b + ca t 2c + a b
(a) a +b +c ≥ 3;
3 3 3
v v v
t a(1 + b + c) t b(1 + c + a) t c(1 + a + b)
(b) a +b +c ≥ 3.
3 3 3
(Vasile Cîrtoaje, 2010)
Symmetric Nonrational Inequalities 91

Solution. (a) If two of a, b, c are zero, then the inequality is trivial. Otherwise, by
Hölder’s inequality, we have

X t 2a + bc 2
‚ v Œ
( a)3
P
9
a ≥P =P a .
3 3a
2a + bc 2a + bc

Therefore, it suffices to show that


X a
≤ 1.
2a + bc
Since
2a bc
=1− ,
2a + bc 2a + bc
we can write this inequality as
X bc
≥ 1.
2a + bc
By the Cauchy-Schwarz inequality, we have

( bc)2 ( bc)2
P P
X bc
≥P = = 1.
2a + bc bc(2a + bc) 2a bc a + b2 c 2
P P

3
The equality holds for a = b = c = 1, and for a = 0 and b = c = (or any cyclic
2
permutation).
(b) Write the inequality in the homogeneous form
X Æ
a a(a + 4b + 4c) ≥ (a + b + c)2 .

By squaring, the inequality becomes


X Æ X X
bc bc(b + 4c + 4a)(c + 4a + 4b) ≥ 3 a2 b2 + 6a bc a.

Applying the Cauchy-Schwarz inequality, we have


Æ Æ
(b + 4c + 4a)(c + 4a + 4b) = (4a + b + c + 3c)(4a + b + c + 3b)
p
≥ 4a + b + c + 3 bc,
hence Æ p
bc bc(b + 4c + 4a)(c + 4a + 4b) ≥ (4a + b + c)bc bc + 3b2 c 2 ,
X Æ X p X
bc bc(b + 4c + 4a)(c + 4a + 4b) ≥ (4a + b + c)bc bc + 3 b2 c 2 .
92 Vasile Cîrtoaje

Thus, it is enough to show that


X p X
(4a + b + c)bc bc ≥ 6a bc a.

Replacing a, b, c by a2 , b2 , c 2 , respectively, this inequality becomes


X X
(4a2 + b2 + c 2 )b3 c 3 ≥ 6a2 b2 c 2 a2 ,
€X Š €X Š X X
a2 b3 c 3 + 3a2 b2 c 2 bc ≥ 6a2 b2 c 2 a2 ,
€X Š €X Š €X X Š
a2 a3 b3 − 3a2 b2 c 2 ≥ 3a2 b2 c 2 a2 − ab .
Since
X €X Š €X X Š 1 €X Š X
a3 b3 − 3a2 b2 c 2 = ab a2 b2 − a bc a = ab a2 (b − c)2 ,
2
and X X 1X
a2 − ab = (b − c)2 ,
2
we can write the inequality as
X
(b − c)2 Sa ≥ 0,

where €X Š €X Š
Sa = a 2 a2 a b − 3a2 b2 c 2 .
Assume that a ≥ b ≥ c. Since Sa ≥ S b ≥ 0 and
€X Š €X Š
S b + Sc = (b2 + c 2 ) a2 a b − 6a2 b2 c 2
€X Š €X Š
≥ 2bc a2 a b − 6a2 b2 c 2
€X Š
≥ 2bca2 a b − 6a2 b2 c 2 = 2a2 bc(a b + ac − 2bc) ≥ 0,

we get X
(b − c)2 Sa ≥ (c − a)2 S b + (a − b)2 Sc ≥ (a − b)2 (S b + Sc ) ≥ 0.
3
The equality holds for a = b = c = 1, and for a = 0 and b = c = (or any cyclic
2
permutation).

P 1.59. If a, b, c are nonnegative real numbers such that a + b + c = 3, then


Æ Æ Æ
8(a2 + bc) + 9 + 8(b2 + ca) + 9 + 8(c 2 + a b) + 9 ≥ 15.

(Vasile Cîrtoaje, 2013)


Symmetric Nonrational Inequalities 93

Solution. Let q = a b + bc + ca and

A = (3a − b − c)2 + 8q, B = (3b − c − a)2 + 8q, C = (3c − a − b)2 + 8q.

Since

8(a2 + bc) + 9 = 8(a2 + q) + 9 − 8a(b + c) = 8(a2 + q) + 9 − 8a(3 − a)


= (4a − 3)2 + 8q = (3a − b − c)2 + 8q = A,

we can rewrite the inequality as follows


Xp
A ≥ 15,
X p
[ A − (3a + b + c)] ≥ 0,
X 2bc − ca − a b
p ≥ 0,
A + 3a + b + c
X• b(c − a) c(b − a)
˜
p + p ≥ 0,
A + 3a + b + c A + 3a + b + c
X c(a − b) X c(b − a)
p + p ≥ 0,
B + 3b + c + a A + 3a + b + c
X p p p
c(a − b)( C + 3c + a + b)[ A − B + 2(a − b)] ≥ 0,
p 4(a + b − c)
X • ˜
c(a − b)2 ( C + 3c + a + b) p p + 1 ≥ 0.
A+ B
Without loss of generality, assume that a ≥ b ≥ c. Since a + b − c > 0, it suffices to show
that
p 4(c + a − b)
• ˜
2
b(a − c) ( B + 3b + c + a) p p +1 ≥
A+ C
p 4(a − b − c)
• ˜
a(b − c)2 ( A + 3a + b + c) p p − 1 .
B+ C
This inequality follows from the inequalities

b2 (a − c)2 ≥ a2 (b − c)2 ,
p p
a( B + 3b + c + a) ≥ b( A + 3a + b + c),
4(c + a − b) 4(a − b − c)
p p +1≥ p p − 1.
A+ C B+ C
Write the second inequality as

a2 B − b2 A
p p + (a − b)(a + b + c) ≥ 0.
a B+b A
94 Vasile Cîrtoaje

Since

a2 B − b2 A = (a − b)(a + b + c)(a2 + b2 − 6a b + bc + ca) + 8q(a2 − b2 )


≥ (a − b)(a + b + c)(a2 + b2 − 6a b) ≥ −4a b(a − b)(a + b + c),

it suffices to show that


−4a b
p p + 1 ≥ 0.
a B+b A
p p p p
Indeed, since A > 8q ≥ 2 a b and B ≥ 8q ≥ 2 a b, we have
p p

p p p p p
a B + b A − 4a b > 2(a + b) a b − 4a b = 2 a b(a + b − 2 a b) ≥ 0.

The third inequality holds if


2(a − b − c)
1≥ p p .
B+ C
p p
Clearly, it suffices to show that B ≥ a and C ≥ a. We have

B − a2 = 8q − 2a(3b − c) + (3b − c)2 ≥ 8a b − 2a(3b − c) = 2a(b + c) ≥ 0

and

C − a2 = 8q − 2a(3c − b) + (3c − b)2 ≥ 8a b − 2a(3c − b) = 2a(5b − 3c) ≥ 0.

The equality holds for a = b = c = 1, and also for a = 3 and b = c = 0 (or any cyclic
permutation).

9
P 1.60. Let a, b, c be nonnegative real numbers such that a + b + c = 3. If k ≥ , then
8
p p p p
a2 + bc + k + b2 + ca + k + c 2 + a b + k ≥ 3 2 + k.

Solution. We will show that


XÆ XÆ Æ
8(a2 + bc + k) ≥ (3a + b + c)2 + 8k − 9 ≥ 6 2(k + 2).

The right inequality is equivalent to


XÆ Æ
(2a + 3)2 + 8k − 9 ≥ 6 2(k + 2),

and follows immediately from Jensen’s inequality applied to the convex function f :
[0, ∞) → R defined by Æ
f (x) = (2x + 3)2 + 8k − 9.
Symmetric Nonrational Inequalities 95

Using the substitution

A1 = 8(a2 + bc + k), B1 = 8(b2 + ca + k), C1 = 8(c 2 + a b + k),

A2 = (3a + b + c)2 + 8k − 9, B2 = (3b + c + a)2 + 8k − 9, C2 = (3c + a + b)2 + 8k − 9,


we can write the left inequality as follows:
A1 − A2 B1 − B2 C1 − C2
p +p p +p p ≥ 0,
A1 + A2 B1 + B2 C1 + C2
p

2bc − ca − a b 2ca − a b − bc 2a b − bc − ca
+ p + p ≥ 0,
A1 + A2 B1 + B2 C1 + C2
p p p p

X  b(c − a) c(b − a)

p +p ≥ 0,
A1 + A2 A1 + A2
p p

X c(a − b) c(b − a)
p +p p ≥ 0,
B1 + B2 A1 + A2
p
X
c(a − b)( C1 + C2 )[( A1 − B1 ) + ( A2 − B2 )] ≥ 0,
p p p p p p

2(a + b − c) 2a + 2b + c
X  
2
c(a − b) ( C1 + C2 ) p p +p
p p
≥ 0.
A1 + B1 A2 + B2
p

Without loss of generality, assume that a ≥ b ≥ c. Clearly, the desired inequality is true
for b + c ≥ a. Consider further the case b + c < a. Since a + b − c > 0, it suffices to show
that

2(b + c − a) 2b + 2c + a
 
2
a(b − c) ( A1 + A2 ) p p +p +
p p
B1 + C1 B2 + C2
p

2(c + a − b) 2c + 2a + b
 
2
+b(a − c) ( B1 + B2 ) p p +p
p p
≥ 0.
C1 + A1 C2 + AC2
p

Since
b2 (a − c)2 ≥ a2 (b − c)2 ,
it suffices to show that

2(b + c − a) 2b + 2c + a
 
b( A1 + A2 ) p p +p +
p p
B1 + C1 B2 + C2
p

2(c + a − b) 2c + 2a + b
 
+a( B1 + B2 ) p p +p
p p
≥ 0.
C1 + A1 C2 + A2
p

From
a2 B1 − b2 A1 = 8c(a3 − b3 ) + 8k(a2 − b2 ) ≥ 0
96 Vasile Cîrtoaje

and

a2 B2 − b2 A2 = (a − b)(a + b + c)(a2 + b2 + 6a b + bc + ca) + (8k − 9)(a2 − b2 ) ≥ 0,


p p p p
we get a B1 ≥ b A1 and a B2 ≥ b A2 , hence

a( B1 + B2 ) ≥ b( A1 + A2 ).
p p p p

Therefore, it is enough to show that

2(b + c − a) 2b + 2c + a 2(c + a − b) 2c + 2a + b
p +p p +p p +p p ≥ 0.
B1 + C1 B2 + C2 C1 + A1 C2 + A2
p

This is true if
2b −2b
p +p p ≥0
B1 + C1 C1 + A1
p

and
−2a 2a 2a
p +p p +p p ≥ 0.
B1 + C1 C1 + A1 C2 + A2
p

The first inequality is true because A1 − B1 = 8(a − b)(a + b − c) ≥ 0. The second


inequality can be written as

1 1 1
+p ≥p .
C1 + C2 + B1 +
p p p p
A1 A2 C1

Since
1 1 4
+p ≥p ,
C1 + C2 + C1 + A1 + C2 +
p p p p p p
A1 A2 A2
it suffices to show that

B1 + 3 C1 ≥ A1 + A2 + C2 .
p p p p p
4

Taking into account of


C1 − C2 = 4(2a b − bc − ca) ≥ 0,
C1 − B1 = 8(b − c)(a − b − c) ≥ 0,
A2 − A1 = 4(a b − 2bc + ca) ≥ 0,
we have

B1 + 3 B1 + 2
p p p p p p p p p
4 C1 − A1 − A2 − C2 ≥ 4 C1 − A1 − A2
B1 + 2 B1 −
p p p p
≥4 A2 − A2
= 2(3 B1 − A2 ).
p p
Symmetric Nonrational Inequalities 97

In addition,

9B1 − A2 = 64k − 8a2 + 72b2 − 4a b + 68ac


≥ 72 − 8a2 + 72b2 − 4a b + 68ac
= 8(a + b + c)2 − 8a2 + 72b2 − 4a b + 68ac
= 4(20b2 + 2c 2 + 3a b + 4bc + 21ac) ≥ 0.

Thus, the proof is completed. The equality holds for a = b = c = 1. If k = 9/8, then the
equality holds also for a = 3 and b = c = 0 (or any cyclic permutation).

P 1.61. If a, b, c are nonnegative real numbers such that a + b + c = 3, then


p p p p
a3 + 2bc + b3 + 2ca + c 3 + 2a b ≥ 3 3.

(Nguyen Van Quy, 2013)

Solution. Since
(a3 + 2bc)(a + 2bc) ≥ (a2 + 2bc)2 ,
it suffices to prove that
X a2 + 2bc p
p ≥ 3 3.
a + 2bc
By Hölder’s inequality, we have
2 X
a + 2bc
X 2
”X —3
p (a2 + 2bc)(a + 2bc) ≥ (a2 + 2bc) = (a + b + c)6 .
a + 2bc

Therefore, it suffices to show that


X
(a + b + c)4 ≥ 3 (a2 + 2bc)(a + 2bc).

which is equivalent to
X
(a + b + c)4 ≥ (a2 + 2bc)(a2 + 6bc + ca + a b).

Indeed,
X X
(a + b + c)4 − (a2 + 2bc)(a2 + 6bc + ca + a b) = 3 a b(a − b)2 ≥ 0.

The equality holds for a = b = c = 1, and also for a = 3 and b = c = 0 (or any cyclic
permutation).
98 Vasile Cîrtoaje

P 1.62. If a, b, c are positive real numbers, then


p p p p
a2 + bc b2 + ca c2 + a b 3 2
+ + ≥ .
b+c c+a a+b 2
(Vasile Cîrtoaje, 2006)

Solution. According to the well-known inequality

(x + y + z)2 ≥ 3(x y + yz + z x), x, y, z ≥ 0,

it suffices to show that p


X (b2 + ca)(c 2 + a b 3
≥ .
(c + a)(a + b) 2
Replacing a, b, c by a2 , b2 , c 2 , respectively, the inequality becomes
X Æ
2 (b2 + c 2 ) (b4 + c 2 a2 )(c 4 + a2 b2 ) ≥ 3(a2 + b2 )(b2 + c 2 )(c 2 + a2 ).

Multiplying the Cauchy-Schwarz inequalities


Æ
(b2 + c 2 )(b4 + c 2 a2 ) ≥ b3 + ac 2 ,
Æ
(c 2 + b2 )(c 4 + a2 b2 ) ≥ c 3 + a b2 ,
we get Æ
(b2 + c 2 ) (b4 + c 2 a2 )(c 4 + a2 b2 ) ≥ (b3 + ac 2 )(c 3 + a b2 )

= b3 c 3 + a(b5 + c 5 ) + a2 b2 c 2 .
Therefore, it suffices to show that
X X
2 b3 c 3 + 2 a(b5 + c 5 ) + 6a2 b2 c 2 ≥ 3(a2 + b2 )(b2 + c 2 )(c 2 + a2 ).

This inequality is equivalent to


X X X
2 b3 c 3 + 2 bc(b4 + c 4 ) ≥ 3 b2 c 2 (b2 + c 2 ),
X
bc[2b2 c 2 + 2(b4 + c 4 ) − 3bc(b2 + c 2 )] ≥ 0,
X
bc(b − c)2 (2b2 + bc + 2c 2 ) ≥ 0.
The equality holds for a = b = c.
Symmetric Nonrational Inequalities 99

P 1.63. If a, b, c are nonnegative real numbers, no two of which are zero,then


p p p
bc + 4a(b + c) ca + 4b(c + a) a b + 4c(a + b) 9
+ + ≥ .
b+c c+a a+b 2
(Vasile Cîrtoaje, 2006)
Solution. Let us denote

A = bc + 4a(b + c), B = ca + 4b(c + a), C = a b + 4c(a + b).

By squaring, the inequality becomes


p
X A X BC 81
+2 ≥ .
(b + c)2 (c + a)(a + b) 4
Further, we use the following identity due to Sung-Yoon Kim:

(b + c)2 BC − 4[a(b2 + c 2 ) + 2bc(b + c) + 3a bc]2 = a bc(b − c)2 (a + 4b + 4c),

which yields
p 2a(b2 + c 2 ) + 4bc(b + c) + 6a bc
BC ≥ ,
b+c
p
4 a(b2 + c 2 ) + 8 bc(b + c) + 36a bc
P P
X BC
2 ≥ ,
(c + a)(a + b) (a + b)(b + c)c + a)
p
12 bc(b + c) + 36a bc
P
X BC
2 ≥ .
(c + a)(a + b) (a + b)(b + c)c + a)
On the other hand, according to the known inequality Iran-1996,
X a b + bc + ca 9
≥ ,
(b + c) 2 4
(see Remark from the proof of P 1.70), we have
X A X a b + bc + ca X a 9 X a
= + 3 ≥ + 3 .
(b + c)2 (b + c)2 b+c 4 b+c
Thus, it suffices to show that
12 bc(b + c) + 36a bc
P
X a
3 + ≥ 18.
b+c (a + b)(b + c)c + a)
This is equivalent to Schur’s inequality of degree three
X X
a3 + 3a bc ≥ bc(b + c).

The equality holds for a = b = c, and also for a = 0 and b = c (or any cyclic permuta-
tion).
100 Vasile Cîrtoaje

P 1.64. If a, b, c are nonnegative real numbers, no two of which are zero,then


p p p
a a2 + 3bc b b2 + 3ca c c 2 + 3a b
+ + ≥ a + b + c.
b+c c+a a+b
(Cezar Lupu, 2006)

Solution. Using the AM-GM inequality, we have


p
a a2 + 3bc 2a(a2 + 3bc) 2a(a2 + 3bc) 2a3 + 6a bc
= p ≥ = ,
b+c 2 (b + c)2 (a2 + 3bc) (b + c)2 + (a2 + 3bc) S + 5bc

where S = a2 + b2 + c 2 . Thus, it suffices to show that


X 2a3 + 6a bc
≥ a + b + c.
S + 5bc
Write this inequality as
X  2a2 + 6bc 
a − 1 ≥ 0,
S + 5bc
or, equivalently,
AX + BY + X Z ≥ 0,
where
1 1 1
A= , B= , C= ,
S + 5bc S + 5ca S + 5a b
X = a3 + a bc − a(b2 + c 2 ), Y = b3 + a bc − b(c 2 + a2 ), Z = c 3 + a bc − c(a2 + b2 ).
Without loss of generality, assume that a ≥ b ≥ c. We have

A ≥ B ≥ C,

X = a(a2 − b2 ) + ac(b − c) ≥ 0, Z = c(c 2 − b2 ) + ac(b − a) ≤ 0


and, according to Schur’s inequality of third degree,
X X
X +Y +Z = a3 + 3a bc − a(b2 + c 2 ) ≥ 0.

Therefore,
AX + BY + C Z ≥ BX + BY + BZ = B(X + Y + Z) ≥ 0.
The equality holds for a = b = c, and also for a = 0 and b = c (or any cyclic permuta-
tion).
Symmetric Nonrational Inequalities 101

P 1.65. If a, b, c are nonnegative real numbers, no two of which are zero,then


v v v
t 2a(b + c) t 2b(c + a) t 2c(a + b)
+ + ≥ 2.
(2b + c)(b + 2c) (2c + a)(c + 2a) (2a + b)(a + 2b)
(Vasile Cîrtoaje, 2006)
p p p
Solution. Making the substitution x = a, y = b, z = c, the inequality becomes
v
X t 2( y 2 + z 2 )
x ≥ 2.
(2 y 2 + z 2 )( y 2 + 2z 2 )
We claim that v
t 2( y 2 + z 2 ) y +z
≥ 2 .
(2 y 2 + z )( y + 2z )
2 2 2 y + yz + z 2
Indeed, be squaring and direct calculation, this inequality reduces to y 2 z 2 ( y − z)2 ≥ 0.
Thus, it suffices to show that
X x( y + z)
≥ 2,
y2 + yz + z 2
which is just the inequality in P 1.67. The equality holds for a = b = c, and also for
a = 0 and b = c (or any cyclic permutation).

P 1.66. If a, b, c are nonnegative real numbers such that a b + bc + ca = 3, then


v s v v s v
t bc ca t ab t bc ca t ab
+ + ≤ 1 ≤ + + .
3a2 + 6 3b2 + 6 3c 2 + 6 6a2 + 3 6b2 + 3 6c 2 + 3
(Vasile Cîrtoaje, 2011)

Solution. By the Cauchy-Schwarz inequality, we have


‚ v Œ2
a b + bc + ca
X ‹ X ‹
Xt bc bc
≤ ,
3a2 + 6 3a2 + 6 a b + bc + ca

hence ‚ v Œ2
Xt bc X 1
≤ .
3a + 6
2 a2 +2
Therefore, to prove the original left inequality, it suffices to show that
X 1
≤ 1.
a2 +2
102 Vasile Cîrtoaje

This inequality is equivalent to


a2
X
≥ 1.
a2 + 2
Indeed, by the Cauchy-Schwarz inequality, we get
X a2 (a + b + c)2 (a + b + c)2
≥ = = 1.
a2 + 2 (a2 + 2) a2 + 6
P P

The equality occurs for a = b = c = 1. By Hölder’s inequality, we have


‚ v Œ2
X t bc ”X — €X Š3
2 2 2
b c (6a + 3) ≥ bc .
6a2 + 3

To prove the original right inequality, it suffices to show that


X
(a b + bc + ca)3 ≥ b2 c 2 (6a2 + a b + bc + ca),
which is equivalent to
X
(a b + bc + ca)[(a b + bc + ca)2 − b2 c 2 ] ≥ 18a2 b2 c 2 ,

2a bc(a b + bc + ca)(a + b + c) ≥ 18a2 b2 c 2 ,


X
2a bc a(b − c)2 ≥ 0.
The equality occurs for a = b = c = 1, and for a = 0 and bc = 3 (or any cyclic
permutation).

P 1.67. Let a, b, c be nonnegative real numbers such that a b + bc + ca = 3. If k > 1, than


a k (b + c) + b k (c + a) + c k (a + b) ≥ 6.

Solution. Let
E = a k (b + c) + b k (c + a) + c k (a + b).
We consider two cases.
Case 1: k ≥ 2. Applying Jensen’s inequality to the convex function f (x) = x k−1 , x ≥ 0,
we get
E = (a b + ac)a k−1 + (bc + ba)b k−1 + (ca + c b)c k−1
(a b + ac)a + (bc + ba)b + (ca + c b)c k−1
• ˜
≥ 2(a b + bc + ca)
2(a b + bc + ca)
k−1
a (b + c) + b2 (c + a) + c 2 (a + b)
 2
=6 .
6
Symmetric Nonrational Inequalities 103

Thus, it suffices to show that

a2 (b + c) + b2 (c + a) + c 2 (a + b) ≥ 6.

Write this inequality as

(a b + bc + ca)(a + b + c) − 3a bc ≥ 6,

a + b + c ≥ 2 + a bc.
It is true since Æ
a+b+c ≥ 3(a b + bc + ca) = 3
and ‹3
a+b+c

a bc ≤ = 1.
3

Case 2: 1 < k ≤ 2. We have

E = a k−1 (3 − bc) + b k−1 (3 − ca) + c k−1 (3 − a b)

= 3(a k−1 + b k−1 + c k−1 ) − a k−1 b k−1 c k−1 (a b)2−k + (bc)2−k + (ca)2−k .
 

Since 0 ≤ 2 − k < 1, f (x) = x 2−k is concave for x ≥ 0. Thus, by Jensen’s inequality, we


have
a b + bc + ca 2−k
 ‹
(a b)2−k + (bc)2−k + (ca)2−k ≤ 3 = 3,
3
and hence
E ≥ 3(a k−1 + b k−1 + c k−1 ) − 3a k−1 b k−1 c k−1 .
Consequently, it suffices to show that

a k−1 + b k−1 + c k−1 ≥ a k−1 b k−1 c k−1 + 2.

Due to symmetry, we may assume that a ≥ b ≥ c. In addition, write the inequality as

a k−1 + b k−1 − 2 ≥ (a k−1 b k−1 − 1)c k−1 ,


‹k−1
3 − ab

k−1
a +b k−1
− 2 ≥ (a k−1 k−1
b − 1) .
a+b
Let p p
x= a b, 1 ≤ x ≤ 3.
By the AM-GM inequality, we have

a + b ≥ 2x, a k−1 + b k−1 ≥ 2x k−1 .


104 Vasile Cîrtoaje

Thus, it suffices to show that


k−1
3 − x2

k−1 2k−2
2(x − 1) ≥ (x − 1) .
2x

Since x ≥ 1, this is true if


k−1
3 − x2

2 ≥ (x k−1
+ 1) ,
2x

which can be written as


k−1 k−1
3 − x2 3 − x2
 
2≥ + .
2 2x

Since
3 − x2 3 − x2
1≥ ≥ ,
2 2x
the conclusion follows. Thus, the proof is completed. The equality holds for a = b =
c = 1.

P 1.68. Let a, b, c be nonnegative real numbers such that a + b + c = 2. If 2 ≤ k ≤ 3, than

a k (b + c) + b k (c + a) + c k (a + b) ≤ 2.

Solution. Denote by Ek (a, b, c) the left hand side of the inequality, assume that a ≤ b ≤
c and show that
Ek (a, b, c) ≤ Ek (0, a + b, c) ≤ 2.
The left inequality is equivalent to

a b k−1
(a + b k−1 ) ≤ (a + b)k − a k − b k .
c
Clearly, it suffices to consider c = b, when the inequality becomes

2a k + b k−1 (a + b) ≤ (a + b)k .

Since 2a k ≤ a k−1 (a + b), it remains to show that

a k−1 + b k−1 ≤ (a + b)k−1 ,


Symmetric Nonrational Inequalities 105

which is true since


‹k−1
a k−1 + b k−1  a k−1

b a b
= + ≤ + = 1.
(a + b) k−1 a+b a+b a+b a+b

The right inequality, Ek (0, a + b, c) ≤ 2, is equivalent to

cd(c k−1 + d k−1 ) ≤ 2,

where d = a + b, and hence c + d = 2. By the Power-Mean inequality (or Jensen’s


k−1
inequality applied to the concave function t 2 ), we have
1/(k−1) 1/2
c k−1 + d k−1 c2 + d 2
 
≤ ,
2 2
(k−1)/2
c2 + d 2

k−1
c +d k−1
≤2 .
2
Thus, it suffices to show that
(k−1)/2
c2 + d 2

cd ≤ 1,
2

which is equivalent to
cd(2 − cd)(k−1)/2 ≤ 1.
Since 2 − cd ≥ 1, we have

cd(2 − cd)(k−1)/2 ≤ cd(2 − cd) = 1 − (1 − cd)2 ≤ 1.

The equality holds for a = 0 and b = c = 1 (or any cyclic permutation).

P 1.69. Let a, b, c be nonnegative real numbers, no two of which are zero. If m > n ≥ 0,
than
bm + c m c m + am am + bm
(b + c − 2a) + (c + a − 2b) + (a + b − 2c) ≥ 0.
bn + c n c n + an an + bn
(Vasile Cîrtoaje, 2006)

Solution. Write the inequality as

AX + BY + C Z ≥ 0,
106 Vasile Cîrtoaje

where
bm + c m c m + am am + bm
A= , B = , C = ,
bn + c n c n + an an + bn
X = b + c − 2a, Y = c + a − 2b, Z = a + b − 2c, X + Y + Z = 0.
Without loss of generality, assume that a ≤ b ≤ c, which involves X ≥ Y ≥ Z and X ≥ 0.
Since

2(AX + BY + C Z) = (2A − B − C)X + (B + C)X + 2(BY + C Z)


= (2A − B − C)X − (B + C)(Y + Z) + 2(BY + C Z)
= (2A − B − C)X + (B − C)(Y − Z),

it suffices to show that B ≥ C and 2A − B − C ≥ 0. The inequality B ≥ C can be written


as
b n c n (c m−n − b m−n ) + a n (c m − b m ) − a m (c n − b n ) ≥ 0,
b n c n (c m−n − b m−n ) + a n [c m − b m − a m−n (c n − b n )] ≥ 0.
This is true since c m−n ≥ b m−n and

c m − b m − a m−n (c n − b n ) ≥ c m − b m − b m−n (c n − b n ) = c n (c m−n − b m−n ) ≥ 0.

The inequality 2A − B − C ≥ 0 follows from

2A ≥ b m−n + c m−n , b m−n ≥ C, c m−n ≥ B.

Indeed, we have

(b n − c n )(b m−n − c m−n )


2A − b m−n − c m−n = ≥ 0,
bn + c n
a n (b m−n − a m−n )
b m−n − C = ≥ 0,
an + bn
a n (c m−n − a m−n )
c m−n − B = ≥ 0.
c n + an
The equality holds for a = b = c, and also for a = 0 and b = c (or any cyclic permuta-
tion).

P 1.70. Let a, b, c be positive real numbers such that a bc = 1. Prove that


p p p
a2 − a + 1 + b2 − b + 1 + c 2 − c + 1 ≥ a + b + c.

(Vasile Cîrtoaje, 2012)


Symmetric Nonrational Inequalities 107

First Solution. Among a − 1, b − 1 and c − 1 there are two with the same sign. Let
(b − 1)(c − 1) ≥ 0, that is,
1
t ≤ , t = b + c − 1.
a
By Minkowsky’s inequality, we have
v v
‹2
1 2 3
u u
1 3 t
p p ‹
b − b+1+ c −c+1= + + +
t
2 2 b− c−
2 4 2 4
p
≥ t 2 + 3.
Thus, it suffices to show that
p p
a2 − a + 1 + t 2 + 3 ≥ a + b + c,

which is equivalent to p
a2 − a + 1 + f (t) ≥ a + 1,
where p
f (t) = t 2 + 3 − t.
Clearly, f (t) is decreasing for t ≤ 0. Since
3
f (t) = p ,
t2 + 3 + t
1
 ‹
f (t) is also decreasing for t ≥ 0. Then, f (t) ≥ f , and it suffices to show that
a
1
p  ‹
a2 − a + 1 + f ≥ a + 1,
a
which is equivalent to
v
p t1 1
a2 − a + 1 + + 3 ≥ a + + 1.
a2 a
By squaring, this inequality becomes
v
1 2
t  ‹
2 (a − a + 1) 2 + 3 ≥ 3a + − 1.
2
a a
Indeed, by the Cauchy-Schwarz inequality, we have
v ‹ v
1 1
t  t  ‹
2 (a − a + 1) 2 + 3 = [(2 − a)2 + 3a2 ] 2 + 3
2
a a
2−a 2
≥ + 3a = 3a + − 1.
a a
108 Vasile Cîrtoaje

The equality holds for a = b = c.


Second Solution. If the inequality

1 3
p  ‹
x2 − x +1− x ≥ −1
2 x + x +1
2

holds for all x > 0, then it suffices to prove that

1 1 1
+ + ≥ 1,
a2 + a + 1 b2 + b + 1 c 2 + c + 1
which is just the known inequality in P 1.43. Indeed, the above inequality is equivalent
to
1− x (1 − x)(2 + x)
p ≥ ,
x − x +1+ x
2 2(x 2 + x + 1)
p
(x − 1)[(x + 2) x 2 − x + 1 − x 2 − 2] ≥ 0,
3x 2 (x − 1)2
p ≥ 0.
(x + 2) x 2 − x + 1 + x 2 + 2

P 1.71. Let a, b, c be positive real numbers such that a bc = 1. Prove that


p p p
16a2 + 9 + 16b2 + 9 + 16b2 + 9 ≥ 4(a + b + c) + 3.

(MEMO, 2012)

First Solution (by Vo Quoc Ba Can). Since


p 9
16a2 + 9 − 4a = p ,
16a2 + 9 + 4a

the inequality is equivalent to


X 1 1
p ≥ .
16a2 + 9 + 4a 3

By the AM-GM inequality, we have


p 16a2 + 9
2 16a2 + 9 ≤ + 2a + 3,
2a + 3
p 16a2 + 9 18(2a2 + 2a + 1)
2( 16a2 + 9 + 4a) ≤ + 10a + 3 = .
2a + 3 2a + 3
Symmetric Nonrational Inequalities 109

Thus, it suffices to show that


X 2a + 3
≥ 3.
2a2 + 2a + 1
If the inequality
2a + 3 3
≥ 8/5
2a2 + 2a + 1 a + a4/5 + 1
holds for all a > 0, then it suffices to show that
X 1
≥ 1,
a + a4/5 + 1
8/5

which follows immediately from the inequality in P 1.43. Therefore, using the substitu-
tion x = a1/5 , x > 0, we need to show that

2x 5 + 3 3
≥ 8 ,
2x + 2x + 1
10 5 x + x4 + 1
which is equivalent to

2x 4 (x 5 − 3x 2 + x + 1) + x 4 − 4x + 3 ≥ 0.

This is true since, by the AM-GM inequality, we have


3
p
x 5 + x + 1 ≥ 3 x 5 · x · 1 = 3x 2

and
4
p
x4 + 3 = x4 + 1 + 1 + 1 ≥ 4 x 4 · 1 · 1 · 1 = 4x.
The equality holds for a = b = c = 1.
Second Solution. Making the substitution
p p p
x = 16a2 + 9 − 4a, y = 16b2 + 9 − 4b, z = 16c 2 + 9 − 4c, x, y, z > 0,

which involves
9 − x2 9 − y2 9 − z2
a= , b= , c= ,
8x 8y 8z
we need to show that

(9 − x 2 )(9 − y 2 )(9 − z 2 ) = 512x yz

yields
x + y + z ≥ 3.
Use the contradiction method. Assume that x + y + z < 3 and show that

(9 − x 2 )(9 − y 2 )(9 − z 2 ) > 512x yz.


110 Vasile Cîrtoaje

According to the AM-GM inequality, we get


p p p
3 + x = 1 + 1 + 1 + x ≥ 4 x, 3 + y ≥ 4 4 y, 3 + z ≥ 4 z,
4 4

hence
p
(3 + x)(3 + y)(3 + z) ≥ 64 4 x yz.
Therefore, it suffices to prove that

(3 − x)(3 − y)(3 − z) > 8(x yz)3/4 .

By the AM-GM inequality,


 x + y + z 3
1> ≥ x yz,
3
and hence

(3 − x)(3 − y)(3 − z) = 9(3 − x − y − z) + 3(x y + yz + z x) − x yz


> 3(x y + yz + z x) − x yz ≥ 9(x yz)2/3 − x yz
> 8(x yz)2/3 > 8(x yz)3/4 .

P 1.72. Let a, b, c be positive real numbers such that a bc = 1. Prove that


p p p
25a2 + 144 + 25b2 + 144 + 25c 2 + 144 ≤ 5(a + b + c) + 24.

(Vasile Cîrtoaje, 2012)

First Solution. Since


p 144
25a2 + 144 − 5a = p ,
25a2 + 144 + 5a
the inequality is equivalent to
X 1 1
p ≤ .
25a2 + 144 + 5a 6
If the inequality
1 1
p ≤ p
25a2 + 144 + 5a 6 5a18/13 + 4
holds for all a > 0, then it suffices to show that
X 1
p ≤ 1,
5a18/13 + 4
Symmetric Nonrational Inequalities 111

which follows immediately from P 1.31. Therefore, using the substitution x = a1/13 ,
x > 0, we only need to show that
p p
25x 26 + 144 + 5x 13 ≥ 6 5x 18 + 4.

By squaring, the inequality becomes


p
10x 13 ( 25x 26 + 144 + 5x 13 − 18x 5 ) ≥ 0.

This is true if
25x 26 + 144 ≥ (18x 5 − 5x 13 )2 ,
which is equivalent to
5x 18 + 4 ≥ 9x 10 .
By the AM-GM inequality, we have

5x 18 + 4 = x 18 + x 18 + x 18 + x 18 + x 18 + 1 + 1 + 1 + 1
9
p
≥ 9 x 18 · x 18 · x 18 · x 18 · x 18 · 1 · 1 · 1 · 1 = 9x 10 .

The equality holds for a = b = c = 1.


Second Solution. Making the substitution
p p p
8x = 25a2 + 144 − 5a, 8 y = 25b2 + 144 − 5b, 8z = 25c 2 + 144 − 5c,

which involves
9 − 4x 2 9 − 4 y2 9 − 4z 2 3
 ‹
a= , b= , c= , x, y, z ∈ 0, ,
5x 5y 5z 2
we need to show that

(9 − 4x 2 )(9 − 4 y 2 )(9 − 4z 2 ) = 125x yz

involves
x + y + z ≤ 3.
Use the contradiction method. Assume that x + y + z > 3 and show that

(9 − 4x 2 )(9 − 4 y 2 )(9 − 4z 2 ) < 125x yz.

Since
12x 2 3( y + z − x)( y + z + 3x)
9 − 4x 2 < 3(x + y + z) − = ,
x + y +z x + y +z
it suffices to show that
27AB ≤ 125x yz(x + y + z)3 ,
112 Vasile Cîrtoaje

where

A = ( y + z − x)(z + x − y)(x + y − z), B = ( y + z + 3x)(z + x + 3 y)(x + y + 3z).

Consider the nontrivial case A ≥ 0. By the AM-GM inequality, we have

125
B≤ (x + y + z)3 .
27
Therefore, it suffices to show that
A ≤ x yz,
which is a well known inequality (equivalent to Schur’s inequality of degree three).

P 1.73. If a, b are positive real numbers such that a b + bc + ca = 3, then


p p p
(a) a2 + 3 + b2 + 3 + b2 + 3 ≥ a + b + c + 3;
p p p p
(b) a+b+ b+c+ c+a≥ 4(a + b + c) + 6.
(Lee Sang Hoon, 2007)

Solution. (a) First Solution (by Pham Thanh Hung). By squaring, the inequality be-
comes XÆ
(b2 + 3)(c 2 + 3) ≥ 3(1 + a + b + c).
Since
1
(b2 + 3)(c 2 + 3) = (b + c)(b + a)(c + a)(c + b) = (b + c)2 (a2 + 3) ≥ (b + c)2 (a + 3)2 ,
4
we have
XÆ 1X 1€ X X Š
(b2 + 3)(c 2 + 3) ≥ (b + c)(a + 3) = 2 bc + 6 a = 3(1 + a + b + c).
2 2
The equality holds for a = b = c = 1.
Second Solution. Write the inequality as follows:
Æ Æ Æ
(a + b)(a + c) + (b + c)(b + a) + (c + a)(c + b) ≥ a + b + c + 3,
” Æ — X €p p Š2
2 a + b + c − 3(a b + bc + ca) ≥ a+b− a+c ,

1 X X (b − c)2
p (b − c)2 ≥ p p 2 ,
a+b+c+ 3(a b + bc + ca) a+b+ a+c
Symmetric Nonrational Inequalities 113

X Sa (b − c)2
p p 2 ≥ 0,
a+b+ a+c
where €p p Š2 Æ
Sa = a+b+ a+c −a−b−c− 3(a b + bc + ca).
The last inequality is true since
Æ Æ
Sa = 3(a + b + c) + 2 (a + b)(a + c) − 3(a b + bc + ca)
Æ Æ
>2 a2 + (a b + bc + ca) − 3(a b + bc + ca) > 0.
Third Solution. Use the substitution
p p p
x = a2 + 3 − a, y = b2 + 3 − b, z = c 2 + 3 − c, x, y, z > 0.

We need to show that


x + y + z ≥ 3.
We have X X Æ Æ
yz =
[ (b + a)(b + c) − b][ (c + a)(c + b) − c]
X Æ X Æ X Æ X
= (b + c) (a + b)(a + c) − b (c + a)(c + b) − c (b + a)(b + c) + bc
X
= bc = 3.
Thus, we get Æ
x + y +z ≥ 3(x y + yz + z x) = 3.
(b) By squaring, we get the inequality in (a). Otherwise, using the substitution
p p p
x= b + c, y= c + a, z = a + b,

the inequality becomes


Ç Æ
x + y +z ≥ 2(x 2 + y 2 + z 2 ) + 3(2x 2 y 2 + 2 y 2 z 2 + 2z 2 x 2 − x 4 − y 4 − z 4 ).

By squaring two times, we get


Æ
2(x y + yz + z x) − x 2 − y 2 − z 2 ≥ 3(2x 2 y 2 + 2 y 2 z 2 + 2z 2 x 2 − x 4 − y 4 − z 4 ),
X
(x − y)2 (x + y − z)2 ≥ 0.
114 Vasile Cîrtoaje

P 1.74. If a, b, c are nonnegative real numbers such that a + b + c = 3, then


Æ Æ Æ
(5a2 + 3)(5b2 + 3) + (5b2 + 3)(5c 2 + 3) + (5c 2 + 3)(5a2 + 3) ≥ 24.

(Nguyen Van Quy, 2012)

Solution. Assume that a ≥ b ≥ c, which involves 1 ≤ a ≤ 3 and b + c ≤ 2. Denote

A = 5a2 + 3, B = 5b2 + 3, C = 5c 2 + 3,

and write the inequality as follows:


p p p p
A ( B + C) + BC ≥ 24,
p q p p
A · A(B + C + 2 BC ) ≥ 24 − BC.
This is true if p p
A(B + C + 2 BC) ≥ (24 − BC)2 ,
which is equivalent to
p
A(A + B + C + 48) ≥ (A + 24 − BC)2 .

Applying Lemma below for k = 5/3 and m = 4/15 yields


p
5 BC ≥ 4(b − c)2 + 25bc + 15.

Therefore, it suffices to show that

25A(A + B + C + 48) ≥ [5A + 120 − 4(b − c)2 − 25bc − 15]2 ,

which is equivalent to

25(5a2 + 3)[5(a2 + b2 + c 2 ) + 57] ≥ [25a2 + 120 − 4(b − c)2 − 25bc]2 .

Since

5(a2 + b2 + c 2 ) + 57 = 5a2 + 5(b + c)2 − 10bc + 57 = 2(5a2 − 15a + 51 − 5bc)

and

25a2 + 120 − 4(b − c)2 − 25bc = 25a2 + 120 − 4(b + c)2 − 9bc = 3(7a2 + 8a + 28 − 3bc),

we need to show that

50(5a2 + 3)(5a2 − 15a + 51 − 5bc) ≥ 9(7a2 + 8a + 28 − 3bc)2 .


Symmetric Nonrational Inequalities 115

From bc ≤ (b + c)2 /4 and (a − b)(a − c) ≥ 0, we get

bc ≤ (3 − a)2 /4, bc ≥ a(b + c) − a2 = 3a − 2a2 .

Consider a fixed, a ≥ 1, and denote x = bc. So, we only need to prove that f (x) ≥ 0
for
a2 − 6a + 9
3a − 2a2 ≤ x ≤ ,
4
where

f (x) = 50(5a2 + 3)(5a2 − 15a + 51 − 5x) − 9(7a2 + 8a + 28 − 3x)2 .

a − 6a + 9
 2 
Since f is concave, it suffices to show that f (3a − 2a ) ≥ 0 and f
2
≥ 0.
4
Indeed,

f (3a − 2a2 ) = 3(743a4 − 2422a3 + 2813a2 − 1332a + 198)


= 3(a − 1)2 [(a − 1)(743a − 193) + 5] ≥ 0,

a2 − 6a + 9
 
375
f = (25a4 − 140a3 + 286a2 − 252a + 81)
4 16
375
= (a − 1)2 (5a − 9)2 ≥ 0.
16
Thus, the proof is completed. The equality holds for a = b = c = 1, and also for a = 9/5
and b = c = 3/5 (or any cyclic permutation).
k
Lemma. Let b, c ≥ 0 such that b + c ≤ 2. If k > 0 and 0 ≤ m ≤ , then
2k + 2
Æ
(k b2 + 1)(kc 2 + 1) ≥ m(b − c)2 + k bc + 1.

Proof. By squaring, the inequality becomes

(b − c)2 [k − 2m − 2kmbc − m2 (b − c)2 ] ≥ 0.

This is true since

k − 2m − 2kmbc − m2 (b − c)2 = k − 2m − 2m(k − 2m)bc − m2 (b + c)2


m(k − 2m)
≥ k − 2m − (b + c)2 − m2 (b + c)2
2
km
= k − 2m − (b + c)2 ≥ k − 2m − 2km ≥ 0.
2
116 Vasile Cîrtoaje

P 1.75. If a, b, c are nonnegative real numbers such that a + b + c = 3, then


v
p p p t 4(a2 + b2 + c 2 ) + 42
a2 + 1 + b2 + 1 + c2 + 1 ≥ .
3

(Vasile Cîrtoaje, 2014)

Solution. Assume that a ≥ b ≥ c, which involves a ≥ 1 and b + c ≤ 2. By squaring, the


inequality becomes

p p p p a2 + b2 + c 2 + 33
A ( B + C) + BC ≥ ,
6

p p a2 + b2 + c 2 + 33
q
A(B + C + 2 BC ) + BC ≥ ,
6
where
A = a2 + 1, B = b2 + 1, C = c 2 + 1.

Applying Lemma from the preceding problem P 1.74 for k = 1 and m = 1/4 gives
p 1
BC ≥ (b − c)2 + bc + 1.
4

Therefore, it suffices to show that


v
t 1 1 a2 + b2 + c 2 + 33
A[B + C + (b − c)2 + 2bc + 2] + (b − c)2 + bc + 1 ≥ ,
2 4 6

which is equivalent to
Æ
6 2(a2 + 1)[3(b + c)2 + 8 − 4bc] ≥ 2a2 − (b + c)2 + 54 − 4bc,
Æ
6 2(a2 + 1)(3a2 − 18a + 35 − 4bc) ≥ a2 + 6a + 45 − 4bc.

From bc ≤ (b + c)2 /4 and (a − b)(a − c) ≥ 0, we get

bc ≤ (3 − a)2 /4, bc ≥ a(b + c) − a2 = 3a − 2a2 .

Consider a fixed, a ≥ 1, and denote x = bc. So, we only need to prove that f (x) ≥ 0
for
a2 − 6a + 9
3a − 2a2 ≤ x ≤ ,
4
where
f (x) = 72(a2 + 1)(3a2 − 18a + 35 − 4x) − (a2 + 6a + 45 − 4x)2 .
Symmetric Nonrational Inequalities 117

a2 − 6a + 9
 
Since f is concave, it suffices to show that f (3a − 2a ) ≥ 0 and f
2
≥ 0.
4
Indeed,

f (3a − 2a2 ) = 9(79a4 − 228a3 + 274a2 − 180a + 55)


= 9(a − 1)2 (79a2 − 70a + 55 ≥ 0,

a2 − 6a + 9
 
f = 144(a4 − 6a3 + 13a2 − 12a + 4)
4
= 144(a − 1)2 (a − 2)2 ≥ 0.

The equality holds for a = b = c = 1, and also for a = 2 and b = c = 1/2 (or any cyclic
permutation).

P 1.76. If a, b, c are nonnegative real numbers such that a + b + c = 3, then


p p p p
(a) a2 + 3 + b2 + 3 + c 2 + 3 ≥ 2(a2 + b2 + c 2 ) + 30;
p p p p
(b) 3a2 + 1 + 3b2 + 1 + 3c 2 + 1 ≥ 2(a2 + b2 + c 2 ) + 30.
(Vasile Cîrtoaje, 2014)

Solution. Assume that a ≥ b ≥ c, which involves a ≥ 1 and b + c ≤ 2.


(a) By squaring, the inequality becomes

p p p p a2 + b2 + c 2 + 21
A ( B + C) + BC ≥ ,
2
p p a2 + b2 + c 2 + 21
q
A(B + C + 2 BC ) + BC ≥ ,
2
where
A = a2 + 3, B = b2 + 3, C = c 2 + 3.
Applying Lemma from problem P 1.74 for k = 1/3 and m = 1/9 gives
p 1
BC ≥ (b − c)2 + bc + 3.
3
Therefore, it suffices to show that
v
t 2 1 a2 + b2 + c 2 + 21
A[B + C + (b − c)2 + 2bc + 6] + (b − c)2 + bc + 3 ≥ ,
3 3 2
118 Vasile Cîrtoaje

which is equivalent to
Æ
2 3(a2 + 3)[5(b + c)2 + 36 − 8bc] ≥ 3a2 + (b + c)2 + 45 − 4bc,
Æ
3(a2 + 3)(5a2 − 30a + 81 − 8bc) ≥ 2a2 − 3a + 27 − 2bc.

From bc ≤ (b + c)2 /4 and (a − b)(a − c) ≥ 0, we get

bc ≤ (3 − a)2 /4, bc ≥ a(b + c) − a2 = 3a − 2a2 .

Consider a fixed, a ≥ 1, and denote x = bc. So, we only need to prove that f (x) ≥ 0
for
a2 − 6a + 9
3a − 2a2 ≤ x ≤ ,
4
where
f (x) = 3(a2 + 3)(5a2 − 30a + 81 − 8x) − (2a2 − 3a + 27 − 2x)2 .
a − 6a + 9
 2 
Since f is concave, it suffices to show that f (3a − 2a ) ≥ 0 and f
2
≥ 0.
4
Indeed,
f (3a − 2a2 ) = 27a2 (a − 1)2 ≥ 0,

a2 − 6a + 9
 
27 4
f = (a − 8a3 + 22a2 − 24a + 9)
4 4
27
= (a − 1)2 (a − 3)2 ≥ 0.
4

The equality holds for a = b = c = 1, and also for a = 3 and b = c = 0 (or any cyclic
permutation).
(b) By squaring, the inequality becomes

p p p p 27 − a2 − b2 − c 2
A ( B + C) + BC ≥ ,
2

p p 27 − a2 − b2 − c 2
q
A(B + C + 2 BC ) + BC ≥ ,
2
where
A = 3a2 + 1, B = 3b2 + 1, C = 3c 2 + 1.

Applying Lemma from problem P 1.74 for k = 3 and m = 1/3 gives


p 1
BC ≥ (b − c)2 + 3bc + 1.
3
Symmetric Nonrational Inequalities 119

Therefore, it suffices to show that


v
t 2 1 27 − a2 − b2 − c 2
A[B + C + (b − c)2 + 6bc + 2] + (b − c)2 + 3bc + 1 ≥ ,
3 3 2
which is equivalent to
Æ
2 3(3a2 + 1)[11(b + c)2 + 12 − 8bc] ≥ 75 − 3a2 − 5(b + c)2 − 4bc,
Æ
3(3a2 + 1)(11a2 − 66a + 111 − 8bc) ≥ 15 + 15a − 4a2 − 2bc.
From bc ≤ (b + c)2 /4 and (a − b)(a − c) ≥ 0, we get

bc ≤ (3 − a)2 /4, bc ≥ a(b + c) − a2 = 3a − 2a2 .

Consider a fixed, a ≥ 1, and denote x = bc. So, we only need to prove that f (x) ≥ 0
for
a2 − 6a + 9
3a − 2a2 ≤ x ≤ ,
4
where

f (x) = 3(3a2 + 1)(11a2 − 66a + 111 − 8x) − (15 + 15a − 4a2 − 2x)2 .

a − 6a + 9
 2 
Since f is concave, it suffices to show that f (3a − 2a ) ≥ 0 and f
2
≥ 0.
4
Indeed,
f (3a − 2a2 ) = 27(a − 1)2 (3a − 2)2 ≥ 0,

a2 − 6a + 9
 
27
f = (9a4 − 48a3 + 94a2 − 80a + 25)
4 4
27
= (a − 1)2 (3a − 5)2 ≥ 0.
4
The equality holds for a = b = c = 1, and also for a = 5/3 and b = c = 2/3 (or any
cyclic permutation).
Remark. Similarly, we can prove the following generalization.
• Let a, b, c be nonnegative real numbers such that a + b + c = 3. If k > 0, then
v
p p p t 8k(a2 + b2 + c 2 ) + 3(9k2 + 10k + 9)
ka2 + 1 + k b2 + 1 + kc 2 + 1 ≥ ,
3(k + 1)
3k + 1 3k − 1
with equality for a = b = c = 1, and also for a = and b = c = (or any
2k 4k
cyclic permutation).
120 Vasile Cîrtoaje

P 1.77. If a, b, c are nonnegative real numbers such that a + b + c = 3, then


Æ Æ Æ
(32a2 + 3)(32b2 + 3) + (32b2 + 3)(32c 2 + 3) + (32c 2 + 3)(32a2 + 3) ≤ 105.

(Vasile Cîrtoaje, 2014)

Solution. Assume that a ≤ b ≤ c, which involves a ≤ 1 and b + c ≥ 2. Denote

A = 32a2 + 3, B = 32b2 + 3, C = 32c 2 + 3,

and write the inequality as follows:


p p p p
A ( B + C) + BC ≤ 105,
p Æ p p
A · B + C + 2 BC ≤ 105 − BC.
By Lemma below, we have
p
BC ≤ 5(b + c)2 + 12bc + 3 ≤ 8(b + c)2 + 3 ≤ 8(a + b + c)2 + 3 = 75 < 105.

Therefore, we can write the desired inequality as


p p
A(B + C + 2 BC) ≤ (105 − BC)2 ,

which is equivalent to
p
A(A + B + C + 210) ≤ (A + 105 − BC)2 .

According to Lemma below, it suffices to show that

A(A + B + C + 210) ≤ [A + 105 − 5(b2 + c 2 ) − 22bc − 3]2 ,

which is equivalent to

[32a2 + 105 − 5(b2 + c 2 ) − 22bc]2 ≥ (32a2 + 3)[32(a2 + b2 + c 2 ) + 219].

Since

32(a2 + b2 + c 2 ) + 219 = 32a2 + 32(b + c)2 − 64bc + 219 = 64a2 − 192a + 507 − 64bc

and

32a2 +105−5(b2 +c 2 )−22bc = 32a2 +105−5(b+c)2 −12bc = 3(9a2 +10a+20−4bc),

we need to show that

9(9a2 + 10a + 20 − 4bc)2 ≥ (32a2 + 3)(64a2 − 192a + 507 − 64bc).


Symmetric Nonrational Inequalities 121

From bc ≤ (b + c)2 /4, we get


bc ≤ (3 − a)2 /4.
Consider a fixed, 0 ≤ a ≤ 1, and denote x = bc. So, we only need to prove that f (x) ≥ 0
for
a2 − 6a + 9
x≤ ,
4
where

f (x) = 9(9a2 + 10a + 20 − 4x)2 − (32a2 + 3)(64a2 − 192a + 507 − 64x).

Since

f 0 (x) = 72(4x − 9a2 − 10a − 20) + 64(32a2 + 3)


≤ 72[(a2 − 6a + 9) − 9a2 − 10a − 20) + 64(32a2 + 3)
= 8[184a(a − 1) + (44a − 75)] < 0,

a − 6a + 9
 2 
f is decreasing, hence f (x) ≥ f . Therefore, it suffices to show that
4
a − 6a + 9
 2 
f ≥ 0. We have
4

a − 6a + 9
 2 
f =9[9a2 + 10a + 20 − (a2 − 6a + 9)]2
4
− (32a2 + 3)[64a2 − 192a + 507 − 16(a2 − 6a + 9)]
=9(8a2 + 16a + 11)2 − (32a2 + 3)(48a2 − 96a + 363)
=192a(a − 1)2 (18 − 5a) ≥ 0.

Thus, the proof is completed. The equality holds for a = b = c = 1, and also for a = 0
and b = c = 3/2 (or any cyclic permutation).
Lemma. If b, c ≥ 0 such that b + c ≥ 2, then
Æ
(32b2 + 3)(32c 2 + 3) ≤ 5(b2 + c 2 ) + 22bc + 3.

Proof. By squaring, the inequality becomes

(5b2 + 5c 2 + 22bc)2 − 322 b2 c 2 ≥ 96(b2 + c 2 ) − 6(5b2 + 5c 2 + 22bc),

5(b − c)2 (5b2 + 5c 2 + 54bc) ≥ 66(b − c)2 .


It suffices to show that
5(5b2 + 5c 2 + 10bc) ≥ 100,
which is equivalent to the obvious inequality (b + c)2 ≥ 4.
122 Vasile Cîrtoaje

P 1.78. If a, b, c are positive real numbers, then



b+c

c + a
+

a b
a − 3 + b − 3 + c − 3 ≥ 2.

(Vasile Cîrtoaje, 2012)

Solution. Without loss of generality, assume that a ≥ b ≥ c.


Case 1: a > b + c. We have
b+c a + b c + a b+c b+c


a − 3 + c − 3 + b − 3 ≥ a − 3 = 3 − a > 2.

Case 2: a ≤ b + c. We have
b+c a + b c + a b+c c + a

− 3 + − 3 + − 3 ≥ − 3 + − 3

a c
b a b

b+c c + a 2b b + a (a − b)(2b − a)
 ‹ 
= 3− + 3− ≥6− − =2+ ≥ 2.
a b a b ab
a
Thus, the proof is completed. The equality holds for = b = c (or any cyclic permuta-
2
tion).

P 1.79. If a, b, c are real numbers such that a bc 6= 0, then

b + c c + a a + b

+ +
≥ 2.
a b c

First Solution. Let


|a| = max{|a|, |b|, |c|}.
We have
b + c c + a a + b b + c c + a a + b

+ +
≥ + +

a b c a a a
|(−b − c) + (c + a) + (a + b)|
≥ = 2.
|a|
The equality holds for a = 1, b = −1 and |c| ≤ 1 (or any permutation).
Second Solution. Since the inequality remains unchanged by replacing a, b, c with
−a, −b, −c, it suffices to consider two cases: a, b, c > 0, and a < 0 and b, c > 0.
Symmetric Nonrational Inequalities 123

Case 1: a, b, c > 0. We have



b + c c + a a + b  a b ‹  b c ‹  c a 

+ +
= + + + + + ≥ 6.
a b c b a c b a c

Case 2: a < 0 and b, c > 0. Replacing a by −a, we need to show that

b + c |a − c| |a − b|
+ + ≥2
a b c
for all a, b, c > 0. Without loss of generality, assume that b ≥ c.
For b ≥ c ≥ a, we have
b + c |a − c| |a − b| b+c
+ + ≥ ≥ 2.
a b c a
For b ≥ a ≥ c, we have

b + c |a − c| |a − b| b+c a−c (a − b)2 + c(b − a)


+ + −2≥ + −2= ≥ 0.
a b c a b ab
For a ≥ b ≥ c, we have
b + c |a − c| |a − b| b+c a−c a−b
+ + = + +
a b c a b c
a−b 1 1 (a − b)2 (a − b)(a b − c 2 )
 ‹  ‹
a b
= + −2 + +c − = + ≥ 0.
b a c a b ab a bc

Third Solution. Using the substitution

b+c c+a a+b


x= , y= , z= ,
a b c
we need to show that
x + y + z + 2 = x yz, x, y, z ∈ R
involves
|x| + | y| + |z| ≥ 2.
If x yz ≤ 0, then
−x − y − z = 2 − x yz ≥ 2,
hence
|x| + | y| + |z| ≥ |x + y + z| = | − x − y − z| ≥ −x − y − z ≥ 2.
If x yz > 0, then either x, y, z > 0 or only one of x, y, z is positive (for instance, x > 0
and y, z < 0).
124 Vasile Cîrtoaje

Case 1: x, y, z > 0. We need to show that x + y + z ≥ 2. We have

x yz = x + y + z + 2 > 2

and, by the AM-GM inequality, we get


p p
3
x + y + z ≥ 3 3 x yz > 3 2 > 2,

Case 2: x > 0 and y, z < 0. Replacing y, z by − y, −z, we need to prove that

x − y − z + 2 = x yz

involves
x + y +z ≥2

for all x, y, z > 0. Since

x + y + z − 2 = x + y + z − (x yz − x + y + z) = x(2 − yz),

we need to show that yz ≤ 2. Indeed, we have


p
x + 2 = y + z + x yz ≥ 2 yz + x yz,

p
x(1 − yz) + 2(1 − yz ) ≥ 0,
p p
(1 − yz )[x(1 + yz ) + 2] ≥ 0,

hence
yz ≤ 1 < 2.

P 1.80. Let a, b, c be nonnegative real numbers, no two of which are zero, and let

2a 2b 2c
x= , y= , z= .
b+c c+a a+b

Prove that
p p p
x + y +z+ xy+ yz + z x ≥ 6.
Symmetric Nonrational Inequalities 125

First Solution. Since


p p p
p 2 bc(a + b)(c + a) 2 bc (a + bc)
yz = ≥
(a + b)(c + a) (a + b)(c + a)
p
2a(b + c) bc + 2bc(b + c) 4a bc + 2bc(b + c)
= ≥ ,
(a + b)(b + c)(c + a) (a + b)(b + c)(c + a)

we have

12a bc + 2 bc(b + c)
P
Xp
yz ≥
(a + b)(b + c)(c + a)
8a bc
= + 2.
(a + b)(b + c)(c + a)

Therefore, it suffices to show that

a b c 4a bc
+ + + ≥ 2,
b + c c + a a + b (a + b)(b + c)(c + a)

which is equivalent to Schur’s inequality of degree three


X
a3 + b3 + c 3 + 3a bc ≥ a b(a + b).

The equality holds for a = b = c, and also for a = 0 and b = c (or any cyclic permuta-
tion).
Second Solution. Write the inequality as
X Xp p
4 (x − 1) ≥ ( y − z)2 .

Since
X X (a − b) + (a − c) X a − b X b − a X (a − b)2
(x − 1) = = + =
b+c b+c c+a (b + c)(c + a)

X (b − c)2
= ,
(a + b)(a + c)
and
p p 2(b − c)2 (a + b + c)2
( y − z)2 = p p 2 ,
(a + b)(a + c) b2 + a b + c 2 + ac
we can write the inequality as
X
(b − c)2 Ea ≥ 0,
126 Vasile Cîrtoaje

where  
(a + b + c) 2
Ea = (b + c) 2 − p p 2  .
b + a b + c + ac
2 2

By Minkowski’s inequality, we have


€p p Š2 p p
b2 + a b + c 2 + ac ≥ (b+c)2 +a( b+ c)2 ≥ (b+c)2 +a(b+c) = (b+c)(a+b+c).

Therefore,
a+b+c
 ‹
Ea ≥ (b + c) 2 − = b + c − a,
b+c
and hence X X
(b − c)2 Ea ≥ (b − c)2 (b + c − a) ≥ 0.
The right inequality is just Schur’s inequality of third degree.
Third Solution. Using the Cauchy-Schwarz inequality, we have

a b c (a + b + c)2 (a + b + c)2
+ + ≥ = .
b+c c+a a+b a(b + c) + b(c + a) + c(a + b) 2(a b + bc + ca)
Using Hölder’s inequality, we have
‚s v Œ2
(a + b + c)3
s
a t b c
+ + ≥ 2 .
b+c c+a a+b a (b + c) + b2 (c + a) + c 2 (a + b)

Thus, it suffices to prove that

(a + b + c)2 2(a + b + c)3


+ 2 ≥ 12.
a b + bc + ca a (b + c) + b2 (c + a) + c 2 (a + b)
Due to homogeneity, we may assume that a + b + c = 1. Substituting q = a b + bc + ca,
3q ≤ 1, the inequality becomes
1 2
+ ≥ 12.
q q − 3a bc
From the fourth degree Schur’s inequality

6a bc p ≥ (p2 − q)(4q − p2 ), p = a + b + c,

we obtain
6a bc ≥ (4q − 1)(1 − q).
Therefore,
1 2 1 2
+ − 12 ≥ + (4q−1)(1−q)
− 12
q q − 3a bc q q−
2
Symmetric Nonrational Inequalities 127

1 4 (1 − 3q)(1 − 4q)2
= + 2 − 12 = ≥ 0.
q 4q − 3q + 1 q(4q2 − 3q + 1)

P 1.81. Let a, b, c be nonnegative real numbers, no two of which are zero, and let

2a 2b 2c
x= , y= , z= .
b+c c+a a+b
Prove that p p p
1 + 24x + 1 + 24 y + 1 + 24z ≥ 15.
(Vasile Cîrtoaje, 2005)

Solution (by Vo Quoc Ba Can). Assume that c = min{a, b, c}, hence z = min{x, y, z},
z ≤ 1. By Hölder’s inequality
‚s v Œ2
a t b  2
a (b + c) + b2 (c + a) ≥ (a + b)3 ,

+
b+c c+a

we get

p p 2 2(a + b)3 2(a + b)3


x+ y ≥ ≥
c(a + b)2 + a b(a + b − 2c) c(a + b)2 + 14 (a + b)2 (a + b − 2c)

8(a + b) 8
= = .
a + b + 2c 1+z
Using this result and Minkowski’s inequality, we have
v
p p q p p t 48
1 + 24x + 1 + 24 y ≥ (1 + 1)2 + 24( x + y)2 ≥ 2 1 + .
1+z

Therefore, it suffices to show that


v
t 48 p
2 1+ + 1 + 24z ≥ 15.
1+z

Using the substitution


p 1
1 + 24z = 5t, ≤ t ≤ 1,
5
the inequality turns into v
t t 2 + 47
2 ≥ 3 − t.
25t 2 + 23
128 Vasile Cîrtoaje

By squaring, this inequality becomes

25t 4 − 150t 3 + 244t 2 − 138t + 19 ≤ 0,

which is equivalent to the obvious inequality

(t − 1)2 (5t − 1)(5t − 19) ≤ 0.

The equality holds for a = b = c, and also for a = 0 and b = c (or any cyclic permuta-
tion).

P 1.82. If a, b, c are positive real numbers, then


v v v
t 7a t 7b t 7c
+ + ≤ 3.
a + 3b + 3c b + 3c + 3a c + 3a + 3b

(Vasile Cîrtoaje, 2005)

First Solution. Making the substitution


v v v
t 7a t 7b t 7c
x= , y= , z= ,
a + 3b + 3c b + 3c + 3a c + 3a + 3b

we have 

 (x 2 − 7)a + 3x 2 b + 3x 2 c = 0


3 y 2 a + ( y 2 − 7)b + 3 y 2 c = 0 ,


 3z a + 3z b + (z − 7)c = 0

 2 2 2

which involves
x2 − 7 3x 2 3x 2

3 y2 y − 7 3 y2
2 =0 ;

3z 2 3z 2 z2 − 7

that is,
F (x, y, z) = 0,
where X X
F (x, y, z) = 4x 2 y 2 z 2 + 8 x2 y2 + 7 x 2 − 49.
We need to show that F (x, y, z) = 0 involves x + y + z ≤ 3, where x, y, z > 0. To
do this, we use the contradiction method. Assume that x + y + z > 3 and show that
F (x, y, z) > 0. Since F (x, y, z) is strictly increasing in each of its arguments, it is enough
Symmetric Nonrational Inequalities 129

to prove that x + y + z = 3 involves F (x, y, z) ≥ 0. Assume that x = max{x, y, z} and


denote
y +z
t= , 0 < t ≤ 1 ≤ x.
2
We will show that
F (x, y, z) ≥ F (x, t, t) ≥ 0.
We have

F (x, y, z) − F (x, t, t) = (8x 2 + 7)( y 2 + z 2 − 2t 2 ) − 4(x 2 + 2)(t 4 − y 2 z 2 )


1
= (8x 2 + 7)( y − z)2 − (x 2 + 2)(t 2 + yz)( y − z)2
2
1
≥ (8x 2 + 7)( y − z)2 − 2(x 2 + 2)( y − z)2
2
1
= (4x 2 − 1)( y − z)2 ≥ 0
2
and
3− x 3− x 1
 ‹
F (x, t, t) = F x, , = (x − 1)2 (x − 2)2 (x 2 − 6x + 23) ≥ 0.
2 2 4
a
= b = c (or any cyclic permutation).
The equality holds for a = b = c, and also for
8
Second Solution. Due to homogeneity, we may assume that a + b + c = 3, when the
inequality becomes v
X t 7a
≤ 3.
9 − 2a
Using the substitution
v v v
t 7a t 7b t 7c
x= , y= , z= ,
9 − 2a 9 − 2b 9 − 2c

we need to show that if x, y, z are positive real number such that


X x2 1
= ,
2x + 7 3
2

then
x + y + z ≤ 3.
For the sake of contradiction, assume that x + y + z > 3 and show that F (x, y, z) > 0,
where
X x2 1
F (x, y, z) = − .
2x 2 + 7 3
130 Vasile Cîrtoaje

Since F (x, y, z) is strictly increasing in each of its arguments, it is enough to prove that
x + y + z = 3 involves F (x, y, z) ≥ 0. This is just the inequality in P 1.29. We give here
another proof. By the Cauchy-Schwarz inequality, we have

( x 3/2 )2 ( x 3/2 )2
P P
X x2
≥P = P .
2x 2 + 7 x(2x 2 + 7) 2 x 3 + 21
Therefore, it suffices to show that
X X
3( x 3/2 )2 ≥ 2 x 3 + 21,

which is equivalent to the homogeneous inequality


X X 7
x3 + 6 (x y)3/2 ≥ (x + y + z)3 .
9
Replacing x, y, z by x 2 , y 2 , z 2 , we need to prove that G(x, y, z) ≥ 0, where
X X 7
G(x, y, z) = x6 + 6 x 3 y 3 − (x 2 + y 2 + z 2 )3 .
9
Assume that x = max{x, y, z} and denote
v
t y 2 + z2
t= , 0 < t ≤ x.
2
We will show that
G(x, y, z) ≥ G(x, t, t) ≥ 0.
We have

G(x, y, z) − G(x, t, t) = y 6 + z 6 + 6 y 3 z 3 − 8t 4 + 6x 3 ( y 3 + z 3 − 2t 3 ).

Since y 3 + z 3 − 2t 3 ≥ 0, we get

6x 3 ( y 3 + z 3 − 2t 3 ) ≥ 4x 3 ( y 3 + z 3 − 2t 3 ) ≥ ( y 3 + z 3 + 2t 3 )( y 3 + z 3 − 2t 3 ).

Thus,
G(x, y, z) − G(x, t, t) ≥ y 6 + z 6 + 6 y 3 z 3 − 8t 4 + [( y 3 + z 3 )2 − 4t 6 ]
= 2( y 6 + z 6 + 4 y 3 z 3 − 6t 6 ) = 2[( y 2 + z 2 )3 − 3 y 2 z 2 ( y 2 + z 2 ) + 4 y 3 z 3 − 6t 6 ]
= 4(t 6 − 3t 2 y 2 z 2 + 2 y 3 z 3 ) = 4(t 2 − yz)2 (t 2 + 2 yz) ≥ 0.
Also,
7
G(x, t, t) = x 6 + 2t 6 + 6(t 6 + 2x 3 t 3 ) − (x 2 + 2t 2 )3
9
2
= (x − t)2 (x − 2t)2 (x 2 + 6x t + 2t 2 ) ≥ 0.
9
Symmetric Nonrational Inequalities 131

P 1.83. If a, b, c are positive real numbers such that a + b + c = 3, then


Æ
3
Æ
3
Æ
3
p
3
a2 (b2 + c 2 ) + b2 (c 2 + a2 ) + c 2 (a2 + b2 ) ≤ 3 2.

(Michael Rozenberg, 2013)

Solution. By Hölder’s inequality, we have

”X Æ
3
—3 ”X —2 X b 2 + c 2
a2 (b2 + c 2 ) ≤ a(b + c) · .
(b + c)2

Therefore, it suffices to show that


X b2 + c 2 27
≤ ,
(b + c)2 2(a b + bc + ca)2

which is equivalent to the homogeneous inequalities

X  b2 + c 2 p4

− 1 ≤ − 3,
(b + c)2 6q2

X 2bc p4
+ ≥ 3,
(b + c)2 6q2
where
p = a + b + c, q = a b + bc + ca.

According to P 1.60, the following inequality holds


X 2bc p2 9
+ ≥ .
(b + c)2 q 2

Thus, it is enough to show that

9 p2 p4
− + 2 ≥ 3,
2 q 6q

which is equivalent to
2
p2

−3 ≥ 0.
q
The equality holds for a = b = c = 1.
132 Vasile Cîrtoaje

P 1.84. If a, b, c are nonnegative real numbers, no two of which are zero, then

1 1 1 1 2
+ + ≥ +p .
a+b b+c c+a a+b+c a b + bc + ca
(Vasile Cîrtoaje, 2005)

Solution. Using the notation

p = a + b + c, q = a b + bc + ca, r = a bc,

we can write the inequality as

p2 + q 1 2
≥ +p .
pq − r p q

According to P 2.57-(a) in Volume 1, for fixed p and q, the product r = a bc is minimal


when two of a, b, c are equal or one of a, b, c is zero. Therefore, it suffices to prove the
inequality for b = c = 1 and for a = 0. For a = 0, the inequality reduces to

1 1 2
+ ≥p ,
b c bc
which is obvious. For b = c = 1, the inequality becomes as follows:

1 2 1 2
+ ≥ +p ,
2 a+1 a+2 2a + 1

1 1 2 2
− ≥p − ,
2 a+2 2a + 1 a + 1
p
a 2(a + 1 − 2a + 1)
≥ p ,
2(a + 2) (a + 1) 2a + 1
a 2a2
≥ p p .
2(a + 2) (a + 1) 2a + 1(a + 1 + 2a + 1)
So, we need to show that

1 2a
≥ p p .
2(a + 2) (a + 1) 2a + 1(a + 1 + 2a + 1)

Consider two cases: 0 ≤ a ≤ 1 and a > 1.


Case 1: 0 ≤ a ≤ 1. Since
p p p p p
2a + 1(a + 1 + 2a + 1) ≥ 2a + 1( 2a + 1 + 2a + 1) = 2(2a + 1),
Symmetric Nonrational Inequalities 133

it suffices to prove that


1 a
≥ ,
2(a + 2) (a + 1)(2a + 1)
which is equivalent to 1 − a ≥ 0.
Case 2: a > 1. We will show first that
p
(a + 1) 2a + 1 > 3a.

Indeed, by squaring, we get the obvious inequality

a3 + a(a − 2)2 + 1 > 0.

Therefore, it suffices to show that

1 2a
≥ ,
2(a + 2) (a + 1)(3a + 2a + 1)

which is equivalent to (a−1)2 ≥ 0. The proof is completed. The equality holds for a = 0
and b = c.

P 1.85. If a, b ≥ 1, then

1 1 1 1
p + ≥p +p .
3a b + 1 2 3a + 1 3b + 1

Solution. Using the substitution

2 2
x=p , y=p , x, y ∈ (0, 1],
3a + 1 3b + 1
the desired inequality can be written as
v
t 3
xy ≥ x + y − 1.
x2 y2 − x2 − y2 + 4

Consider the non-trivial case x + y − 1 ≥ 0, and denote

t = x + y − 1, p = x y, 1 ≥ p ≥ t ≥ 0.

Since
x 2 + y 2 = (x + y)2 − 2x y = (t + 1)2 − 2p,
134 Vasile Cîrtoaje

we need to prove that v


t 3
p ≥ t.
p2 + 2p − t 2 − 2t + 3
By squaring, we get the inequality

(p − t)[(3 − t 2 )p + t(1 − t)(3 + t)] ≥ 0,

which is clearly true. The equality holds for a = b = 1.

P 1.86. Let a, b, c be positive real numbers such that a ≥ 1 ≥ b ≥ c and a bc = 1. Prove


that
1 1 1 3
p +p +p ≥ .
3a + 1 3b + 1 3c + 1 2
(Vasile Cîrtoaje, 2007)

Solution. Let b1 = 1/b, b1 ≥ 1. We claim that

1 1 1
p +p ≥ .
3b + 1 3b1 + 1 2

This inequality is equivalent to


v
1 t b 1
p + ≥ .
3b + 1 b+3 2

Making the substitution


1 1
p = t, ≤ t < 1,
3b + 1 2
the inequality becomes v
t 1 − t2
≥ 1 − t.
1 + 8t 2
By squaring, we get t(1 − t)(1 − 2t)2 ≥ 0, which is clearly true. Similarly, we have

1 1 1
p +p ≥ ,
3c + 1 3c1 + 1 2

where c1 = 1/c, c1 ≥ 1. Using these inequality, it suffices to show that

1 1 1 1
p + ≥p +p ,
3a + 1 2 3b1 + 1 3c1 + 1
Symmetric Nonrational Inequalities 135

which is equivalent to

1 1 1 1
+ ≥p +p .
3b1 c1 + 1 3b1 + 1 3c1 + 1
p
2

From the inequality in the preceding P 1.85, the conclusion follows. The equality holds
for a = b = c = 1.

1
P 1.87. Let a, b, c be positive real numbers such that a + b + c = 3. If k ≥ p , then
2

(a bc)k (a2 + b2 + c 2 ) ≤ 3.

(Vasile Cîrtoaje, 2006)

Solution. Since ‹3
a+b+c

abc ≤ = 1,
3
p
it suffices to prove the desired inequality for k = 1/ 2. Write the inequality in the
homogeneous form
‹3k+2
a+b+c

(a bc)k (a2 + b2 + c 2 ) ≤ 3 .
3

According to P 2.57-(a) in Volume 1, for fixed a+b+c and a b+bc+ca, the product a bc is
maximal when two of a, b, c are equal. Therefore, it suffices to prove the homogeneous
inequality for b = c = 1; that is, f (a) ≥ 0, where

f (a) = (3k + 2) ln(a + 2) − (3k + 1) ln 3 − k ln a − ln(a2 + 2).

From
3k + 2 k 2a
f 0 (a) = − − 2
a+2 a a +2
p p
2(a − 1)(ka2 − 2a + 2k) 2(a − 1)(a − 2)2
= = ,
a(a + 2)(a2 + 2) a(a + 2)(a2 + 2)

it follows that f is decreasing on (0, 1] and increasing on [1, ∞); therefore, f (a) ≥
f (1) = 0. This completes the proof. The equality holds for a = b = c.
136 Vasile Cîrtoaje

P 1.88. Let p and q be nonnegative real numbers such that p2 ≥ 3q, and let
v v
t 2p − 2w t 2p + w
g(p, q) = +2 ,
3 3

2p + 2w
 s s
2p − w

 +2 , p2 ≤ 4q
3 3
h(p, q) = p p ,
 p + p + q, p2 ≥ 4q
p

where w =
p
p2 − 3q. If a, b, c are nonnegative real numbers such that

a + b + c = p, a b + bc + ca = q,

then
p p p
(a) a+b+ b+c+ c + a ≥ g(p, q),

p + 2w p−w
with equality for a = and b = c = (or any cyclic permutation);
3 3
p p p
(b) a + b + b + c + c + a ≤ h(p, q),

p − 2w p+w
with equality for a = and b = c = (or any cyclic permutation) - when
3 3
p ≤ 4q, and for a = 0, b + c = p and bc = q (or any cyclic permutation) - when p2 ≥ 4q.
2

(Vasile Cîrtoaje, 2013)


Solution. Consider the non-trivial case p > 0. Since

b + c = p − a,

(a + b)(a + c) = a2 + q
and
p
p Ç Æ
a+b+ a+c = a+p+2 a2 + q,
we get p p p
a+b+ b+c+ c + a = f (a),
where Ç Æ
f (a) = p−a+ a+p+2 a2 + q.
p

From
a2 + q + 2a
p
−1
f 0 (a) = p + p ,
2 p − a 2 a 2 + q · a + p + 2p a 2 + q
q
Symmetric Nonrational Inequalities 137

it follows that f 0 (a) has the same sign as F (a), where

( a2 + q + 2a)2 −2(3a2 − 2pa + q)(a + a2 + q)


p p
−1
F (a) = + = .
p − a (a2 + q)(a + p + 2 a2 + q) (p − a)(a2 + q)(a + p + 2 a2 + q)
p p

Therefore, f (a) is increasing on [a1 , a2 ] and decreasing on [0, a1 ] and [a2 , p], where

p + p2 − 3q
p p
p − p2 − 3q
a1 = , a2 = .
3 3
(a) From

0 ≤ (b + c)2 − 4bc = (p − a)2 − 4(a2 − pa + q) = −(3a2 − 2pa + 4q − p2 ),

we get a ∈ [0, a4 ], where


p+2
p
p2 − 3q
a4 = .
3
Since a2 ≤ a4 , f (a) is increasing on [a1 , a2 ] and decreasing on [0, a1 ] ∪ [a2 , a4 ]; there-
fore,
f (a) ≥ min{ f (a1 ), f (a4 )}.
We need to show that
min{ f (a1 ), f (a4 )} = g(p, q).
Indeed, from 2a1 + a4 = p and
q Æ q
a12 + q = 2a1 (p − a1 ), 2 a42 + q = a4 + p,

we get
f (a1 ) = f (a4 ) = g(p, q).

(b) Consider the cases 3q ≤ p2 ≤ 4q and p2 ≥ 4q.


Case 1: 3q ≤ p2 ≤ 4q. From

(b + c)2 − 4bc = (p − a)2 − 4(a2 − pa + q) = −(3a2 − 2pa + 4q − p2 ) ≥ 0,

we get a ∈ [a3 , a4 ], where

p+2
p p
p−2 p2 − 3q p2 − 3q
a3 = ≥ 0, a4 = .
3 3
Since a3 ≤ a1 ≤ a2 ≤ a4 , it follows that f (a) is increasing on [a1 , a2 ] and decreasing on
[a3 , a1 ] ∪ [a2 , a4 ]. Thus,
f (a) ≤ max{ f (a2 ), f (a3 )}.
138 Vasile Cîrtoaje

To complete the proof, we need to show that

max{ f (a2 ), f (a3 )} = h(p, q).

Indeed, from 2a2 + a3 = p and


q Æ q
a22 + q = 2a2 (p − a2 ), 2 a32 + q = a3 + p,

we get
f (a2 ) = f (a3 ) = h(p, q).
Case 2: p2 ≥ 4q. Assume that a = min{a, b, c}, a ≤ p/3. From

0 ≤ bc = q − a(b + c) = q − a(p − a) = a2 − pa + q,

we get a ∈ [0, a5 ] ∪ [a6 , p], where

p+
p p
p− p2 − 4q p2 − 4q
a5 = , a6 = .
2 2
Since a6 > p/3, it follows that a ∈ [0, a5 ]. Since a1 ≤ a5 ≤ a2 , f (a) is decreasing on
[0, a1 ] and increasing on [a1 , a5 ]; thus,

f (a) ≤ max{ f (0), f (a5 )}.

It remains to show that max{ f (0), f (a5 )} = h(p, q). Indeed, from a52 + q = pa5 and
r q Æ p
a5 + p − a5 = p+2 pa5 − a52 = p + 2 q,
p p

we get
f (0) = f (a5 ) = h(p, q).

Remark. Note the following particular cases:

(a) If a, b, c are nonnegative real numbers such that

a + b + c = a b + bc + ca = 4,

then p
2(1 + 10) p p p p
p ≤ a + b + b + c + c + a ≤ 2(1 + 2),
3
with left equality for a = 8/3 and b = c = 2/3 (or any cyclic permutation), and right
equality for a = 0 and b = c = 2 (or any cyclic permutation).
Symmetric Nonrational Inequalities 139

(b) If a, b, c are nonnegative real numbers such that

a + b + c = 4, a b + bc + ca = 5,

then p p
p p p p p 10 + 2 7
2+2 3≤ a+ b+ b+c+ c+a ≤ p ,
3
with left equality for a = 2 and b = c = 1 (or any cyclic permutation), and right equality
for a = 2/3 and b = c = 5/3 (or any cyclic permutation).

(c) If a, b, c are nonnegative real numbers such that

a + b + c = 11, a b + bc + ca = 7,

then p p p p p Æ p
3 6≤ a+b+ b+c+ c+a≤ 11 + 11 + 7,
with left equality for a = 31/3 and b = c = 1/3 (or any cyclic permutation), and right
equality for a = 0, b + c = 11 and bc = 7 (or any cyclic permutation).

P 1.89. If a, b are positive real numbers such that a + b = a4 + b4 , then


3 3
aa b b ≤ 1 ≤ aa b b .

(Vasile Cîrtoaje, 2008)

Solution. We will use the inequality

ln x ≤ x − 1, x > 0.

To prove this inequality, let us denote

f (x) = x − 1 − ln x, x > 0.

From
x −1
f 0 (x) =
,
x
it follows that f (x) is decreasing on (0, 1] and increasing on [1, ∞). Therefore,

f (x) ≥ f (1) = 0.

Using this inequality, we have

ln a a b b = a ln a + b ln b ≤ a(a − 1) + b(b − 1) = a2 + b2 − (a + b).


140 Vasile Cîrtoaje

Therefore, the left inequality a a b b ≤ 1 is true if a2 + b2 ≤ a + b. We write this inequality


in the homogeneous form

(a2 + b2 )3 ≤ (a + b)2 (a4 + b4 ),

which is equivalent to the obvious inequality

a b(a − b)(a3 − b3 ) ≥ 0.
1
Taking now x = in the inequality ln x ≤ x − 1 yields
a
a ln a ≥ a − 1.

Similarly,
b ln b ≥ b − 1,
and hence
3 3
ln a a b b = a3 ln a + b3 ln b ≤ a2 (a − 1) + b2 (b − 1) = a3 + b3 − (a2 + b2 ).
3 3
Thus, to prove the right inequality a a b b ≥ 1, it suffices to show that a3 + b3 ≥ a2 + b2 ,
which is equivalent to the homogeneous inequality

(a + b)(a3 + b3 )3 ≥ (a4 + b4 )(a2 + b2 )3 .

We can write this inequality as


A − 3B ≥ 0,
where
A = (a + b)(a9 + b9 ) − (a4 + b4 )(a6 + b6 ),
B = a2 b2 (a2 + b2 )(a4 + b4 ) − a3 b3 (a + b)(a3 + b3 ).
Since
A = a b(a3 − b3 )(a5 − b5 ), B = a2 b2 (a − b)(a5 − b5 ),
we get
A − 3B = a b(a − b)3 (a5 − b5 ) ≥ 0.
Both inequalities become equalities for a = b = 1.

P 1.90. If a, b are positive real numbers, then

a2a + b2b ≥ a a+b + b a+b .

(Vasile Cîrtoaje, 2010)


Symmetric Nonrational Inequalities 141

Solution. Assume that a ≥ b and consider the following two cases.


Case 1: a ≥ 1. Write the inequality as

a a+b (a a−b − 1) ≥ b2b (b a−b − 1).

For b ≤ 1, we have
a a+b (a a−b − 1) ≥ 0 ≥ b2b (b a−b − 1).
For b ≥ 1, the inequality is also true since

a a+b ≥ a2b ≥ b2b , a a−b − 1 ≥ b a−b − 1 ≥ 0.

Case 2: a ≤ 1. Since
a2a + b2b ≥ 2a a b b ,
it suffices to show that
2a a b b ≥ a a+b + b a+b ,
which can be written as  a b  ‹a
b
+ ≤ 2.
b a
By Bernoulli’s inequality, we get
 ‹a 
 a b a−b b b−a a b(a − b) a(b − a)
‹  ‹
b
+ = 1+ + 1+ ≤1+ +1+ = 2.
b a b a b a
The equality holds for a = b.
Conjecture 1. If a, b are positive real numbers, then

a4a + b4b ≥ a2a+2b + b2a+2b .

Conjecture 2. If a, b, c are positive real numbers, then

a3a + b3b + c 3c ≥ a a+b+c + b a+b+c + c a+b+c .

Conjecture 3. If a, b, c, d are positive real numbers, then

a4a + b4b + c 4c + d 4d ≥ a a+b+c+d + b a+b+c+d + c a+b+c+d + d a+b+c+d .

P 1.91. If a, b are positive real numbers, then

aa + b b ≥ a b + ba .

(M. Laub, 1985)


142 Vasile Cîrtoaje

Solution. Assume that a ≥ b. We will show that if a ≥ 1, then the inequality is true.
From
a a−b ≥ b a−b ,
we get
a b ba
bb ≥ .
aa
Therefore,

a b ba (a a − a b )(a a − b a )
aa + b b − a b − ba ≥ aa + − a b
− b a
= ≥ 0.
aa aa
Consider further the case 0 < b ≤ a < 1.
First Solution. Denoting
a
c = ab, d = bb, k= ,
b
where c ≥ d and k ≥ 1, the inequality becomes

c k − d k ≥ c − d.

Since the function f (x) = x k is convex for x ≥ 0, from the well-known inequality

f (c) − f (d) ≥ f 0 (d)(c − d),

we get
c k − d k ≥ kd k−1 (c − d).
Thus, it suffices to show that
kd k−1 ≥ 1,
which is equivalent to
b1−a+b ≤ a.
Indeed, since 0 < 1 − a + b ≤ 1, by Bernoulli’s inequality, we get

b1−a+b = [1 + (b − 1)]1−a+b ≤ 1 + (1 − a + b)(b − 1) = a − b(a − b) ≤ a.

The equality holds for a = b.


Second Solution. Denoting

ba ab a
c= , d= , k= ,
a b + ba a b + ba b
where c + d = 1 and k ≥ 1, the inequality becomes

ck a + d k−b ≥ 1.
Symmetric Nonrational Inequalities 143

By the weighted AM-GM inequality, we have

ck a + d k−b ≥ k ac · k−bd = k ac−bd .

Thus, it suffices to show that ac ≥ bd; that is,

a1−b ≥ b1−a ,

which is equivalent to f (a) ≥ f (b), where

ln x
f (x) = .
1− x
It is enough to prove that f (x) is an increasing function. Since

g(x) 1
f 0 (x) = , g(x) = − 1 + ln x.
(1 − x)2 x

we need to show that g(x) ≥ 0 for x ∈ (0, 1). Indeed, from


x −1
g 0 (x) = < 0,
x2
it follows that g(x) is strictly decreasing, hence g(x) > g(1) = 0.

P 1.92. If a, b are positive real numbers, then

a2a + b2b ≥ a2b + b2a .

Solution. Assume that a > b, and denote


a
c = ab, d = bb, k= ,
b
where c > d and k > 1. The inequality becomes

c 2k − d 2k ≥ c 2 − d 2 .

We shall show that

c 2k − d 2k > k(cd)k−1 (c 2 + d 2 ) > c 2 − d 2 .

The left inequality follows from Lemma below for x = (c/d)2 . The right inequality is
equivalent to
k(cd)k−1 > 1,
144 Vasile Cîrtoaje

b
(a b)a−b > ,
a
1+a− b
ln a > ln b.
1−a+ b
For fixed a, let us define
1+a− b
f (b) = ln a − ln b, 0 < b < a.
1−a+ b
If f 0 (b) < 0, then f (b) is strictly decreasing, and hence f (b) > f (a) = 0. Since
−2 1
f 0 (b) = ln a − ,
(1 − a + b)2 b
we need to show that g(a) > 0, where

(1 − a + b)2
g(a) = 2 ln a + .
b
From
2 2(1 − a + b) 2(a − b)(a − 1)
g 0 (a) = − = < 0,
a b ab
it follows that g(a) is strictly decreasing on [b, 1), therefore g(a) > g(1) = b > 0. This
completes the proof. The equality holds for a = b.
Lemma. Let k and x be positive real numbers. If either k > 1 and x ≥ 1, or 0 < k < 1
and 0 < x ≤ 1, then
k−1
x k − 1 ≥ k x 2 (x − 1).
Proof. We need to show that f (x) ≥ 0, where
k−1
f (x) = x k − 1 − k x 2 (x − 1).

We have
1 k−3 k+1
f 0 (x) = k x 2 g(x), g(x) = 2x 2 − (k + 1)x + k − 1.
2
Since € k−1 Š
g 0 (x) = (k + 1) x 2 − 1 ≥ 0,
g(x) is increasing. If x ≥ 1, then g(x) ≥ g(1) = 0, f (x) is increasing, hence f (x) ≥
f (1) = 0. If 0 < x ≤ 1, then g(x) ≤ g(1) = 0, f (x) is decreasing, hence f (x) ≥ f (1) =
0. The equality holds for x = 1.

Remark. The following more general inequality holds for 0 < k ≤ e and any positive
numbers a and b (Vasile Cîrtoaje, 2006):

a ka + b kb ≥ a kb + b ka .
Symmetric Nonrational Inequalities 145

Conjecture 1. If 0 < k ≤ e and a, b ∈ (0, 4], then


p
2 a ka b kb ≥ a kb + b ka .

Conjecture 2. If a, b ∈ (0, 5], then

2a a b b ≥ a2b + b2a .

Conjecture 3. If a, b ∈ [0, 5], then


 a+b
a2 + b2
 2
≥ a2b + b2a .
2

Note that Conjecture 1 has been proved for a, b ∈ (0, 1] by A. Coronel and F. Huancas
(2014), and also by L. Matejicka (2014).

P 1.93. If a, b are nonnegative real numbers such that a + b = 2, then

(a) a b + b a ≤ 1 + a b;

(b) a2b + b2a ≤ 1 + a b.


(Vasile Cîrtoaje, 2007)

Solution. Without loss of generality, assume that a ≥ b. Since 0 ≤ b ≤ 1 and 0 ≤


a − 1 ≤ 1, by Bernoulli’s inequality, we have

a b ≤ 1 + b(a − 1) = 1 + b − b2

and
b a = b · b a−1 ≤ b[1 + (a − 1)(b − 1)] = b2 (2 − b).
(a) We have

a b + b a − 1 − a b ≤ (1 + b − b2 ) + b2 (2 − b) − 1 − (2 − b)b = −b(b − 1)2 ≤ 0.

The equality holds for a = b = 1, for a = 2 and b = 0, and for a = 0 and b = 2.


(b) We have

a2b + b2a − 1 − a b ≤ (1 + b − b2 )2 + b4 (2 − b)2 − 1 − (2 − b)b


= b3 (b − 1)2 (b − 2) = −a b3 (b − 1)2 ≤ 0.

The equality holds for a = b = 1, for a = 2 and b = 0, and for a = 0 and b = 2.


146 Vasile Cîrtoaje

P 1.94. If a, b are nonnegative real numbers such that a2 + b2 = 2, then

a2b + b2a ≤ 1 + a b.

(Vasile Cîrtoaje, 2007)

Solution. Without loss of generality, assume that a ≥ 1 ≥ b. Applying Bernoulli’s in-


equality gives
a b ≤ 1 + b(a − 1),
hence
a2b ≤ (1 + a b − b)2 .
Also, since 0 ≤ b ≤ 1 and 2a ≥ 2, we have

b2a ≤ b2 .

Therefore, it suffices to show that

(1 + a b − b)2 + b2 ≤ 1 + a b,

which can be written as

b(2 + 2a b − a − 2b − a2 b) ≥ 0.

So, we need to show that

2 + 2a b − a − 2b − a2 b ≥ 0,

which is equivalent to
2(1 − a)(1 − b) + a(1 − a b) ≥ 0,
4(1 − a)(1 − b) + a(a − b)2 ≥ 0.
Since a ≥ 1, it suffices to show that

4(1 − a)(1 − b) + (a − b)2 ≥ 0.

Indeed,

4(1 − a)(1 − b) + (a − b)2 = −4(a − 1)(1 − b) + [(a − 1) + (1 − b)]2


= [(a − 1) − (1 − b)]2 = (a + b − 2)2 ≥ 0.
p p
The equality holds for a = b = 1, for a = 2 and b = 0, and for a = 0 and b = 2.
Symmetric Nonrational Inequalities 147

P 1.95. If a, b are positive real numbers, then


a+b
a a b b ≤ (a2 − a b + b2 ) 2 .

Solution. By the weighted AM-GM inequality, we have


a b
a · a + b · b ≥ (a + b)a a+b b a+b ,
a+b
a2 + b2

≥ aa b b .
a+b
Thus, it suffices to show that
2
a2 + b2

2 2
a − ab + b ≥ ,
a+b

which is equivalent to
(a + b)(a3 + b3 ) ≥ (a2 + b2 )2 ,
a b(a − b)2 ≥ 0.
The equality holds for a = b.

P 1.96. If a, b ∈ (0, 1], then


a a b b ≤ 1 − a b + a2 b2 .
(Vasile Cîrtoaje, 2010)

Solution. We claim that


x x ≤ x2 − x + 1
for all x ∈ (0, 1]. If this is true, then

1 − a b + a2 b2 − a a b b ≥ 1 − a b + a2 b2 − (a2 − a + 1)(b2 − b + 1)
= (a + b)(1 − a)(1 − b) ≥ 0.

To prove the inequality x x ≤ x 2 − x + 1, we show that f (x) ≤ 0 for x ∈ (0, 1], where

f (x) = x ln x − ln(x 2 − x + 1).

We have
2x − 1
f 0 (x) = ln x + 1 − ,
x2 − x +1
148 Vasile Cîrtoaje

(1 − x)(1 − 2x − x 2 − x 4 )
f 00 (x) = .
x(x 2 − x + 1)2
Let x 1 ∈ (0, 1) be the positive root of the equation x 4 + x 2 + 2x = 1. Then, f 00 (x) > 0
for x ∈ (0, x 1 ) and f 00 (x) < 0 for x ∈ (x 1 , 1), hence f 0 is strictly increasing on (0, x 1 ]
and strictly decreasing on [x 1 , 1]. Since lim x→0 f 0 (x) = −∞ and f 0 (1) = 0, there is
x 2 ∈ (0, x 1 ) such that f 0 (x 2 ) = 0, f 0 (x) < 0 for x ∈ (0, x 2 ) and f 0 (x) > 0 for x ∈ (x 2 , 1).
Therefore, f is decreasing on (0, x 2 ] and increasing on [x 2 , 1]. Since lim x→0 f (x) = 0
and f (1) = 0, it follows that f (x) ≤ 0 for x ∈ (0, 1]. The proof is completed. The
equality holds for a = b = 1.

P 1.97. If a, b are positive real numbers such that a + b ≤ 2, then


 a  b  b ‹a
+ ≤ 2.
b a

(Vasile Cîrtoaje, 2010)

Solution. Using the substitution a = t c and b = t d, where c, d, t are positive real


numbers such that c + d = 2 and t ≤ 1, we need to show that
 c t d  d ‹t c
+ ≤ 2.
d c

Write this inequality as f (t) ≤ 2, where


 c d  ‹c
d
f (t) = At + B t , A = , B= .
d c

Since f (t) is a convex function, we have

f (t) ≤ max{ f (0), f (1)} = max{2, f (1)}.

Therefore, it suffices to show that f (1) ≤ 2; that is,

2c c d d ≥ c 2 + d 2 .

Setting c = 1 + x and d = 1 − x, where 0 ≤ x < 1, this inequality turns into

(1 + x)1+x (1 − x)1−x ≥ 1 + x 2 ,

which is equivalent to f (x) ≥ 0, where

f (x) = (1 + x) ln(1 + x) + (1 − x) ln(1 − x) − ln(1 + x 2 ).


Symmetric Nonrational Inequalities 149

We have
2x
f 0 (x) = ln(1 + x) − ln(1 − x) − ,
1 + x2
1 1 2(1 − x 2 ) 8x 2
f 00 (x) = + − = .
1 + x 1 − x (1 + x 2 )2 (1 − x 2 )(1 + x 2 )2
Since f 00 (x) ≥ 0 for x ∈ [0, 1), it follows that f 0 is increasing, f 0 (x) ≥ f 0 (0) = 0, f (x)
is increasing, hence f (x) ≥ f (0) = 0. The proof is completed. The equality holds for
a = b.

P 1.98. If a, b are positive real numbers such that a + b = 2, then


1
2a a b b ≥ a2b + b2a + (a − b)2 .
2
(Vasile Cîrtoaje, 2010)

Solution. According to the inequalities in P 1.93-(b) and P 1.97, we have

a2b + b2a ≤ 1 + a b

and
2a a b b ≥ a2 + b2 .
Therefore, it suffices to show that
1
a2 + b2 ≥ 1 + a b + (a − b)2 .
2
which is an identity. The equality holds for a = b = 1.

P 1.99. If a, b ∈ (0, 1] or a, b ∈ [1, ∞), then

2a a b b ≥ a2 + b2 .

Solution. For a = x and b = 1, the desired inequality becomes

2x x ≥ x 2 + 1, x > 0.

If this inequality is true, then

4a a b b − 2(a2 + b2 ) ≥ (a2 + 1)(b2 + 1) − 2(a2 + b2 ) = (a2 − 1)(b2 − 1) ≥ 0.


150 Vasile Cîrtoaje

To prove the inequality 2x x ≥ x 2 + 1, we show that f (x) ≥ 0, where

f (x) = ln 2 + x ln x − ln(x 2 + 1).

We have
2x
f 0 (x) = ln x + 1 − ,
x2 +1
x 2 (x + 1)2 + (x − 1)2
f 00 (x) = .
x(x 2 + 1)2
Since f 00 (x) > 0 for x > 0, f 0 is strictly increasing. Since f 0 (1) = 0, it follows that
f 0 (x) < 0 for x ∈ (0, 1) and f 0 (x) > 0 for x ∈ (1, ∞). Therefore, f is decreasing on
(0, 1] and increasing on [1, ∞), hence f (x) ≥ f (1) = 0 for x > 0. This completes the
proof. The equality holds for a = b = 1.

P 1.100. If a, b are positive real numbers, then

2a a b b ≥ a2 + b2 .

(Vasile Cîrtoaje, 2014)

Solution. By Lemma below, it suffices to show that

(a4 − 2a3 + 4a2 − 2a + 3)(b4 − 2b3 + 4b2 − 2b + 3) ≥ 8(a2 + b2 ),

which is equivalent to A ≥ 0, where

A =a4 b4 − 2a3 b3 (a + b) + 4a2 b2 (a2 + b2 + a b) − [2a b(a3 + b3 ) + 8a2 b2 (a + b)]


+ [3(a4 + b4 ) + 4a b(a2 + b2 ) + 16a2 b2 ] − [6(a3 + b3 ) + 8a b(a + b)]
+ 4(a2 + b2 + a b) − 6(a + b) + 9.

We can check that

A = [a2 b2 − a b(a + b) + a2 + b2 − 1]2 + B,

where

B =a2 b2 (a + b)2 − 6a2 b2 (a + b) + [2(a4 + b4 ) + 4a b(a2 + b2 ) + 16a2 b2 ]


− [6(a3 + b3 ) + 10a b(a + b)] + [6(a2 + b2 ) + 4a b] − 6(a + b) + 8.

Also, we have
B = [a b(a + b) − 3a b + 1]2 + C,
Symmetric Nonrational Inequalities 151

where

C =[2(a4 + b4 ) + 4a b(a2 + b2 ) + 7a2 b2 ] − [6(a3 + b3 ) + 12a b(a + b)]


+ [6(a2 + b2 ) + 10a b] − 6(a + b) + 7,

and
C = (a b − 1)2 + 2D,
where

D =[a4 + b4 + 2a b(a2 + b2 ) + 3a2 b2 ] − [3(a3 + b3 ) + 6a b(a + b)]


+ 3(a + b)2 − 3(a + b) + 3,

It suffices to show that D ≥ 0. Indeed,

D =[(a + b)4 − 2a b(a + b)2 + a2 b2 ] − 3[(a + b)3 − a b(a + b)]


+ 3(a + b)2 − 3(a + b) + 3
=[(a + b)2 − a b]2 − 3(a + b)[(a + b)2 − a b] + 3(a + b)2 − 3(a + b) + 3
˜2 ‹2
3 a+b
• 
2
= (a + b) − a b − (a + b) + 3 − 1 ≥ 0.
2 2
This completes the proof. The equality holds for a = b = 1.
Lemma. If x > 0, then
1
x x ≥ x + (x − 1)2 (x 2 + 3).
4
Proof. We need to show that f (x) ≥ 0 for x > 0, where

f (x) = ln 4 + x ln x − ln g(x), g(x) = x 4 − 2x 3 + 4x 2 − 2x + 3.

We have
2(2x 3 − 3x 2 + 4x − 1)
f 0 (x) = 1 + ln x − ,
g(x)
x 8 + 6x 4 − 32x 3 + 48x 2 − 32x + 9 (x − 1)2 h(x)
f 00 (x) = = ,
g 2 (x) g 2 (x)
where
h(x) = x 6 + 2x 5 + 3x 4 + 4x 3 + 11x 2 − 14x + 9.
Since
h(x) > 7x 2 − 14x + 7 = 7(x − 1)2 ≥ 0,
we have f 00 (x) ≥ 0, hence f 0 is strictly increasing on (0, ∞). Since f 0 (1) = 0, it follows
that f 0 (x) < 0 for x ∈ (0, 1) and f 0 (x) > 0 for x ∈ (1, ∞). Therefore, f is decreasing
on (0, 1] and increasing on [1, ∞), hence f (x) ≥ f (1) = 0 for x > 0.
152 Vasile Cîrtoaje

P 1.101. If a, b are positive real numbers, then


 a+b
a2 + b2
 2
a b
a b ≥ .
2

First Solution. Using the substitution a = b x, where x > 0, the inequality becomes as
follows:
 bx+b
b x + b2 2
 2 2
bx b
(b x) b ≥ ,
2
 x+1
b x + b2 2
 2 2
x
(b x) b ≥ ,
2
 x+1
+
 2 2
x 1
b x+1 x x ≥ b x+1 ,
2
 x+1
x +1 2
 2
x
x ≥ .
2
It is true if f (x) ≥ 0 for all x > 0, where

x 1 x2 + 1
f (x) = ln x − ln .
x +1 2 2
We have
1 1 x g(x)
f 0 (x) = ln x + − 2 = ,
(x + 1)2 x + 1 x + 1 (x + 1)2
where
x2 − 1
g(x) = ln x − .
x2 + 1
Since
(x 2 − 1)2
g 0 (x) = ≥ 0,
x(x 2 + 1)2
g is strictly increasing, therefore g(x) < 0 for x ∈ (0, 1), g(1) = 0, g(x) > 0 for x ∈
(1, ∞). Thus, f is decreasing on (0, 1] and increasing on [1, ∞), hence f (x) ≥ f (1) =
0. This completes the proof. The equality holds for a = b.
Second Solution. Write the inequality in the form

a+b a2 + b2
a ln a + b ln b ≥ log .
2 2
Without loss of generality, consider that a + b = 2k, k > 0, and denote

a = k + x, b = k − x, 0 ≤ x < k.
Symmetric Nonrational Inequalities 153

We need to show that f (x) ≥ 0, where


f (x) = (k + x) ln(k + x) + (k − x) ln(k − x) − k ln(x 2 + k2 ).
We have
2k x
f 0 (x) = ln(k + x) − ln(k − x) − ,
x2+ k2

1 1 2k(x 2 − k2 )
f 00 (x) = + +
k+x k−x (x 2 + k2 )2
8k2 x 2
= 2 .
(k − x 2 )(x 2 + k2 )2
Since f 00 (x) ≥ 0 for x ≥ 0, f 0 is increasing, hence f 0 (x) ≥ f 0 (0) = 0. Therefore, f is
increasing on [0, k), hence f (x) ≥ f (0) = 0.

Remark. For a + b = 2, this inequality can be rewritten as


2a a b b ≥ a2 + b2 ,
 a  b  b ‹a
2≥ + .
b a
Also, for a + b = 1, the inequality becomes
2a2a b2b ≥ a2 + b2 ,
 a 2b  b ‹2a
2≥ + .
b a

P 1.102. If a, b are positive real numbers such that a2 + b2 = 2, then


1
2a a b b ≥ a2b + b2a + (a − b)2 .
2
(Vasile Cîrtoaje, 2010)
Solution. According to the inequalities in P 1.94 and P 1.100, we have
a2b + b2a ≤ 1 + a b
and
2a a b b ≥ a2 + b2 .
Therefore, it suffices to show that
1
a2 + b2 ≥ 1 + a b + (a − b)2 ,
2
which is an identity. The equality holds for a = b = 1.
154 Vasile Cîrtoaje

P 1.103. If a, b ∈ (0, 1], then


1 1
 ‹
(a2 + b2 ) 2a
+ 2b ≤ 4.
a b
(Vasile Cîrtoaje, 2014)
Solution. For a = x and b = 1, the desired inequality becomes
1 + x2
x 2x ≥ , x ∈ (0, 1].
3 − x2
If this inequality is true, it suffices to show that

3 − a2 3 − b2
 
2 2
(a + b ) + ≤ 4,
1 + a2 1 + b2
which is equivalent to

a2 b2 (2 + a2 + b2 ) + 2 − (a2 + b2 ) − (a2 + b2 )2 ≥ 0,

(2 + a2 + b2 )(1 − a2 )(1 − b2 ) ≥ 0.
1 + x2
To prove the inequality x 2x ≥ for 0 < x ≤ 1 , we show that f (x) ≥ 0, where
3 − x2
1 1
f (x) = x ln x + ln(3 − x 2 ) − ln(1 + x 2 ).
2 2
We have
x x
f 0 (x) = 1 + ln x − − ,
3− x 2 1 + x2

1 3 + x2 1 − x2
f 00 (x) = − −
x (3 − x 2 )2 (1 + x 2 )2
(1 − x)(9 + 6x − x 3 ) 1 − x2
= − .
x(3 − x)2 (1 + x 2 )2
We will show that f 00 (x) > 0 for 0 < x < 1. This is true if
9 + 6x − x 3 1+ x
− > 0.
x(3 − x)2 (1 + x 2 )2
Indeed,
9 + 6x − x 3 1+ x 9 1+ x 1
− > − = > 0.
x(3 − x)2 (1 + x )
2 2 9x x(1 + x)2 1+ x
Since f 00 (x) > 0, f 0 is strictly increasing on (0, 1]. Since f 0 (1) = 0, it follows that
f 0 (x) < 0 for x ∈ (0, 1), f is strictly decreasing on (0, 1], hence f (x) ≥ f (1) = 0. This
completes the proof. The equality holds for a = b = 1.
Symmetric Nonrational Inequalities 155

P 1.104. If a, b are positive real numbers such that a + b = 2, then

a b b a + 2 ≥ 3a b.

(Vasile Cîrtoaje, 2010)

Solution. Setting a = 1+ x and b = 1− x, where 0 ≤ x < 1, the inequality is equivalent


to
(1 + x)1−x (1 − x)1+x ≥ 1 − 3x 2 .
1
Consider further the non-trivial case 0 ≤ x < p , and write the desired inequality as
3
f (x) ≥ 0, where

f (x) = (1 − x) ln(1 + x) + (1 + x) ln(1 − x) − ln(1 − 3x 2 ).

We have
1− x 1+ x 6x
f 0 (x) = − ln(1 + x) + ln(1 − x) + − + ,
1 + x 1 − x 1 − 3x 2
1 00 −1 2(x 2 + 1) 3(3x 2 + 1)
f (x) = − + .
2 1 − x 2 (1 − x 2 )2 (1 − 3x 2 )2
1
Making the substitution t = x 2 , 0 ≤ t < , we get
3
1 00 3(3t + 1) t +3 4t(5 − 9t)
f (x) = − = > 0.
2 (3t − 1)2 (t − 1)2 (t − 1)2 (3t − 1)2

Therefore, f 0 (x) is strictly increasing, f 0 (x) ≥ f 0 (0) = 0, f (x) is strictly increasing, and
hence f (x) ≥ f (0) = 0. This completes the proof. The equality holds for a = b = 1.

P 1.105. If a, b ∈ [0, 1], then

a b−a + b a−b + (a − b)2 ≤ 2.

(Vasile Cîrtoaje, 2010)

Solution (by Vo Quoc Ba Can). Without loss of generality, assume that a ≥ b. Using the
substitution
c = a − b,
we need to show that
(b + c)−c + b c + c 2 ≤ 2
for
0 ≤ b ≤ 1 − c, 0 ≤ c ≤ 1.
156 Vasile Cîrtoaje

If c = 1, then b = 0, and the inequality is an equality. Also, for c = 0, the inequality is


an equality. Consider further that 0 < c < 1. We need to show that f (x) ≤ 0, where

f (x) = (x + c)−c + x c + c 2 − 2, x ∈ [0, 1 − c].

We claim that f 0 (x) > 0 for x > 0. Then, f (x) is strictly increasing on [0, 1 − c], hence

f (x) ≤ f (1 − c) = (1 − c)c − (1 − c 2 ).

In addition, by Bernoulli’s inequality,

f (x) ≤ 1 + c(−c) − (1 − c 2 ) = 0.

Since
c[(x + c)1+c − x 1−c ]
f 0 (x) = ,
(x + c)1+c x 1−c
we have f 0 (x) > 0 for x > 0 if and only if
1−c
x + c > x 1+c .
2c
Let d = 1−c . Using the weighted AM-GM inequality, we have

‹  1 ‹ 1+c
1 − c d 1+d 1 + c 1 − c 1−c 1−c
 
1−c

x +c =1· x +d · ≥ (1 + d) x = x 1+c .
2 2 2 2

Thus, it suffices to show that


‹ 1−c
1+c 1 − c 1+c

≥ .
2 2
Indeed, using Bernoulli’s inequality, we get
1−c ‹ 1−c
1−c 1 + c 1+c 1−c 1+c 1+c
 ‹ 1+c 
= 1− ≤1− · = .
2 2 1+c 2 2
The equality holds for a = b, for a = 1 and b = 0, and for a = 0 and b = 1.

P 1.106. If a, b are nonnegative real numbers such that a + b ≤ 2, then


7
a b−a + b a−b + (a − b)2 ≤ 2.
16
(Vasile Cîrtoaje, 2010)
Symmetric Nonrational Inequalities 157

Solution. Assume that a > b and use the substitution

c = a − b.

We need to show that


7 2
a−c + (a − c)c + c ≤2
16
for
c
0 < c ≤ 2, c ≤ a ≤1+ .
2
Equivalently, we need to show that f (x) ≤ 0 for
h ci
x ∈ c, 1 + , 0<c≤2,
2
where
7 2
f (x) = x −c + (x − c)c + c − 2.
16
As we have shown
h in the preceding proof, f 0 (x) > 0 for x > c. Therefore, f (x) is strictly
c i
increasing on c, 1 + , and hence
2
 c  c −c  c c 7 2
f (x) ≤ f 1 + = 1+ + 1− + c − 2.
2 2 2 16
To end the proof we need to show that
 c −c  c c 7 2
1+ + 1− + c ≤2
2 2 16
for c ∈ [0, 2]. Indeed, by Lemma 1 and Lemma 2 below, we have
 c −c 3 2
1+ + c ≤ 1,
2 16
 c c 1 2
1− + c ≤ 1.
2 4
Adding these inequalities yields the desired inequality. The equality holds for a = b, for
a = 2 and b = 0, and for a = 0 and b = 2.
Lemma 1. If 0 ≤ x ≤ 2, then
 x −x 3 2
1+ + x ≤ 1,
2 16
with equality for x = 0 and x = 2.
Proof. We need to show that f (x) ≤ 0 for 0 ≤ x ≤ 2, where
3 2
 ‹
 x
f (x) = −x ln 1 + − ln 1 − x .
2 16
158 Vasile Cîrtoaje

We have
 x x(3x 2 + 6x − 4)
f 0 (x) = − ln 1 + + ,
2 (2 + x)(16 − 3x 2 )
g(x)
f 00 (x) = ,
(2 + x)2 (16 − 3x 2 )2
where
g(x) = −9x 5 − 18x 4 + 168x 3 + 552x 2 + 128x − 640.
Since g(x 1 ) = 0 for x 1 ≈ 0, 88067, g(x) < 0 for x ∈ [0, x 1 ) and g(x) > 0 for x ∈ (x 1 , 2],
f 0 is strictly decreasing on [0, x 1 ] and strictly increasing on [x 1 , 2]. Since f 0 (0) = 0
5
and f 0 (2) = − ln 2 + > 0, there is x 2 ∈ (x 1 , 2) such that f 0 (x 2 ) = 0, f 0 (x) < 0 for
2
x ∈ (0, x 2 ), and f 0 (x) > 0 for x ∈ (x 2 , 2]. Therefore, f is decreasing on [0, x 2 ] and
increasing on [x 2 , 2]. Since f (0) = f (2) = 0, it follows that f (x) ≤ 0 for x ∈ [0, 2].
Lemma 2. If 0 ≤ x ≤ 2, then
 x x 1 2
1− + x ≤ 1,
2 4
with equality for x = 0 and x = 2.
Proof. We need to show that f (x) ≤ 0, where
1 2
 ‹
 x
f (x) = x ln 1 − − ln 1 − x .
2 4
We have
 x x2
f 0 (x) = ln 1 − − ,
2 4 − x2
−1 8x
f 00 (x) = − .
2 − x (4 − x 2 )2
Since f 00 < 0 for 0 ≤ x < 2, f 0 is strictly decreasing, hence f 0 (x) ≤ f 0 (0) = 0, f is
strictly decreasing, therefore f (x) ≤ f (0) = 0 for x ∈ [0, 2].
1
Conjecture. If a, b are nonnegative real numbers such that a + b = , then
4
a2(b−a) + b2(a−b) ≤ 2.

P 1.107. If a, b are nonnegative real numbers such that a + b ≤ 4, then

a b−a + b a−b ≤ 2.

(Vasile Cîrtoaje, 2010)


Symmetric Nonrational Inequalities 159

Solution. Without loss of generality, assume that a ≥ b. Consider first that a − b ≥ 2.


We have
a ≥ a − b ≥ 2,
and from
4 ≥ a + b = (a − b) + 2b ≥ 2 + 2b,
we get b ≤ 1. Clearly, the desired inequality is true because

a b−a < 1, b a−b ≤ 1.

Since the case a − b = 0 is trivial, consider further that 0 < a − b < 2 and use the
substitution
c = a − b.
So, we need to show that
a−c + (a − c)c ≤ 2
for
c
0 < c < 2, c ≤ a ≤2+ .
2
Equivalently, we need to show that f (x) ≤ 0 for
h ci
x ∈ c, 2 + , 0<c<2,
2
where
f (x) = x −c + (x − c)c − 2.
The derivative
c[x 1+c − (x − c)1−c ]
f 0 (x) =
x 1+c (x − c)1−c
has the same sign as

g(x) = (1 + c) ln x − (1 − c) ln(x − c).

We have
c(2x − 1 − c)
g 0 (x) = .
x(x − c)
5
• ˜
Case 1: c = 1. We need to show that f (x) ≤ 0 for x ∈ 1, , where
2

x 2 − 3x + 1
f (x) = .
x
Indeed, we have
(x − 1)(2x − 5) + (x − 3)
f (x) = < 0.
2x
160 Vasile Cîrtoaje

Case 2: 1 < c < 2. Since


2x − 1 − c ≥ 2c − 1 − c > 0,
we have g 0 (x) > 0, hence g(x) is strictly increasing. Since g(x) → −∞ when x → c
and  c  c  c
g 2+ = (1 + c) ln 2 + + (c − 1) ln 2 −
2 2 2
c2
 
 c   c 
> (c − 1) ln 2 + + (c − 1) ln 2 − = (c − 1) ln 4 − > 0,
2 2 4
 c
there exists x 1 ∈ c, 2 + such that g(x 1 ) = 0, g(x) < 0 for x ∈ (c, x 1 ) and g(x) > 0
2
 c h ci
for x ∈ x 1 , 2 + . Thus, f (x) is decreasing on [c, x 1 ] and increasing on x 1 , 2 + .
2 2
 c
Then, it suffices to show that f (c) ≤ 0 and f 2 + ≤ 0. We have
2
f (c) = c −c − 2 < 1 − 2 < 0.
 c
Also, the inequality f 2 + ≤ 0 has the form
2
 c −c  c c
2+ + 2− ≤ 2,
2 2
which follows immediately from Lemma 1 below.
h ci
Case 3: 0 < c < 1. We claim that g(x) > 0 for x ∈ c, 2 + . From
2
c(2x − 1 − c)
g 0 (x) = ,
x(x − c)

1+c 1+c
 ˜ • ˜
c
it follows that g(x) is decreasing on c, , and increasing on , 2 + . Then,
2 2 2
1+c 1+c 1−c
 ‹
g(x) ≥ g = (1 + c) ln − (1 − c) ln ,
2 2 2
and it suffices to show that
‹1+c ‹1−c
1+c 1−c
 
> .
2 2
This inequality follows from Bernoulli’s inequality, as follows
‹1+c
1+c 1 − c 1+c (1 + c)(1 − c) 1 + c 2
  ‹
= 1− >1− =
2 2 2 2
Symmetric Nonrational Inequalities 161

and ‹1−c
1−c 1 + c 1−c (1 − c)(1 + c) 1 + c 2
  ‹
= 1− <1− = .
2 2 2 2
h ci
Since g(x) > 0 involves f 0 (x) > 0, it follows that f (x) is strictly increasing on c, 2 + ,
2
and hence  c 
f (x) ≤ f 2 + .
2
 c
So, we need to show that f 2 + ≤ 0 for 0 < c < 2, which follows immediately from
2
Lemma 1.
The proof is completed. The equality holds for a = b.
Lemma 1. If 0 ≤ c ≤ 2, then
 c −c  c c
2+ + 2− ≤ 2.
2 2
Proof. According to Lemma 2, the following inequalities hold for c ∈ [0, 2]:
 c −c c3
2+ ≤ 1 − c ln 2 + ,
2 9
 c c c3
2− ≤ 1 + c ln 2 − .
2 9
Summing these inequalities, the desired inequality follows.
Lemma 2. If
−2 ≤ x ≤ 2,
then
 x x x3
2− ≤ 1 + x ln 2 − .
2 9
Proof. We have
ln 2 ≈ 0.693 < 7/9.
If x ∈ [0, 2], then
x3 x3 8
1 + x ln 2 − ≥1− ≥ 1 − > 0.
9 9 9
Also, if x ∈ [−2, 0], then

x3 7x x 3 8 + 7x − x 3
1 + x ln 2 − ≥1+ − >
9 9 9 9
2(x + 2)2 + (−x)(x + 1)2
= > 0.
9
162 Vasile Cîrtoaje

So, we can write the desired inequality as f (x) ≥ 0, where

x3
   x
f (x) = ln 1 + d x − − x ln 2 − , d = ln 2.
9 2

We have
9d − 3x 2 x  x
f 0 (x) = + − ln 2 − .
9 + 9d x − x 3 4 − x 2
Since f (0) = 0, it suffices to show that f 0 (x) ≤ 0 for x ∈ [−2, 0], and f 0 (x) ≥ 0 for
x ∈ [0, 2]; that is, x f 0 (x) ≤ 0 for x ∈ [−2, 2]. According to Lemma 3, the inequality
x f 0 (x) ≤ 0 is true if x g(x) ≥ 0, where

9d − 3x 2 1 2
 ‹
x x
g(x) = + − d− − x .
9 + 9d x − x 3 4 − x 4 32
We have

9d − 3x 2
  
1 2
‹
x x
g(x) = −d + + + x
9 + 9d x − x 3 4 − x 4 32
 2
d x − 3x − 9d 2 64 − 4x − x 2

=x + ,
9 + 9d x − x 3 32(4 − x)

hence
x 2 g1 (x)
x g(x) = ,
32(4 − x)(9 + 9d x − x 3 )
where

g1 (x) =32(4 − x)(d x 2 − 3x − 9d 2 ) + (64 − 4x − x 2 )(9 + 9d x − x 3 )


=x 5 + 4x 4 − (64 + 41d)x 3 + (87 + 92d)x 2 + 12(24d 2 + 48d − 35)x
+ 576(1 − 2d 2 ).

Since g1 (x) ≥ 0 for x ∈ [a1 , b1 ], where a1 ≈ −12.384 and b1 =≈ 2.652, we have


g1 (x) ≥ 0 for x ∈ [−2, 2].
Lemma 3. If x < 4, then
xh(x) ≤ 0,
where
1 2
 ‹
 x x
h(x) = ln 2 − − ln 2 − − x .
2 4 32
Proof. From
−x 2
h0 (x) = ≤ 0,
16(4 − x)
Symmetric Nonrational Inequalities 163

it follows that h(x) is decreasing. Since h(0) = 0, we have h(x) ≥ 0 for x ≤ 0, and
h(x) ≤ 0 for x ∈ [0, 4); that is, xh(x) ≤ 0 for x < 4.

1
P 1.108. Let a, b be positive real numbers such that a + b = 2. If k ≥ , then
2
kb ka
a a b b ≥ 1.

(Vasile Cîrtoaje, 2010)

Solution. Setting a = 1+ x and b = 1− x, where 0 ≤ x < 1, the inequality is equivalent


to
(1 + x)k(1−x) ln(1 + x) + (1 − x)k(1+x) ln(1 − x) ≥ 0.
Consider further the non-trivial case 0 < x < 1, and write the desired inequality as
f (x) ≥ 0, where

f (x) = k(1 − x) ln(1 + x) − k(1 + x) ln(1 − x) + ln ln(1 + x) − ln(− ln(1 − x)).

We have
2k(1 + x 2 ) 1 1
f 0 (x) = − k ln(1 − x 2 ) + +
1− x 2 (1 + x) ln(1 + x) (1 − x) ln(1 − x)
2k 1 1
> + +
1− x 2 (1 + x) ln(1 + x) (1 − x) ln(1 − x)
1 1 1
≥ + +
1− x 2 (1 + x) ln(1 + x) (1 − x) ln(1 − x)
g(x)
= ,
(1 − x ) ln(1 + x) ln(1 − x)
2

where
g(x) = ln(1 + x) ln(1 − x) + (1 + x) ln(1 + x) + (1 − x) ln(1 − x).
If g(x) ≤ 0, then f 0 (x) > 0, f (x) is strictly increasing, and hence

f (x) > lim f (x) = 0.


x→0

Thus, it suffices to show that g(x) ≤ 0. We have


−x
g 0 (x) = h(x),
1 − x2
where
h(x) = (1 + x) ln(1 + x) + (1 − x) ln(1 − x).
164 Vasile Cîrtoaje

Since
1+ x
h0 (x) = ln > 0,
1− x
h(x) is strictly increasing, h(x) > h(0) = 0, g 0 (x) < 0, g(x) is strictly decreasing, and
hence g(x) < g(0) = 0. This completes the proof. The equality holds for a = b = 1.

P 1.109. If a, b are positive real numbers such that a + b = 2, then


p p
a b
a b ≥ 1.

(Vasile Cîrtoaje, 2010)

Solution. For a = b = 1, the equality holds. In what follows, we will assume that
a > 1 > b. Taking logarithms of both sides, the inequality becomes in succession
p p
a ln a + b ln b ≥ 0,
p p
a ln a ≥ b(− ln b),
1 1
ln a + ln ln a ≥ ln b + ln(− ln b).
2 2
Setting a = 1 + x and b = 1 − x, we need to show that f (x) ≥ 0 for 0 < x < 1, where

1 1
f (x) = ln(1 + x) − ln(1 − x) + ln ln(1 + x) − ln(− ln(1 − x)).
2 2
We have
1 1 1
f 0 (x) = + + .
1− x 2 (1 + x) ln(1 + x) (1 − x) ln(1 − x)
As shown in the proof of the preceding P 1.108, we have f 0 (x) > 0. Therefore, f (x) is
strictly increasing and
f (x) > lim f (x) = 0.
x→0

The equality holds for a = b = 1.

P 1.110. If a, b are positive real numbers such that a + b = 2, then

1
1 − a a+1 b b+1 ≥ (1 − a b)2 .
3
(Vasile Cîrtoaje, 2010)
Symmetric Nonrational Inequalities 165

Solution. Putting a = 1 + x and b = 1 − x, 0 ≤ x < 1, the inequality becomes


1 4
(1 + x)2+x (1 − x)2−x ≤ 1 − x .
3
Write this inequality as f (x) ≤ 0, where
1
 ‹
f (x) = (2 + x) ln(1 + x) + (2 − x) ln(1 − x) − ln 1 − x 4 .
3
We have
2x 4x 3
f 0 (x) = ln(1 + x) − ln(1 − x) − + ,
1 − x2 3 − x4
2 2(1 + x 2 ) 4x 2 (x 4 + 9)
f 00 (x) = − +
1 − x 2 (1 − x 2 )2 (3 − x 4 )2
−4x 2 4x 2 (x 4 + 9) −8x 4 [x 4 + 1 + 8(1 − x 2 )]
= + = ≤ 0.
(1 − x 2 )2 (3 − x 4 )2 (1 − x 2 )2 (3 − x 4 )2
Therefore, f 0 (x) is decreasing, f 0 (x) ≤ f 0 (0) = 0, f (x) is decreasing, f (x) ≤ f (0) = 0.
The equality holds for a = b = 1.

P 1.111. If a, b are positive real numbers such that a + b = 2, then

a−a + b−b ≤ 2.

(Vasile Cîrtoaje, 2010)

Solution. Consider that a ≥ b, when

0 < b ≤ 1 ≤ a < 2,

and write the inequality as


aa − 1 b b − 1
+ ≥ 0.
aa bb
According to Lemma from the proof of P 1.92, we have
a+1 b+1
aa − 1 ≥ a 2 (a − 1), bb − 1 ≥ b 2 (b − 1).

Therefore, it suffices to show that


1−a 1−b
a 2 (a − 1) + b 2 (b − 1) ≥ 0,

which is equivalent to
1−a 1−b
a 2 ≥b 2 ,
166 Vasile Cîrtoaje

1−b
(a b) 2 ≤ 1,
a b ≤ 1.
The last inequality follows immediately from the AM-GM inequality
1
ab ≤ (a + b)2 = 1.
4
The equality holds for a = b = 1.

P 1.112. If a, b are nonnegative real numbers such that a + b = 2, then

a2b + b2a ≥ a b + b a ≥ a2 b2 + 1.

(Vasile Cîrtoaje, 2010)

Solution. Since a, b ∈ [0, 2] and (1 − a)(1 − b) ≤ 0, from Lemma below, we have

b(a b + 3 − a − b)(a − 1) b(a b + 1)(a − 1)


ab − 1 ≥ =
2 2
and
a(a b + 1)(b − 1)
ba − 1 ≥ .
2
Based on these inequalities, we get

a b + b a − a2 b2 − 1 = (a b − 1) + (b a − 1) + 1 − a2 b2
b(a b + 1)(a − 1) a(a b + 1)(b − 1)
≥ + + 1 − a2 b2
2 2
= (ab + 1)(a b − 1) + 1 − a2 b2 = 0

and

a2b + b2a − a b − b a = a b (a b − 1) + b a (b a − 1)
a b b(a b + 1)(a − 1) b a a(a b + 1)(b − 1)
≥ +
2 2
a b(a b + 1)(a − b)(a b−1 − b a−1 )
= .
4

On the valid assumption a ≥ b, we only need to show that a b−1 ≥ b a−1 , which is
equivalent to
b−a a−b a−b
a 2 ≥b 2 , 1 ≥ (a b) 2 , 1 ≥ a b, (a − b)2 ≥ 0.
Symmetric Nonrational Inequalities 167

For both inequalities, the equality holds when a = b = 1, when a = 0 and b = 2, and
when a = 2 and b = 0.
Lemma. If x, y ∈ [0, 2] such that (1 − x)(1 − y) ≤ 0, then

y(x y + 3 − x − y)(x − 1)
xy −1≥ ,
2
with equality for x = 1, for y = 0, for y = 1, and for x = 0 and y = 2.
Proof. For y = 0, y = 1 and y = 2, the inequality is an identity. For fixed y ∈ (0, 1) ∪
(1, 2), let us define

y(x y + 3 − x − y)(x − 1)
f (x) = x y − 1 − .
2
We have
x y + 3 − x − y (x − 1)( y − 1)
• ˜
0
f (x) = y x y−1
− − ,
2 2
f 00 (x) = y( y − 1)(x y−2 − 1).

Since f 00 (x) ≥ 0 for x ∈ (0, 2], f 0 is increasing. There are two cases to consider.
Case 1: x ≥ 1 > y. We have f 0 (x) ≥ f 0 (1) = 0, f (x) is increasing, hence f (x) ≥ f (1) =
0.
Case 2: y > 1 ≥ x. We have f 0 (x) ≤ f 0 (1) = 0, f (x) is decreasing, hence f (x) ≥
f (1) = 0.

P 1.113. If a, b are positive real numbers such that a + b = 2, then

a3b + b3a ≤ 2.

(Vasile Cîrtoaje, 2007)

Solution. Without loss of generality, assume that a ≥ b. Using the substitution a = 1+ x


and b = 1 − x, 0 ≤ x < 1, we can write the inequality as

e3(1−x) ln(1+x) + e3(1+x) ln(1−x) ≤ 2.

Applying Lemma below, it suffices to show that f (x) ≤ 2, where

x2 x3 x2 x3
f (x) = e3(1−x)(x− 2 + 3 ) + e−3(1+x)(x+ 2 + 3 ) .
168 Vasile Cîrtoaje

Since f (0) = 2, it suffices to show that f 0 (x) ≤ 0 for x ∈ [0, 1). From
15 2 9x 2 5x 3 4
f 0 (x) =(3 − 9x + x − 4x 3 )e3x− 2 + 2 −x
2
15 2 9x 2 5x 3 4
−(3 + 9x + x + 4x 3 )e−3x− 2 − 2 −x ,
2
it follows that f 0 (x) ≤ 0 is equivalent to
3 6 − 18x + 15x 2 − 8x 3
e−6x−5x ≥ .
6 + 18x + 15x 2 + 8x 3
For the nontrivial case 6 − 18x + 15x 2 − 8x 3 > 0, we rewrite this inequality as g(x) ≥ 0,
where

g(x) = −6x − 5x 3 − ln(6 − 18x + 15x 2 − 8x 3 ) + ln(6 + 18x + 15x 2 + 8x 3 ).

Since g(0) = 0, it suffices to show that g 0 (x) ≥ 0 for x ∈ [0, 1). From
1 0 (6 + 8x 2 ) − 10x (6 + 8x 2 ) + 10x
g (x) = −2 − 5x 2 + + ,
3 6 + 15x 2 − (18x + 8x 3 ) 6 + 15x 2 + (18x + 8x 3 )
it follows that g 0 (x) ≥ 0 is equivalent to

2(6 + 8x 2 )(6 + 15x 2 ) − 20x(18x + 8x 3 ) ≥ (2 + 5x 2 )[(6 + 15x 2 )2 − (18x + 8x 3 )2 ].

Since

(6 + 15x 2 )2 − (18x + 8x 3 )2 ≤ (6 + 15x 2 )2 − 324x 2 − 288x 4 ≤ 4(9 − 36x 2 ),

it suffices to show that

(3 + 4x 2 )(6 + 15x 2 ) − 5x(18x + 8x 3 ) ≥ (2 + 5x 2 )(9 − 36x 2 ).

This reduces to 6x 2 +200x 4 ≥ 0, which is clearly true. The equality holds for a = b = 1.

Lemma. If t > −1, then


t2 t3
ln(1 + t) ≤ t − + .
2 3
Proof. We need to prove that f (t) ≥ 0, where
t2 t3
f (t) = t − + − ln(1 + t).
2 3
Since
t3
f 0 (t) =,
t +1
f (t) is decreasing on (−1, 0] and increasing on [0, ∞). Therefore, f (t) ≥ f (0) = 0.
Symmetric Nonrational Inequalities 169

P 1.114. If a, b are nonnegative real numbers such that a + b = 2, then


‹4
a−b

a3b + b3a + ≤ 2.
2

(Vasile Cîrtoaje, 2007)

Solution (by M. Miyagi and Y. Nishizawa). We may assume that a = 1+ x and b = 1− x,


where 0 ≤ x ≤ 1. The desired inequality is equivalent to

(1 + x)3(1−x) + (1 − x)3(1+x) + x 4 ≤ 2.

By Lemma below, we have

1
(1 + x 1−x ≤ (1 + x)2 (2 − x 2 )(2 − 2x + x 2 ),
4
1
(1 − x 1+x ≤ (1 − x)2 (2 − x 2 )(2 + 2x + x 2 ).
4
Therefore, it suffices to show that

(1 + x)6 (2 − x 2 )3 (2 − 2x + x 2 )3 + (1 − x)6 (2 − x 2 )3 (2 + 2x + x 2 )3 + 64x 2 ≤ 128,

which is equivalent to

x 4 (1 − x 2 )[x 6 (x 6 − 8x 4 + 31x 2 − 34) − 2(17x 6 − 38x 4 + 16x 2 + 8)] ≤ 0.

Clearly, it suffices to show that

t 3 − 8t 2 + 31t − 34 < 0

and
17t 3 − 38t 2 + 16t + 8 > 0
for all t ∈ [0, 1]. Indeed, we have

t 3 − 8t 2 + 31t − 34 < t 3 − 8t 2 + 31t − 24 = (t − 1)(t 2 − 7t + 24) ≤ 0,

17t 3 − 38t 2 + 16t + 8 = 17t(t − 1)2 + (−4t 2 − t + 8) > 0.

Lemma. If −1 ≤ t ≤ 1, then

1
(1 + t)1−t ≤ (1 + t)2 (2 − t 2 )(2 − 2t + t 2 ),
4
with equality for t = −1, t = 0 and t = 1.
170 Vasile Cîrtoaje

Proof. We need to prove that f (t) ≥ 0 for −1 < t ≤ 1, where

f (t) = (1 + t) ln(1 + t) + ln(2 − t 2 ) + ln(2 − 2t + t 2 ) − ln 4.

We have
2t 2(t − 1)
f 0 (t) = 1 + ln(1 + t) − + ,
2 − t 2 2 − 2t + t 2
t 2 g(t)
f 00 (t) = ,
(1 + t)(2 − t 2 )2 (2 − 2t + t 2 )2
where
g(t) = t 6 − 8t 5 + 12t 4 + 8t 3 − 20t 2 − 16t + 16.
Case 1: 0 ≤ t ≤ 1. From

g 0 (t) = 6t 5 − 40t 4 + 48t 3 + 24t 2 − 40t − 16 = 6t 5 − 8t − 16 − 8t(5t 3 − 6t 2 − 3t + 4)


= (6t 5 − 8t − 16) − 8t(t − 1)2 (5t + 4) < 0,

it follows that g is strictly decreasing on [0, 1]. Since g(0) = 16 and g(1) = −7, there
exists a number c ∈ (0, 1) such that g(c) = 0, g(t) > 0 for 0 ≤ t < c and g(t) < 0
for c < t ≤ 1. Therefore, f 0 is strictly increasing on [0, c] and strictly decreasing on
[c, 1]. From f 0 (0) = 0 and f 0 (1) = ln 2 − 1 < 0, it follows that there exists a number
d ∈ (0, 1) such that f 0 (d) = 0, f 0 (t) > 0 for 0 < t < d and f 0 (t) < 0 for c < t ≤ 1. As
a consequence, f is strictly increasing on [0, d] and strictly decreasing on [d, 1]. Since
f (0) = 0 and f (1) = 0, we have f (t) ≥ 0 for 0 ≤ t ≤ 1.
Case 2: −1 < t ≤ 0. From

g(t) = t 4 (t − 2)(t − 6) + 4(t + 1)(2t 2 − 7t + 3) + 4 > 0,

it follows that f 0 is strictly increasing on (−1, 0]. Since f 0 (0) = 0, we have f 0 (t) < 0
for −1 < t < 0, hence f is strictly decreasing on (−1, 0]. From f (0) = 0, it follows that
f (t) ≥ 0 for −1 < t ≤ 0.
Conjecture. If a, b are nonnegative real numbers such that a + b = 2, then

a−b 2
 ‹
a3b + b3a + ≥ 2.
2

P 1.115. If a, b are positive real numbers such that a + b = 2, then


3 3
a a + b b ≥ 2.

(Vasile Cîrtoaje, 2008)


Symmetric Nonrational Inequalities 171

Solution. Assume that a ≤ b; that is,

0 < a ≤ 1 ≤ b < 2.
3 3
There are two cases to consider: 0 < a ≤ and ≤ a ≤ 1.
5 5
3 7
Case 1: 0 < a ≤ . From a + b = 2, we get ≤ b < 2. Let
5 5
3
f (x) = x x , 0 < x < 2.

Since
3
f 0 (x) = 3x x −2 (1 − ln x) > 0,
7 7
• ‹  ‹
f (x) is increasing on , 2 , and hence f (b) ≥ f ; that is,
5 5
 ‹15/7
3 7
bb ≥ .
5
Using Bernoulli’s inequality gives
 ‹15/7
7 7 2 8/7 7 16 51
 ‹  ‹
3
b ≥
b = 1+ > 1+ = > 2.
5 5 5 5 35 25
hence
3 3
a a + b b > 2.

3
Case 2: ≤ a ≤ 1. By Lemma below, we have
5
3
2a a ≥ 3 − 15a + 21a2 − 7a3

and
3
2b b ≥ 3 − 15b + 21b2 − 7b3 .
Summing these inequalities, we get
€ 3 3
Š
2 a a + b b ≥ 6 − 15(a + b) + 21(a2 + b2 ) − 7(a3 + b3 )
= 6 − 15(a + b) + 21(a + b)2 − 7(a + b)3 = 4.

This completes the proof. The equality holds for a = b = 1.


3
Lemma. If ≤ x ≤ 2, then
5
3
2x x ≥ 3 − 15x + 21x 2 − 7x 3 ,
172 Vasile Cîrtoaje

with equality for x = 1.


Proof. We first show that h(x) > 0, where

h(x) = 3 − 15x + 21x 2 − 7x 3 .

From
h0 (x) = 3(−5 + 14x − 7x 2 ),
 s s   s 
2 2 2
it follows that h(x) is increasing on 1 − ,1 + , and decreasing on 1 + ,∞ .
7 7 7
3
 ‹
Then, it suffices to show that f ≥ 0 and f (2) ≥ 0. Indeed
5
3 6
 ‹
f = , f (2) = 1.
5 125
Write now the desired inequality as f (x) ≥ 0, where
3 3
f (x) = ln 2 + ln x − ln(3 − 15x + 21x 2 − 7x 3 ), ≤ x ≤ 2.
x 5
We have
x2 0 x 2 (7x 2 − 14x + 5)
f (x) = g(x), g(x) = 1 − ln x + ,
3 3 − 15x + 21x 2 − 7x 3
g1 (x)
g 0 (x) = ,
(3 − 15x + 21x 2 − 7x 3 )2
where

g1 (x) = −49x 7 + 245x 6 − 280x 5 − 147x 4 + 471x 3 − 321x 2 + 90x − 9.

In addition,

g1 (x = (x − 1)2 g2 (x), g2 (x) = −49x 5 + 147x 4 + 63x 3 − 168x 2 + 72x − 9,

g2 (x) = 11x 5 + 3g3 (x), g3 (x) = −20x 5 + 49x 4 + 21x 3 − 56x 2 + 24x − 3,
g3 (x) = (4x − 1)g4 (x), g4 (x) = −5x 4 + 11x 3 + 8x 2 − 12x + 3,
g4 (x) = x 5 + g5 (x), g5 (x) = −6x 4 + 11x 3 + 8x 2 − 12x + 3,
g5 (x) = (2x − 1)g6 (x), g6 (x) = −3x 3 + 4x 2 + 6x − 3,
g6 (x) = 1 + (2 − x)(3x 2 + 2x − 2).
Therefore, we get in succession g6 (x) > 0, g5 (x) > 0, g4 (x) > 0, g3 (x) > 0, g2 (x)
• > 0,
3
‹
0
g1 (x) ≥ 0, g (x) ≥ 0, g(x) is increasing. Since g(1) = 0, we have g(x) < 0 on ,1
5
3
• ˜
and g(x) > 0 on (1, 2]. Then, f (x) is decreasing on , 1 and increasing on [1, 2],
5
hence f (x) ≥ f (1) = 0.
Symmetric Nonrational Inequalities 173

P 1.116. If a, b are positive real numbers such that a + b = 2, then


2 2
a5b + b5a ≤ 2.

(Vasile Cîrtoaje, 2010)

Solution. Assume that a ≥ b. For a = 2 and b = 0, the inequality is obvious. Otherwise,


using the substitution a = 1 + x and b = 1 − x, 0 ≤ x < 1, we can write the desired
inequality as
2 2
e5(1−x) ln(1+x) + e5(1+x) ln(1−x) ≤ 2.
According to Lemma below, it suffices to show that f (x) ≤ 2, where

f (x) = e5(u−v) + e−5(u+v) ,


7 3 31 5 5 17 4 9 6
u= x+ x + x , v = x2 + x + x .
3 30 2 12 20
If f 0 (x) ≤ 0, then f (x) is decreasing, hence f (x) ≤ f (0) = 2. Since

f 0 (x) = 5(u0 − v 0 )e5(u−v) − 5(u0 + v 0 )e−5(u+v) ,


31 4 17 3 27 5
u0 = 1 + 7x 2 + x , v 0 = 5x + x + x ,
6 3 10
the inequality f 0 (x) ≤ 0 becomes

e−10u (u0 + v 0 ) ≥ u0 − v 0

For the nontrivial case u0 − v 0 > 0, we rewrite this inequality as g(x) ≥ 0, where

g(x) = −10u + ln(u0 + v 0 ) − ln(u0 − v 0 ).

If g 0 (x) ≥ 0, then g(x) is increasing, hence g(x) ≥ f (0) = 2. We have

u00 + v 00 u00 − v 00
g 0 (x) = −10u0 + − 0 ,
u0 + v 0 u − v0
where
62 3 27 4
u00 = 14x + x , v 00 = 5 + 17x 2 + x .
3 2
Thus, the inequality g 0 (x) ≥ 0 is equivalent to

u0 v 00 − v 0 u00 ≥ 5u0 (u0 + v 0 )(u0 − v 0 ),

a1 t + a2 t 2 + a3 t 3 + a4 t 4 + a5 t 5 + a6 t 6 + a7 t 7 ≥ 0,
where t = x 2 , 0 ≤ t < 1, and

a1 = 2, a2 = 321.5, a3 ≈ 152.1, a4 ≈ −498.2,


174 Vasile Cîrtoaje

a5 ≈ −168.5, a6 ≈ 356.0, a7 ≈ 188.3.


This inequality is true if

300t 2 + 150t 3 − 500t 4 − 200t 5 + 250t 6 ≥ 0.

Since the last inequality is equivalent to the obvious inequality

50t 2 (1 − t)(6 + 9t − t 2 − 5t 3 ) ≥ 0,

the proof is completed. The equality holds for a = b = 1.


Lemma. If −1 < t < 1, then
5 2 7 3 17 4 31 5 9 6
(1 − t)2 ln(1 + t) ≤ t − t + t − t + t − t .
2 3 12 30 20
Proof. We shall show that
1 1 1 1
 ‹
(1 − t)2 ln(1 + t) ≤ (1 − t)2 t − t 2 + t 3 − t 4 + t 5
2 3 4 5
5 2 7 3 17 4 31 5 9 6
≤t− t + t − t + t − t .
2 3 12 30 20
The left inequality is equivalent to f (t) ≥ 0, where
1 2 1 3 1 4 1 5
f (t) = t − t + t − t + t − ln(1 + t).
2 3 4 5
Since
t5
f 0 (t) = ,
1+ t
f (t) is decreasing on (−1, 0] and increasing on [0, 1); therefore, f (t) ≥ f (0) = 0.
The right inequality is equivalent to t 6 (t − 1) ≤ 0, which is clearly true.

P 1.117. If a, b are positive real numbers such that a + b = 2, then


p p
a2 b
+ b2 a
≤ 2.

(Vasile Cîrtoaje, 2010)

Solution. Assume that a ≥ b. For a = 2 and b = 0, the inequality is obvious. Otherwise,


using the substitution a = 1 + x and b = 1 − x, 0 ≤ x < 1, we can write the desired
inequality as f (x) ≤ 2, where
p p
f (x) = e21−x ln(1+x)
+ e2 1+x ln(1−x)
.
Symmetric Nonrational Inequalities 175

There are two cases to consider.


13
Case 1: ≤ x < 1. If f 0 (x) ≤ 0, then f (x) is decreasing, and hence
20
q q
7 33  ‹5/4  ‹2
13 33 7 5 1
 ‹  ‹ 5
 ‹ 5
f (x) ≤ f = + < + < 2.
20 20 20 3 4
Since
 p
2 1 − x ln(1 + x) 2p1−x ln(1+x)

0
f (x) = − p e
1+ x 1− x
 p
2 1 + x ln(1 − x) 2p1+x ln(1−x)

− − p e
1− x 1+ x
 p
2 1 − x ln(1 + x) 2p1−x ln(1+x)

< − p e ,
1+ x 1− x
it is enough to show that g(x) ≤ 0,where
2(1 − x)
g(x) = − ln(1 + x).
1+ x
Clearly, g is decreasing, and hence
13 14 33
‹ 
g(x) ≤ g = − ln < 0.
20 33 20

13
Case 2: 0 ≤ x ≤ . According to Lemma below, it suffices to show that f (x) ≤ 2,
20
where
11 3 1 4
2
+ 12 x −2 x 2
+ 11 3 1 4
12 x + 2 x )
f (x) = e2x−2x + e−(2x+2x .
If f 0 (x) ≤ 0, then f (x) is decreasing, hence f (x) ≤ f (0) = 2. Since
11 2 2 11 3 1 4
f 0 (x) = (2 − 4x + x − 2x 3 )e2x−2x + 12 x − 2 x
4
11 2 2 11 3 1 4
−(2 + 4x + x + 2x 3 )e−(2x+2x + 12 x + 2 x ) ,
4
0
the inequality f (x) ≤ 0 is equivalent to
11
x3 8 − 16x + 11x 2 − 8x 3
e−4x− 6 ≥ .
8 + 16x + 11x 2 + 8x 3
For the non-trivial case 8 − 16x + 11x 2 − 8x 3 > 0, rewrite this inequality as g(x) ≥ 0,
where
11 3
g(x) = −4x − x − ln(8 − 16x + 11x 2 − 8x 3 ) + ln(8 + 16x + 11x 2 + 8x 3 ).
6
176 Vasile Cîrtoaje

If g 0 (x) ≥ 0, then g(x) is increasing, hence g(x) ≥ g(0) = 0. From

11 2 (16 + 24x 2 ) − 22x (16 + 24x 2 ) + 22x


g 0 (x) = −4 − x + + ,
2 8 + 11x 2 − (16x + 8x 3 ) 8 + 11x 2 + (16x + 8x 3 )

it follows that g 0 (x) ≥ 0 is equivalent to

1
(16 + 24x 2 )(8 + 11x 2 ) − 22x(16x + 8x 3 ) ≥ (8 + 11x 2 )[(8 + 11x 2 )2 − (16x + 8x 3 )2 ].
4
Since

(8 + 11x 2 )2 − (16x + 8x 3 )2 ≤ (8 + 11x 2 )2 − 256x 2 − 256x 4 ≤ 16(4 − 5x 2 ),

it suffices to show that

(4 + 6x 2 )(8 + 11x 2 ) − 11x(8x + 4x 3 ) ≥ (8 + 11x 2 )(4 − 5x 2 ).

This reduces to 77x 4 ≥ 0. The proof is completed. The equality holds for a = b = 1.
13
Lemma. If −1 < t ≤ , then
20
p 11 3 1 4
1 − t ln(1 + t) ≤ t − t 2 + t − t .
24 4
Proof. We consider two cases.
13
Case 1: 0 ≤ t ≤ . We can prove the desired inequality by multiplying the following
20
inequalities
p 1 1 1 3
1 − t ≤ 1 − t − t2 − t ,
2 8 16
1 1 1 1
ln(1 + t) ≤ t − t 2 + t 3 − t 4 + t 5 ,
2 3 4 5
1 1 1 3 1 1 1 1 11 3 1 4
 ‹ ‹
1 − t − t2 − t t − t2 + t3 − t4 + t5 ≤ t − t2 + t − t .
2 8 16 2 3 4 5 24 4

The first inequality is equivalent to f (t) ≥ 0, where

1 1 2 1 3 1
 ‹
f (t) = ln 1 − t − t − t − ln(1 − t).
2 8 16 2

Since

1 8 + 4t + 3t 2 5t 3
f 0 (t) = − = ≥ 0,
2(1 − t) 16 − 8t − 2t 2 − t 3 2(1 − t)(16 − 8t − 2t 2 − t 3 )
Symmetric Nonrational Inequalities 177

f (t) is increasing, and hence f (t) ≥ f (0) = 0.


The second inequality is equivalent to f (t) ≥ 0, where
1 2 1 3 1 4 1 5
f (t) = t − t + t − t + t − ln(1 + t).
2 3 4 5
Since
1 t5
f 0 (t) = 1 − t + t 2 − t 3 + t 4 − = ≥ 0,
1+ t 1+ t
f (t) is increasing, and hence f (t) ≥ f (0) = 0.
The third inequality is equivalent to

t 4 (160 − 302t + 86t 2 + 9t 3 + 12t 4 ) ≥ 0.

This is true since

160 − 302t + 86t 2 + 9t 3 + 12t 4 ≥ 2(80 − 151t + 43t 2 ) > 0.

Case 2: −1 < t ≤ 0. Write the desired inequality as


p 11 3 1 4
− 1 − t ln(1 + t) ≥ −t + t 2 − t + t .
24 4
This is true if
p 1 1
1 − t ≥ 1 − t − t 2,
2 8
1 1
− ln(1 + t) ≥ −t + t 2 − t 3 + t 4 ,
3 4
1 1 1 1 11 3 1 4
 ‹ ‹
1 − t − t 2 −t + t 2 − t 3 + t 4 ≥ −t + t 2 − t + t .
2 8 3 4 24 4

The first inequality is equivalent to f (t) ≥ 0, where

1 1 1 2
 ‹
f (t) = ln(1 − t) − ln 1 − t − t .
2 2 8
Since
−1 2(2 + t) −3t 2
f 0 (t) = + = ≤ 0,
2(1 − t) 8 − 4t − t 2 2(1 − t)(8 − 4t − t 2 )
f (t) is decreasing, and hence f (t) ≥ f (0) = 0.
The second inequality is equivalent to f (t) ≥ 0, where
1 2 1 3 1 4
f (t) = t − t + t − t − ln(1 + t).
2 3 4
178 Vasile Cîrtoaje

Since
1 −t 4
f 0 (t) = 1 − t + t 2 − t 3 − = ≤ 0,
1+ t 1+ t
f (t) is decreasing, and hence f (t) ≥ f (0) = 0.
The third inequality reduces to the obvious inequality

t 4 (10 − 8t − 3t 2 ) ≥ 0.

P 1.118. If a, b are nonnegative real numbers such that a + b = 2, then

a b(1 − a b)2 a b(1 − a b)2


≤ a b+1 + b a+1 − 2 ≤ .
2 3
(Vasile Cîrtoaje, 2010)

Solution. Assume that a ≥ b, which yields 1 ≤ a ≤ 2 and 0 ≤ b ≤ 1.


(a) To prove the left inequality we apply Lemma 1 below. For x = a and k = b, we
have
b(1 + b) b(1 + b)(1 − b)
a b+1 ≥ 1 + (1 + b)(a − 1) + (a − 1)2 − (a − 1)3 ,
2 6
b(1 + b) b(1 + b)
a b+1 ≥ a − b + a b + (a − 1)2 − (a − 1)4 . (*)
2 6
Also, for x = b and k = a − 1, we have

a(a − 1) a(a − 1)(2 − a)


b a ≥ 1 + a(b − 1) + (b − 1)2 − (b − 1)3 ,
2 6
a ab
b a ≥ 1 − a + a b + (a − 1)3 + (a − 1)4 ,
2 6
ab a b2
b a+1 ≥ b − a b + a b2 + (a − 1)3 + (a − 1)4 . (**)
2 6
Summing up (*) and (**) gives

b(3 − a b) b(1 + b − a b)
a b+1 + b a+1 − 2 ≥ −b(a − 1)2 + (a − 1)2 − (a − 1)4 .
2 6
Since
b(3 − a b) b
−b(a − 1)2 + (a − 1)2 = (a − 1)4 ,
2 2
Symmetric Nonrational Inequalities 179

we have
b b(1 + b − a b)
a b+1 + b a+1 − 2 ≥ (a − 1)4 − (a − 1)4
2 6
a b(1 + b) ab a b(1 − a b)2
= (a − 1)4 ≥ (a − 1)4 = .
6 6 6
The equality holds for a = b = 1, for a = 2 and b = 0, and for a = 0 and b = 2.
(b) To prove the right inequality we apply Lemma 2 below. For x = a and k = b,
we have
(b + 1)b (b + 1)b(b − 1)
a b+1 ≤ 1 + (b + 1)(a − 1) + (a − 1)2 + (a − 1)3
2 6

(b + 1)b(b − 1)(b − 2)
+ (a − 1)4 ,
24
b(b + 1) b(b + 1) a b(b + 1)
a b+1 ≤ 1 + (b + 1)(a − 1) + (a − 1)2 − (a − 1)4 + (a − 1)5 .
2 6 24
Also, for x = b and k = a, we have

a(a + 1) a(a + 1) a b(a + 1)


b a+1 ≤ 1 + (a + 1)(b − 1) + (b − 1)2 − (b − 1)4 + (b − 1)5 .
2 6 24
Summing these inequalities gives

a2 + b2 + 2 a2 + b2 + 2 ab
a b+1 + b a+1 − 2 ≤ −2(a − 1)2 + (a − 1)2 − (a − 1)4 − (a − 1)6
2 6 12

a2 + b2 − 2 a2 + b2 + 2 a2 + b2 + 2
≤ (a − 1)2 − (a − 1)4 = (a − 1)4 − (a − 1)4
2 6 6
ab a b(1 − a b)2
= (a − 1)4 = .
3 3
The equality holds for a = b = 1, for a = 2 and b = 0, and for a = 0 and b = 2.
Lemma 1. If x ≥ 0 and 0 ≤ k ≤ 1, then

k(1 + k) k(1 + k)(1 − k)


x k+1 ≥ 1 + (1 + k)(x − 1) + (x − 1)2 − (x − 1)3 ,
2 6
with equality for x = 1, for k = 0, for k = 1.
Proof. For k = 0 and k = 1, the inequality is an identity. For fixed k, 0 < k < 1, let us
define
k(1 + k) k(1 + k)(1 − k)
f (x) = x k+1 − 1 − (1 + k)(x − 1) − (x − 1)2 + (x − 1)3 .
2 6
180 Vasile Cîrtoaje

We need to show that f (x) ≥ 0. We have

1 k(1 − k)
f 0 (x) = x k − 1 − k(x − 1) + (x − 1)2 ,
1+k 2
1
f 00 (x) = x k−1 − 1 + (1 − k)(x − 1),
k(1 + k)
1
f 000 (x) = −x k−2 + 1.
k(1 + k)(1 − k)

Case 1: 0 ≤ x ≤ 1. Since f 000 ≤ 0, f 00 is decreasing, f 00 (x) ≥ f 00 (1) = 0, f 0 is increasing,


f 0 (x) ≤ f 0 (1) = 0, f is decreasing, f (x) ≥ f (1) = 0.
Case 2: x ≥ 1. Since f 000 ≥ 0, f 00 is increasing, f 00 (x) ≥ f 00 (1) = 0, f 0 is increasing,
f 0 (x) ≥ f 0 (1) = 0, f is increasing, f (x) ≥ f (1) = 0.
Lemma 2. If either x ≥ 1 and 0 ≤ k ≤ 1, or 0 ≤ x ≤ 1 and 1 ≤ k ≤ 2, then

(k + 1)k (k + 1)k(k − 1)
x k+1 ≤ 1 + (k + 1)(x − 1) + (x − 1)2 + (x − 1)3
2 6
(k + 1)k(k − 1)(k − 2)
+ (x − 1)4 ,
24
with equality for x = 1, for k = 0, for k = 1, for k = 2.
Proof. For k = 0, k = 1 and k = 2, the inequality is an identity. For fixed k, k ∈
(0, 1) ∪ (1, 2), let us define

(k + 1)k (k + 1)k(k − 1)
f (x) = x k+1 − 1 − (k + 1)(x − 1) − (x − 1)2 − (x − 1)3
2 6
(k + 1)k(k − 1)(k − 2)
− (x − 1)4 .
24
We need to show that f (x) ≤ 0. We have

1 k(k − 1) k(k − 1)(k − 2)


f 0 (x) = x k − 1 − k(x − 1) − (x − 1)2 − (x − 1)3 ,
k+1 2 6
1 (k − 1)(k − 2)
f 00 (x) = x k−1 − 1 − (k − 1)(x − 1) − (x − 1)2 ,
k(k + 1) 2
1
f 000 (x) = x k−2 − 1 − (k − 2)(x − 1),
k(k + 1)(k − 1)
1
f (4) (x) = x k−3 − 1.
k(k + 1)(k − 1)(k − 2)
Symmetric Nonrational Inequalities 181

Case 1: x ≥ 1, 0 < k < 1. Since f (4) (x) ≤ 0, f 000 (x) is decreasing, f 000 (x) ≤ f 000 (1) =
0, f 00 is decreasing, f 00 (x) ≤ f 00 (1) = 0, f 0 is decreasing, f 0 (x) ≤ f 0 (1) = 0, f is
decreasing, f (x) ≤ f (1) = 0.
Case 2: 0 ≤ x ≤ 1, 1 < k < 2. Since f (4) ≤ 0, f 000 is decreasing, f 000 (x) ≥ f 000 (1) = 0, f 00
is increasing, f 00 (x) ≤ f 00 (1) = 0, f 0 is decreasing, f 0 (x) ≥ f 0 (1) = 0, f is increasing,
f (x) ≤ f (1) = 0.

P 1.119. If a, b are nonnegative real numbers such that a + b = 1, then

a2b + b2a ≤ 1.

(Vasile Cîrtoaje, 2007)

Solution. Without loss of generality, assume that

1
0≤ b≤ ≤ a ≤ 1.
2
Applying Lemma 1 below for c = 2b, 0 ≤ c ≤ 1, we get

a2b ≤ (1 − 2b)2 + 4a b(1 − b) − 2a b(1 − 2b) ln a,

which is equivalent to
a2b ≤ 1 − 4a b2 − 2a b(a − b) ln a.
Similarly, applying Lemma 2 below for d = 2a − 1, d ≥ 0, we get

b2a−1 ≤ 4a(1 − a) + 2a(2a − 1) ln(2a + b − 1),

which is equivalent to
b2a ≤ 4a b2 + 2a b(a − b) ln a.
Adding up these inequalities, the desired inequality follows. The equality holds for
a = b = 1/2, for a = 0 and b = 1, and for a = 1 and b = 0.
Lemma 1. If 0 < a ≤ 1 and c ≥ 0, then

a c ≤ (1 − c)2 + ac(2 − c) − ac(1 − c) ln a,

with equality for a = 1, for c = 0, and for c = 1.


Proof. Making the substitution a = e−x , x ≥ 0, we need to prove that f (x) ≥ 0, where

f (x) = (1 − c)2 e x + c(2 − c) + c(1 − c)x − e(1−c)x ,


182 Vasile Cîrtoaje

f 0 (x) = (1 − c)[(1 − c)e x + c − e(1−c)x ].


If f 0 ≥ 0 on [0, ∞), then f is increasing, and hence f (x) ≥ f (0) = 0. In order to prove
that f 0 ≥ 0, we consider two cases.
Case 1: 0 ≤ c ≤ 1. By the weighted AM-GM inequality, we have

(1 − c)e x + c ≥ e(1−c)x ,

and hence f 0 (x) ≥ 0.


Case 2: c ≥ 1. By the weighted AM-GM inequality, we have

(c − 1)e x + e(1−c)x ≥ c,

which yields
f 0 (x) = (c − 1)[(c − 1)e x + e(1−c)x − c] ≥ 0.

Lemma 2. If 0 ≤ b ≤ 1 and d ≥ 0, then

b d ≤ 1 − d 2 + d(1 + d) ln(b + d),

with equality for d = 0, and for b = 0 and d = 1.


Proof. Write the inequality as

(1 + d)[1 − d + d ln(b + d)] ≥ b d .

Excepting the equality cases, since

1 − d + d ln(b + d) ≥ 1 − d + d ln d > 0,

we can rewrite the inequality in the form

ln(1 + d) + ln[1 − d + d ln(b + d)] ≥ d ln b.

Using the substitution b = e−x − d, where − ln(1 + d) ≤ x ≤ − ln d, we need to prove


that f (x) ≥ 0, where

f (x) = ln(1 + d) + ln(1 − d − d x) + d x − d ln(1 − d e x ).

Since
d 2 (e x − 1 − x)
f 0 (x) = ≥ 0,
(1 − d − d x)(1 − d e x )
f is increasing, and hence

f (x) ≥ f (− ln(1 + d)) = ln[1 − d 2 + d(1 + d) ln(1 + d)].


Symmetric Nonrational Inequalities 183

To complete the proof, we only need to show that −d 2 + d(1 + d) ln(1 + d) ≥ 0; that is,

(1 + d) ln(1 + d) ≥ d.

−d
This inequality follows from e x ≥ 1 + x, where x = .
1+d
Conjecture. If a, b are nonnegative real numbers such that 1 ≤ a + b ≤ 15, then

a2b + b2a ≤ a a+b + b a+b .

P 1.120. If a, b are positive real numbers such that a + b = 1, then

2a a b b ≥ a2b + b2a .

Solution. Taking into account the inequality a2b + b2a ≤ 1 in the preceding P 1.119, it
suffices to show that
2a a b b ≥ 1.

Write this inequality as


2a a b b ≥ a a+b + b a+b ,
 a  b  b ‹a
2≥ + .
b a
Since a < 1 and b < 1, we apply Bernoulli’s inequality as follows
 a b  ‹a  ‹
b a  b
+ ≤1+ b −1 +1+a − 1 = 2.
b a b a

Thus, the proof is completed. The equality holds for a = b = 1/2.

P 1.121. If a, b are positive real numbers such that a + b = 1, then

a−2a + b−2b ≤ 4.
184 Vasile Cîrtoaje

Solution. Applying Lemma below, we have


a−2a ≤ 4 − 2 ln 2 − 4(1 − ln 2)a,
b−2b ≤ 4 − 2 ln 2 − 4(1 − ln 2)b.
Adding these inequalities, the desired inequality follows. The equality holds for a = b =
1/2.
Lemma. If x ∈ (0, 1], then
x −2x ≤ 4 − 2 ln 2 − 4(1 − ln 2)x,
with equality for x = 1/2.
Proof. Write the inequality as
1 −2x 1
x ≤ 1 − c − (1 − 2c)x, c= ln 2 ≈ 0.346.
4 2
This is true if f (x) ≤ 0, where
f (x) = −2 ln 2 − 2x ln x − ln[1 − c − (1 − 2c)x].
We have
1 − 2c
f 0 (x) = −2 − 2 ln x + ,
1 − c − (1 − 2c)x
2 (1 − 2c)2 g(x)
f 00 (x) = − + = ,
x [1 − c − (1 − 2c)x] 2 x[1 − c − (1 − 2c)x]2
where
g(x) = 2(1 − 2c)2 x 2 − (1 − 2c)(5 − 6c)x + 2(1 − c)2 .
Since
g 0 (x) = (1 − 2c)[4(1 − 2c)x − 5 + 6c] ≤ (1 − 2c)[4(1 − 2c) − 5 + 6c]
= (1 − 2c)(−1 − 2c) < 0,
g is decreasing on (0, 1], hence g(x) ≥ g(1) = −2c 2 + 4c − 1 > 0, f 00 (x) > 0 for
x ∈ (0, 1]. Since f 0 is increasing and f 0 (1/2) = 0, we have f 0 (x) ≤ 0 for x ∈ (0, 1/2],
and f 0 (x) ≥ 0 for x ∈ [1/2, 1]. Therefore, f is decreasing on (0, 1/2] and increasing on
[1/2, 1], hence f (x) ≥ f (1/2) = 0.

Remark. According to the inequalities in P 1.119 and P 1.121, the following inequality
holds for all positive numbers a, b such that a + b = 1:
 1 1
‹
a2b + b2a + ≤ 4.
a2a b2b
Actually, this inequality holds for all a, b ∈ (0, 1]. In this case, it is sharper than the
inequality in P 1.103.
Symmetric Nonrational Inequalities 185

P 1.122. Let a, b, c, d be positive real numbers such that a2 + b2 + c 2 + d 2 = 1. Prove that


p p p p p p p p
1 − a + 1 − b + 1 − c + 1 − d ≥ a + b + c + d.

(Vasile Cîrtoaje, 2007)

First Solution. We can obtain the desired inequality by summing the inequalities
p p p p
1 − a + 1 − b ≥ c + d,
p p p p
1 − c + 1 − d ≥ a + b.
Since p p Æ
4
1 − a + 1 − b ≥ 2 (1 − a)(1 − b)
and v v
p p tc + d 4 c +d
t 2 2
c+ d ≤2 ≤2 ,
2 2
the former inequality above holds if

c2 + d 2
(1 − a)(1 − b) ≥ .
2
Indeed,

2(1 − a)(1 − b) − c 2 − d 2 = 2(1 − a)(1 − b) + a2 + b2 − 1 = (a + b − 1)2 ≥ 0.


1
Similarly, we can prove the second inequality. The equality holds for a = b = c = d = .
2

Second Solution. We can obtain the desired inequality by summing the inequalities
p p 1 p p 1
1 − a − a ≥ p (1 − 4a2 ), 1− b− b ≥ p (1 − 4b2 ),
2 2 2 2
p p 1 p p 1
1 − c − c ≥ p (1 − 4c 2 ), 1−d − d ≥ p (1 − 4d 2 ).
2 2 2 2
To prove the first inequality, we write it as
1 − 2a 1
p p ≥ p (1 − 2a)(1 + 2a).
1−a+ a 2 2

1
Case 1: 0 < a ≤ . We need to show that
2
p p p
2 2 ≥ (1 + 2a)( 1 − a + a).
186 Vasile Cîrtoaje

p p p p
Since 1 − a + a ≤ 2[(1 − a) + a] = 2, we have
p p p p
2 2 − (1 + 2a)( 1 − a + a) ≥ 2(1 − 2a) ≥ 0.

1
Case 2: ≤ a < 1. We need to show that
2
p p p
2 2 ≤ (1 + 2a)( 1 − a + a).
p
Since 1 + 2a ≥ 2 2a, it suffices to prove that
Æ
1 ≤ a(1 − a) + a.

Indeed,
p
Æ p p p 1 − a (1 − 2a)
1−a− a(1 − a) = 1 − a ( 1 − a − a) = p p ≤ 0.
1−a+ a

P 1.123. Let a, b, c, d be positive real numbers. Prove that


p
A + 2 ≥ B + 4,

where
1 1 1 1
 ‹
A = (a + b + c + d)+ + + − 16,
a b c d
1 1 1 1
 ‹
2 2 2 2
B = (a + b + c + d ) 2 + 2 + 2 + 2 − 16.
a b c d
(Vasile Cîrtoaje, 2004)
Solution. By squaring, the inequality becomes

A2 + 4A ≥ B.

Let us denote
x y z x2 y 2 z2
f (x, y, z) = + + − 3, F (x, y, z) = + + 2 − 3,
y z x y 2 z2 x
where x, y, z > 0. By the AM-GM inequality, it follows that f (x, y, z) ≥ 0 and F (x, y, z) ≥
0. We can check that

A = f (a, b, c) + f (b, a, d) + f (c, d, a) + f (d, c, b)


= f (c, b, a) + f (d, a, b) + f (a, d, c) + f (b, c, d)
Symmetric Nonrational Inequalities 187

and
B = F (a, b, c) + F (b, a, d) + F (c, d, a) + F (d, c, b).
Since

F (x, y, z) = [ f (x, y, z) + 3]2 − 2[ f (z, y, x) + 3] − 3


= f 2 (x, y, z) + 6 f (x, y, z) − 2 f (z, y, x),

we get
B = f 2 (a, b, c) + f 2 (b, a, d) + f 2 (c, d, a) + f 2 (d, c, b) + 4A.
Therefore,

A2 + 4A − B = [ f (a, b, c) + f (b, a, d) + f (c, d, a) + f (d, c, b)]2


− f 2 (a, b, c) − f 2 (b, a, d) − f 2 (c, d, a) − f 2 (d, c, b) ≥ 0.

The equality holds for a = b = c = d.

P 1.124. Let a1 , a2 , . . . , an be nonnegative real numbers such that a1 + a2 + · · · + an = 1.


Prove that
3a1 + 1 + 3a2 + 1 + · · · + 3an + 1 ≥ n + 1.
p p p

First Solution. Without loss of generality, assume that a1 = max{a1 , a2 , · · · , an }. Write


the inequality as follows:

( 3a1 + 1 − 2) + ( 3a2 + 1 − 1) + · · · + ( 3an + 1 − 1) ≥ 0,


p p p

a1 − 1 a2 an
+p + ··· + p ≥ 0,
3a1 + 1 + 2 3a2 + 1 + 1 3an + 1 + 1
p

a2 an a2 + · · · + an
+ ··· + p ≥p ,
3a2 + 1 + 1 3an + 1 + 1 3a1 + 1 + 2
p
   
1 1 1 1
a2 p −p + · · · + an p −p ≥ 0.
3a2 + 1 + 1 3a1 + 1 + 2 3an + 1 + 1 3a1 + 1 + 2
The last inequality is clearly true. The equality holds for a1 = 1 and a2 = · · · = an = 0
(or any cyclic permutation).
Second Solution. We use the induction method. For n = 1, the inequality is an equality.
We claim that Æ
3a1 + 1 + 3an + 1 ≥ 3(a1 + an ) + 1 + 1.
p p
188 Vasile Cîrtoaje

By squaring, this becomes


Æ Æ
(3a1 + 1)(an + 1) ≥ 3(a1 + an ) + 1,

which is equivalent to a1 an ≥ 0. Thus, to prove the original inequality, it suffices to


show that Æ
3(a1 + an ) + 1 + 3a2 + 1 + · · · + 3an−1 + 1 ≥ n.
p p

Using the substitution b1 = a1 + an and b2 = a2 , · · · , bn−1 = an−1 , this inequality be-


comes Æ Æ Æ
3b1 + 1 + 3b2 + 1 + · · · + 3bn−1 + 1 ≥ n
for b1 + b2 + · · · + bn−1 = 1. Clearly, this is true by the induction hypothesis.

P 1.125. Let 0 ≤ a < b and a1 , a2 , . . . , an ∈ [a, b]. Prove that


€p p Š2
a1 + a2 · · · + an − n n a1 a2 · · · an ≤ (n − 1)
p
b− a .

(Vasile Cîrtoaje, 2005)


Solution. Based on Lemma below, it suffices to consider that

a1 = · · · = ak = a, ak+1 = · · · = an = b,

where k ∈ {1, 2, · · · , n − 1}. The original inequality becomes


k n−k p
(n − k − 1)a + (k − 1)b + na n b n ≥ (2n − 2) a b.

Clearly, this inequality follows by the weighted AM-GM inequality. For n ≥ 3, the equal-
ity holds when a = 0, one of ai is also 0 and the other ai are equal to b.
Lemma. Let 0 ≤ a < b and a1 , a2 , . . . , an ∈ [a, b]. Then, the expression

a1 + a2 · · · + an − n n a1 a2 · · · an
p

is maximal when a1 , a2 , . . . , an ∈ {a, b}.


Proof. We use the contradiction method. Consider that a2 , . . . , an are fixed and define
the function
f (a1 ) = a1 + a2 + · · · + an − n n a1 a2 · · · an .
p

For the sake of contradiction, assume that there exits a1 ∈ (a, b) such that
p f (a1 ) > f (a)
p p n
and f (a1 ) > f (b). Let us denote x i = n ai for all i, c = n a and d = b (c < x 1 < d).
From

f (a1 ) − f (a) = x 1n − c n − n(x 1 − c)x 2 · · · x n


= (x 1 − c)(x 1n−1 + x 1n−2 c + · · · + c n−1 − nx 2 · · · x n ),
Symmetric Nonrational Inequalities 189

we get

x 1n−1 + x 1n−2 c + · · · + c n−1 > nx 2 · · · x n . (*)

Analogously, from

f (a1 ) − f (b) = x 1n − d n − n(x 1 − d)x 2 · · · x n


= (x 1 − d)(x 1n−1 + x 1n−2 d + · · · + d n−1 − nx 2 · · · x n ),

we get

nx 2 · · · x n > x 1n−1 + x 1n−2 d + · · · + d n−1 . (**)

Summing up (*) and (**) yields

x 1n−1 + x 1n−2 c + · · · + c n−1 > x 1n−1 + x 1n−2 d + · · · + d n−1 ,

which is clearly false.

P 1.126. Let a1 , a2 , . . . , an be positive real numbers such that a1 a2 · · · an = 1. Prove that

1 1 1
+p + ··· + p ≥ 1.
1 + (n2 1 + (n2 − 1)a2 1 + (n2 − 1)an
p
− 1)a1

Solution. For the sake of contradiction, assume that

1 1 1
+p + ··· + p < 1.
1 + (n2 1 + (n2 1 + (n2 − 1)an
p
− 1)a1 − 1)a2

It suffices to show that a1 a2 · · · an > 1. Let

1
xi = p , 0 < x i < 1, i = 1, 2, · · · , n.
1 + (n2 − 1)ai

1 − x i2
Since ai = for all i, we need to show that
(n2 − 1)x i2

x1 + x2 + · · · + x n < 1

implies
(1 − x 12 )(1 − x 22 ) · · · (1 − x n2 ) > (n2 − 1)n x 12 x 22 · · · x n2 .
190 Vasile Cîrtoaje

Using the AM-GM inequality gives


Y Y h€X Š2 i Y
(1 − x 12 ) > x 1 − x 12 = (x 2 + · · · + x n )(2x 1 + x 2 + · · · + x n )
Y€ p q Š
≥ (n2 − 1)n x 12 x 2 · · · x n = (n2 − 1)n x 12 x 22 · · · x n2 .
n+1
n−1
x2 · · · x n ·

The equality holds for a1 = a2 = · · · = an = 1.

P 1.127. Let a1 , a2 , . . . , an be positive real numbers such that a1 a2 · · · an = 1. Prove that


n
X 1 1
≥ .
1+ 1 + 4n(n − 1)ai
p
i=1
2

First Solution. Write the inequality as follows


n p
X 1 + 4n(n − 1)ai − 1
≥ 2n(n − 1),
i=1
a1
v
n u
X
t1 4n(n − 1) X 1
+ ≥ 2n(n − 1) + .
i=1
a12 a1 a1

By squaring, the inequality becomes


v
X u 1
u – ™
4n(n − 1) 1 4n(n − 1) 2 2
X 1
t
2
+ 2
+ ≥ 2n (n − 1) + .
1≤i< j≤n
a i
a i a j
a j 1≤i< j≤n
a i a j

The Cauchy-Schwarz inequality gives


v
u – ™
u 1 4n(n − 1) 1 4n(n − 1) 1 4n(n − 1)
t
2
+ 2
+ ≥ + p .
ai ai aj aj ai a j ai a j

Thus, it suffices to show that


X 1 n(n − 1)
≥ ,
ai a j 2
p
1≤i< j≤n

which follows immediately from the AM-GM inequality. The equality holds for a1 =
a2 = · · · = an = 1.
Symmetric Nonrational Inequalities 191

Second Solution. For the sake of contradiction, assume that


n
X 1 1
< .
1+ 1 + 4n(n − 1)ai
p
i=1
2

It suffices to show that a1 a2 · · · an > 1. Using the substitution


xi 1
= , i = 1, 2, · · · , n,
2n 1 + 1 + 4n(n − 1)ai
p

which yields
n − xi
ai = , 0 < x i < n, i = 1, 2, · · · , n,
(n − 1)x i2
we need to show that
x1 + x2 + · · · + x n < n
implies
(n − x 1 )(n − x 2 ) · · · (n − x n ) > (n − 1)n x 12 x 22 · · · x n2 .
By the AM-GM inequality, we have
 x + x + · · · + x n
1 2 n
x1 x2 · · · x n ≤ <1
n
and
v
t x1 x2 · · · x n
n − x i > (x 1 + x 2 + · · · + x n ) − x i ≥ (n − 1) n−1
, i = 1, 2, · · · , n.
xi

Therefore, we get

(n − x 1 )(n − x 2 ) · · · (n − x n ) > (n − 1)n x 1 x 2 · · · x n > (n − 1)n x 12 x 22 · · · x n2 .

P 1.128. If f is a convex function on a real interval I and a1 , a2 , . . . , an ∈ I, then


a + a + ··· + a 
1 2 n
f (a1 ) + f (a2 ) + · · · + f (an ) + n(n − 2) f ≥
n
≥ (n − 1)[ f (b1 ) + f (b2 ) + · · · + f (bn )],
where
1 X
bi = aj, i = 1, 2, · · · , n.
n − 1 j6=i

(Tiberiu Popoviciu, 1965)


192 Vasile Cîrtoaje

Solution. Without loss of generality, we may assume that n ≥ 3 and a1 ≤ a2 ≤ · · · ≤ an .


There is an integer m, 1 ≤ m ≤ n − 1, such that
a1 ≤ · · · ≤ am ≤ a ≤ am+1 ≤ · · · ≤ an ,
b1 ≥ · · · ≥ bm ≥ a ≥ bm+1 ≥ · · · ≥ bn .
We can get the desired inequality by summing the following two inequalities:
f (a1 ) + f (a2 ) + · · · + f (am ) + n(n − m − 1) f (a) ≥
≥ (n − 1)[ f (bm+1 ) + f (bm+2 ) + · · · + f (bn )], (*)

f (am+1 ) + f (am+2 ) + · · · + f (an ) + n(m − 1) f (a) ≥


≥ (n − 1)[ f (b1 ) + f (b2 ) + · · · + f (bm )]. (**)
In order to prove (*), we apply Jensen’s inequality to get
f (a1 ) + f (a2 ) + · · · + f (am ) + (n − m − 1) f (a) ≥ (n − 1) f (b),
where
a1 + a2 + · · · + am + (n − m − 1)a
b= .
n−1
Thus, it suffices to show that
(n − m − 1) f (a) + f (b) ≥ f (bm+1 ) + f (bm+2 ) + · · · + f (bn ).
Since
a ≥ bm+1 ≥ bm+2 ≥ · · · ≥ bn , (n − m − 1)a + b = bm+1 + bm+2 + · · · + bn ,

→ −

we see that A n−m = (a, · · · , a, b) majorizes B n−m = (bm+1 , bm+2 , · · · , bn ). Therefore,
the inequality is a consequence of Karamata’s inequality.
Similarly, we can prove the inequality (**), by summing Jensen’s inequality
f (am+1 ) + f (am+2 ) + · · · + f (an ) + (m − 1) f (a)
≥ f (c)
n−1
and the inequality
f (c) + (m − 1) f (a) ≥ f (b1 ) + f (b2 ) + · · · + f (bm ),
where
am+1 + am+2 + · · · + an + (m − 1)a
c= .
n−1
The last inequality follows by Karamata’s inequality, because
b1 ≥ b2 ≥ · · · ≥ bm ≥ a, c + (m − 1)a = b1 + b2 + · · · + bm ,

→ −

therefore C m = (c, a, · · · , a) majorizes D m = (b1 , b2 , · · · , bm ).
Symmetric Nonrational Inequalities 193

P 1.129. Let a1 , a2 , . . . , an (n ≥ 3) be positive real numbers such that a1 a2 · · · an = 1.


Prove that
n
X 1 1
≤ .
i=1 n − 1 + (n − 1)2 + 4nai
p
2

Solution. Use the contradiction method. Assume that


n
X 1 1
> ,
n−1+ (n − 1)2 + 4nai
p
i=1
2

and show that a1 a2 · · · an < 1. Using the substitution


1 n − ci
= , i = 1, 2, · · · , n,
n−1+ (n − 1)2 + 4nai 2n(n − 1)
p

which involves
(n − 1)2 ci
ai = , 0 < ci < n, i = 1, 2, · · · , n,
(n − ci )2
we need to show that
c1 + c2 + · · · + cn < n
implies
 n − c 2  n − c 2  n − c 2
1 2 n
··· > c1 c2 · · · cn .
n−1 n−1 n−1
Clearly, it suffices to show that

c1 + c2 + · · · + cn = n

implies
 n − c 2  n − c 2  n − c 2
1 2 n
···
≥ c1 c2 · · · cn .
n−1 n−1 n−1
Popoviciu’s inequality (see the preceding P 1.128) applied to the convex function f (x) =
− ln x, x > 0 gives
 n − c n−1  n − c n−1  n − c n−1  c + c + · · · + c n(n−2)
1 2 n 1 2 n
··· ≥ c1 c2 · · · cn ,
n−1 n−1 n−1 n
 n − c n−1  n − c n−1  n − c n−1
1 2 n
··· ≥ c1 c2 · · · cn ,
n−1 n−1 n−1
 n − c 2  n − c 2  n − c 2 2
1 2 n
··· ≥ (c1 c2 · · · cn ) n−1 .
n−1 n−1 n−1
Thus, it suffices to show that
2
(c1 c2 · · · cn ) n−1 ≥ c1 c2 · · · cn .
194 Vasile Cîrtoaje

For n = 3, this inequality is an equality, while for n ≥ 4, it is equivalent to c1 c2 · · · cn ≤ 1.


Indeed, by the AM-GM inequality, we have
 c + c + · · · + c n
1 2 n
c1 c2 · · · cn ≤ = 1.
n

The equality holds for a1 = a2 = · · · = an = 1.

P 1.130. If a1 , a2 , . . . , an are positive real numbers such that a1 a2 · · · an = 1, then


v
t a1 + a22 + · · · + an2
u 2
a1 + a2 + · · · + an ≥ n − 1 + .
n

Solution. Let us denote


v P
a1 + a2 + · · · + an t 2 1≤i< j≤n ai a j
u
a= , b= ,
n n(n − 1)

where b ≥ 1 (by the AM-GM inequality). We need to show that


v
t n2 a2 − n(n − 1)b2
na − n + 1 ≥ .
n

By squaring, this inequality becomes

(n − 1)[n(a − 1)2 + b2 − 1] ≥ 0,

which is clearly true. The equality holds for a1 = a2 = · · · = an = 1.

P 1.131. If a1 , a2 , . . . , an are positive real numbers such that a1 a2 · · · an = 1, then


q Æ
(n − 1)(a12 + a22 + · · · + an2 ) + n − n(n − 1) ≥ a1 + a2 + · · · + an .

(Vasile Cîrtoaje, 2006)


Symmetric Nonrational Inequalities 195

Solution. We use the induction method. For n = 2, the inequality is equivalent to the
obvious inequality
1
a1 + ≥ 2.
a1
Assume now that the inequality holds for n − 1 numbers, n ≥ 3, and prove that it holds
also for n numbers. Let a1 = min{a1 , a2 , . . . , an }, and denote
a2 + a3 + · · · + an
x= y=
p
, n−1
a2 a3 · · · an ,
n−1
q Æ
f (a1 , a2 , . . . , an ) = (n − 1)(a12 + a22 + · · · + an2 ) + n − n(n − 1) − (a1 + a2 + · · · + an ).
By the AM-GM inequality, we have x ≥ y. We will show that

f (a1 , a2 , . . . , an ) ≥ f (a1 , y, · · · , y) ≥ 0. (*)

Write the left inequality as


q q p
a12 + a22 + · · · + an2 − a12 + (n − 1) y 2 ≥ n − 1 (x − y).

To prove this inequality, we use the induction hypothesis, written in the homogeneous
form
q ” Æ —
(n − 2)(a22 + a32 + · · · + an2 ) + n − 1 − (n − 1)(n − 2) y ≥ (n − 1)x,

which is equivalent to
a22 + · · · + an2 ≥ (n − 1)A2 ,
where v
tn − 1
A = k x − (k − 1) y, k= .
n−2
So, we need to prove that
q q p
a12 + (n − 1)A2 − a12 + (n − 1) y 2 ≥ n − 1 (x − y).

Write this inequality as

A2 − y 2 x−y
q q ≥p .
a1 + (n − 1)A2 + a1 + (n − 1) y 2
2 2 n−1

Since x ≥ y and
A2 − y 2 = k(x − y)[k x − (k − 2) y],
we need to show that
k[k x − (k − 2) y] 1
q q ≥p .
a12 + (n − 1)A2 + a1 + (n − 1) y 2
2 n−1
196 Vasile Cîrtoaje

In addition, since a1 ≤ y, it suffices to show that

k[k x − (k − 2) y] 1
p ≥p .
y 2 + (n − 1)A2 + n y
p
n−1

We claim that Æ p
y 2 + (n − 1)A2 ≤ k n − 1 [k x − (k − 1) y] .
If this inequality is true, then it is enough to prove that

k[k x − (k − 2) y] 1
p p ≥p .
k n − 1[k x − (k − 1) y] + ny n−1

Rewrite this inequality as

k[k x − (k − 2) y]
s
n
≥ 1, m= .
k[k x − (k − 1) y] + m y n−1

Since m < k, we have

k[k x − (k − 2) y] k[k x − (k − 2) y]
> = 1.
k[k x − (k − 1) y] + m y k[k x − (k − 1) y] + k y

Using the relation a1 y n−1 = 1, we can write the right inequality in (*) as
Æ
(n − 1)[(n − 1) y 2n + 1] ≥ (n − 1) y n − t y n−1 + 1,

where Æ
t = n− n(n − 1).
This inequality is true if

(n − 1)[(n − 1) y 2n + 1] ≥ [(n − 1) y n − t y n−1 + 1]2 ,

that is,
2(n − 1)t y 2n−1 − t 2 y 2n−2 − 2(n − 1) y n + 2t k y n−1 + n − 2 ≥ 0.
Since
t 2 = n(2t − 1),
we can write this inequality in the form

2t y n−1 B + C ≥ 0,

where
B = (n − 1) y n − n y n−1 + 1, C = n y 2n−2 − 2(n − 1) y n + n − 2.
Symmetric Nonrational Inequalities 197

This inequality is true if b ≥ 0 and C ≥ 0. Indeed, by the AM-GM inequality, we have


Æ
n
(n − 1) y n + 1 ≥ n y (n−1)n · 1 = n y n−1

and Æ
2n−2
n y 2n−2 + n − 2 ≥ (2n − 2) y n(2n−2) · 1n−2 = 2(n − 1) y n .
The proof is completed. The equality holds for a1 = a2 = · · · = an = 1.

P 1.132. Let a1 , a2 , . . . , an (n ≥ 3) be positive real numbers such that a1 a2 · · · an = 1. If


2n − 1
0<p≤ ,
(n − 1)2
then
1 1 1 n
+p +p ≤p .
1 + pa1 1 + pa2 1 + pan 1+p
p

(Vasile Cîrtoaje and Gabriel Dospinescu, 2006)

Solution. We use the contradiction method. Assume that the reverse inequality holds,
namely
1 1 1 n
+p +p >p ,
1 + pa1 1 + pa2 1 + pan 1+p
p

and show that this implies a1 a2 · · · an < 1. Using the substitution

1+p
p
p
1 + pai = , 0 < x i < p + 1, i = 1, 2, · · · , n,
p
xi
we need to show that x 1 + x 2 + · · · + x n > n yields

1+p 1+p 1+p


    
−1 − 1 ··· − 1 < pn.
x 12 x 22 x n2

Clearly, it suffices to prove that

x1 + x2 + · · · + x n = n

yields
1+p 1+p 1+p
    
−1 − 1 ··· − 1 ≤ pn.
x 12 x 22 x n2
Denoting
p n
1 + p = q, 1<q≤ ,
n−1
198 Vasile Cîrtoaje

we need to show that

(q2 − x 12 )(q2 − x 22 ) · · · (q2 − x n2 ) ≤ (q2 − 1)n (x 1 x 2 · · · x n )2 (*)

for all x i ∈ (0, q) satisfying x 1 + x 2 + · · · + x n = n. Applying Popoviciu’s inequality (see


P 1.128) to the convex function
 n 
f (x) = − ln − x , 0 < x < 1,
n−1
we get

(x 1 x 2 · · · x n )n−1 ≥ [n − (n − 1)x 1 ][n − (n − 1)x 2 ] · · · [n − (n − 1)x n ]. (**)

On the other hand, Jensen’s inequality applied to the convex function


n − (n − 1)x
f (x) = ln
q−x
yields
[n − (n − 1)x 1 ][n − (n − 1)x 2 ] · · · [n − (n − 1)x n ] 1
≥ .
(q − x 1 )(q − x 2 ) · · · (q − x n ) (q − 1)n
Multiplying this inequality and (**) gives
(q − x 1 )(q − x 2 ) · · · (q − x n )
(x 1 x 2 · · · x n )n−1 ≥ .
(q − 1)n
Therefore, in order to prove (*), we still have to show that

(x 1 x 2 · · · x n )n−3 (q + x 1 )(q + x 2 ) · · · (q + x n ) ≤ (q + 1)n .

This is true because, by the AM-GM inequality, we have


 x + x + · · · + x n
1 2 n
x1 x2 · · · x n ≤ =1
n
and  x 1 + x 2 + · · · + x n n
(q + x 1 )(q + x 2 ) · · · (q + x n ) ≤ q + = (q + 1)n .
n
The equality occurs for a1 = a2 = · · · = an = 1.
2n − 1
Remark. For p = , we get the following inequality
(n − 1)2
n
X 1
≤ 1,
(n − 1)2 + (2n − 1)ai
p
i=1

which holds for all positive numbers a1 , a2 , . . . , an (n ≥ 3) satisfying a1 a2 · · · an = 1.


Symmetric Nonrational Inequalities 199

P 1.133. If a1 , a2 , . . . , an (n ≥ 3) are positive real numbers such that a1 a2 · · · an = 1, then

n Ç
‚ n
Œ2
X X
(n − 1)2 ai4 + 2n − 1 ≥ ai ;
i=1 i=1

n
‚ n
Œ2
X q X
ai (n − 1)2 ai2 + 2n − 1 ≥ ai .
i=1 i=1

(Vasile Cîrtoaje, 2007)

Solution. According to the preceding P 1.132, the following inequality holds for any
real m:
n
X 1
≤ 1.
(n − 1)2 + (2n − 1)aim
Æ
i=1

On the other hand, by the Cauchy-Schwarz inequality, we have


‚ n
Œ‚ n
Œ ‚ n
Œ2
X 1 X q X
ai2 (n − 1)2 + (2n − 1)aim ≥ ai ,
(n − 1)2 + (2n − 1)aim
Æ
i=1 i=1 i=1

hence Œ2
n n
‚
X q X
ai2 (n − 1)2 + (2n − 1)aim ≥ ai .
i=1 i=1

Setting m = −4 and m = −2 gives the desired inequalities. The equality occurs for
a1 = a2 = · · · = an = 1.

P 1.134. Let a1 , a2 , . . . , an be positive real numbers such that a1 a2 · · · an ≥ 1. If k > 1,


then
X a1k
≥ 1.
a1k + a2 + · · · + an
(Vasile Cîrtoaje, 2006)
p
First Solution. Let us denote r = n a1 a2 · · · an and bi = ai /r for i = 1, 2, · · · , n. Note
that r ≥ 1 and b1 b2 · · · bn = 1. The desired inequality becomes

X b1k
≥ 1,
b1k + (b2 + · · · + bn )/r k−1
200 Vasile Cîrtoaje

and we see that it suffices to prove it for r = 1; that is, for a1 a2 · · · an = 1. On this
hypothesis, we will show that there exists a positive number p such that
p
a1k a1
≥ p p p.
a1k + a2 + · · · + an a1 + a2 + · · · + an

If this is true, by adding this inequality and the analogous inequalities for a2 , . . . , an , we
get the desired inequality. Write the claimed inequality as
p
a2 + · · · + anp ≥ (a2 · · · an )k−p (a2 + · · · + an ).

For
(n − 1)k + 1
p= , p > 1,
n
this inequality turns into the homogeneous inequality
p−1
p
a2 + · · · + anp ≥ (a2 · · · an ) n−1 (a2 + · · · + an ).

Based on the AM-GM inequality


p−1
 a + · · · + a  p−1
2 n
(a2 · · · an ) n−1 ≤ ,
n−1
we only need to show that
p p
a2 + · · · + a n  a + · · · + a p
2 n
≥ ,
n−1 n−1
which is just Jensen’s inequality applied to the convex function f (x) = x p . The equality
holds for a1 = a2 = · · · = an = 1.
Second Solution. By the Cauchy-Schwarz inequality, we have
 ‹2
P k+1 2 k+1
a a1 + 2 1≤i< j≤n (a1 a2 ) 2
P k+1 P
X a1k 1
≥P = P k+1 .
a1k + a2 + · · · + an a1 (a1k + a2 + · · · + an ) a1 + 2 1≤i< j≤n a1 a2
P

Thus, it suffices to show that


X k+1
X
(a1 a2 ) 2 ≥ a1 a2 .
1≤i< j≤n 1≤i< j≤n

k+1
Jensen’s inequality applied to the convex function f (x) = x 2 yields
‚ P Œ k+1
X k+1 n(n − 1) 2 1≤i< j≤n a1 a2 2
(a1 a2 ) 2 ≥
1≤i< j≤n
2 n(n − 1)
Symmetric Nonrational Inequalities 201

On the other hand, by the AM-GM inequality, we get


2 X 2
a1 a2 ≥ (a1 a2 · · · an ) n ≥ 1.
n(n − 1) 1≤i< j≤n

Therefore,
‚ P Œ k+1 ‚ P Œ
2 1≤i< j≤n a1 a2 2 2 1≤i< j≤n a1 a2
≥ ,
n(n − 1) n(n − 1)
hence ‚ P Œ
X k+1 n(n − 1) 2 1≤i< j≤n a1 a2 X
(a1 a2 ) 2 ≥ = a1 a2 .
1≤i< j≤n
2 n(n − 1) 1≤i< j≤n

P 1.135. Let a1 , a2 , . . . , an be positive real numbers such that a1 a2 · · · an ≥ 1. If


−2
≤ k < 1,
n−2
then
X a1k
≤ 1.
a1k + a2 + · · · + an
(Vasile Cîrtoaje, 2006)
p
Solution. Let us denote r = n a1 a2 · · · an and bi = ai /r for i = 1, 2, · · · , n. Notice that
r ≥ 1 and b1 b2 · · · bn = 1. The desired inequality becomes
X b1k
≤ 1,
b1k + (b2 + · · · + bn )r 1−k

and we see that it suffices to prove it for r = 1; that is, for a1 a2 · · · an = 1. On this
hypothesis, we will prove the desired inequality by summing the inequality
p
a1k a1
≤ p p p
a1k + a2 + · · · + an a1 + a2 + · · · + an

and the analogous inequalities for a2 , . . . , an , where


(n − 1)k + 1 −1
p= , ≤ p < 1.
n n−2
Rewrite this inequality as
1−p
p
a2 + · · · + an ≥ (a2 · · · an ) n−1 (a2 + · · · + anp ). (*)
202 Vasile Cîrtoaje

To prove it, we use the weighted AM-GM inequality

1 + (n − 2)p n − 1 1+(n−2)p 1−p


a2 + a3 + · · · + an ≥ a2 n−1 (a3 · · · an ) n−1 ,
1−p 1−p

which is equivalent to

1 + (n − 2)p n−1 p 1−p


a2 + a3 + · · · + an ≥ a2 (a2 a3 · · · an ) n−1 .
1−p 1−p

Adding this inequality and the analogous inequalities for a3 , · · · , an yields the inequality
(*). Thus, the proof is completed. The equality holds for a1 = a2 = · · · = an = 1.

P 1.136. Let a1 , a2 , . . . , an be positive real numbers such that a1 a2 · · · an ≥ 1. If k > 1,


then X a1
≤ 1.
a1 + a2 + · · · + an
k

(Vasile Cîrtoaje, 2006)


1
Solution. We consider two cases: 1 < k ≤ n + 1 and k ≥ n − .
n−1
Case 1: 1 < k ≤ n + 1. By the AM-GM inequality, the hypothesis a1 a2 · · · an ≥ 1 yields
a1 + a2 + · · · + an ≥ n. Therefore, it suffices to prove that the desired inequality holds
for a1 + a2 + · · · + an ≥ n. Actually, we only need to consider that a1 + a2 + · · · + an = n.
Indeed, if we denote p = (a1 + a2 + · · · + an )/n and bi = ai /p for i = 1, 2, · · · , n, then
the desired inequality becomes
X b1
≤ 1, p ≥ 1.
p k−1 b1k + b2 + · · · + bn

Clearly, it suffices to consider the case p = 1; that is, a1 + a2 + · · · + an = n. On this


hypothesis, we can rewrite the desired inequality as
X a1
≤ 1.
a1 − a1 + n
k

Since k > 1, by Bernoulli’s inequality, we have

a1k − a1 + n ≥ 1 + k(a1 − 1) − a1 + n = n − k + 1 + (k − 1)a1 > 0.

Thus, it is enough to show that


X a1
≤ 1,
n − k + 1 + (k − 1)a1
Symmetric Nonrational Inequalities 203

which is equivalent to
X 1
≥ 1.
n − k + 1 + (k − 1)a1
This inequality follows immediately from the AM-HM inequality
€X Š X 1 ‹
x1 ≥ n2 ,
x1

for x i = n − k + 1 + (k − 1)ai , i = 1, 2, · · · , n.
1 p
Case 2: k ≥ n − . Let us denote r = n a1 a2 · · · an and bi = ai /r for i = 1, 2, · · · , n.
n−1
Note that r ≥ 1 and b1 b2 · · · bn = 1. The desired inequality becomes
X b1
≤ 1,
b1k r k−1 + b2 + · · · + b n

and we see that it suffices to prove it for r = 1; that is, for a1 a2 · · · an = 1. On this
hypothesis, it suffices to show that
p
(n − 1)a1 a1
+ p p p ≤1
a1k + a2 + · · · + an a1 + a2 + · · · + an

for a suitable real p; then, adding this inequality and the analogous inequalities for
p
a2 , · · · an yields the desired inequality. Let us denote t = n−1 a2 · · · an . By the AM-GM
inequality, we have
p
a2 + · · · an ≥ (n − 1)t, a2 + · · · + anp ≥ (n − 1)t p .

Thus, it suffices to show that


p
(n − 1)a1 a1
+ p ≤ 1.
a1k + (n − 1)t a1 + (n − 1)t p

Since a1 = 1/t n−1 , this inequality is equivalent to

(n − 1)t n+q − (n − 1)t q − t q−np + 1 ≥ 0,

where
q = (n − 1)(k − 1).
Choosing
(n − 1)(k − n − 1)
p= ,
n
the inequality becomes as follows

(n − 1)t n+q − (n − 1)t q − t n(n−1) + 1 ≥ 0,


204 Vasile Cîrtoaje

2 2
−2n −3n
(n − 1)t q (t n − 1) − (t n − 1)(t n + tn + · · · + 1) ≥ 0,
2 2
−2n −3n
(t n − 1)[(t q − t n ) + (t q − t n ) + · · · + (t q − 1)] ≥ 0.
1
The last inequality is clearly true for q ≥ n2 − 2n; that is, for k ≥ n − . Thus, the
n−1
proof is completed. The equality holds for a1 = a2 = · · · = an = 1.

P 1.137. Let a1 , a2 , . . . , an be positive real numbers such that a1 a2 · · · an ≥ 1. If

2
−1 − ≤ k < 1,
n−2
then X a1
≥ 1.
a1k + a2 + · · · + an
(Vasile Cîrtoaje, 2006)
p
Solution. Let us denote r = n a1 a2 · · · an and bi = ai /r for i = 1, 2, · · · , n. Note that
r ≥ 1 and b1 b2 · · · bn = 1. The desired inequality becomes
X b1
≥ 1,
b1k /r 1−k + b2 + · · · + bn

and we see that it suffices to prove it for r = 1; that is, for a1 a2 · · · an = 1. On this
hypothesis, by the Cauchy-Schwarz inequality, we have

( a1 )2 ( a1 )2
P P
X a1
≥P = P P .
a1k + a2 + · · · + an a1 (a1k + a2 + · · · + an ) ( a1 )2 + a11+k − a12
P

Thus, we still have to show that


X X
a12 ≥ a11+k .

Case 1: −1 ≤ k < 1. Using Chebyshev’s inequality and the AM-GM inequality yields
X 1 €X 1−k Š €X 1+k Š X X
a12 ≥ a1 a1 ≥ (a1 a2 · · · an )(1−k)/n a11+k = a11+k .
n
2
Case 2: −1 − ≤ k < −1. It is convenient to replace the numbers a1 , a2 , · · · , an
n−1
(n−1)/2 (n−1)/2
by a1 , a2 , · · · , an(n−1)/2 , respectively. So, we need to show that a1 a2 · · · an = 1
involves X X
q
a1n−1 ≥ a1 ,
Symmetric Nonrational Inequalities 205

where
(n − 1)(1 + k)
q= , −1 ≤ q < 0.
2
By the AM-GM inequality, we get
X 1 X n−1 X X 1
a1n−1 = (a2 + · · · + ann−1 ) ≥ a2 · · · an = .
n−1 a1

Thus, it suffice to show that


X 1 X
q
≥ a1 .
a1
By Chebyshev’s inequality and the AM-GM inequality, we have
X 1 1 €X −1−q Š €X q Š €X Š X
q q
≥ a1 a1 ≥ (a1 a2 · · · an )−(1+q)/n a1 = a1 .
a1 n

Thus, the proof is completed. The equality holds for a1 = a2 = · · · = an = 1.

P 1.138. Let a1 , a2 , . . . , an be positive real numbers such that a1 a2 · · · an = 1. If k ≥ 0,


then X 1
≤ 1.
a1 + a2 + · · · + an
k

(Vasile Cîrtoaje, 2006)

Solution. Consider two cases: 0 ≤ k ≤ 1 and k ≥ 1.


Case 1: 0 ≤ k ≤ 1. By the Cauchy-Schwarz inequality and the AM-GM inequality, we
have
1 a11−k + 1 + · · · + 1
≤ p p p
a1k + a2 + · · · + an ( a1 + a2 + · · · + an )2

a11−k + n − 1 a11−k + n − 1
=P ≤P ,
a1 + 2 1≤i< j≤n ai a j a1 + n(n − 1)
P p

hence
a11−k + n(n − 1)
P
X 1
≤ .
a1 + n(n − 1)
P
a1k + a2 + · · · + an
Therefore, it suffices to show that
X X
a11−k ≤ a1 .
206 Vasile Cîrtoaje

Indeed, by Chebyshev’s inequality and the AM-GM inequality, we have


X X 1 €X k Š €X 1−k Š €X Š X
a1 = a1k · a11−k ≥ a1 a1 ≥ (a1 a2 · · · an )k/n a11−k = a11−k .
n

Case 2: k ≥ 1. Write the inequality as

X p 
n−1 a1
+ p p p − 1 ≤ 0,
a1k + a2 + · · · + an a1 + a2 + · · · + an

where
(n − 1)(k − 1)
p= ≥ 0.
n
To complete the proof, it suffices to show that
p
n−1 a1
≤1− p p p.
a1k + a2 + · · · + an a1 + a2 + · · · + an

Let
x= x > 0.
p
n−1
a1 ,
By the AM-GM inequality, we have

n−1 n−1
a2 + · · · + an ≥ (n − 1) n−1 a2 · · · an = n−1
p =
p
a1 x

and
p
Æ n−1 n−1
a2 + · · · + anp ≥ (a2 · · · an ) p = q p =
n−1
.
n−1
a1 xp

Thus, it suffices to show that

n−1 x (n−1)p
n−1
≤1− n−1
,
x (n−1)k + x x (n−1)p + xp

which is equivalent to
x 1
≤ ,
x (n−1)k+1 +n−1 x np + n − 1
(n−1)k+1
x − x np+1 − (n − 1)(x − 1) ≥ 0,

x q (x n−1 − 1) − (n − 1)(x − 1) ≥ 0,
where
q = (n − 1)(k − 1) + 1 ≥ 1.
Symmetric Nonrational Inequalities 207

The last inequality is true since

x q (x n−1 − 1) − (n − 1)(x − 1) = (x q − 1)(x n−1 − 1) + (x n−1 − 1) − (n − 1)(x − 1),

and
(x q − 1)(x n−1 − 1) ≥ 0,
(x n−1 − 1) − (n − 1)(x − 1) = (x − 1)[(x n−2 − 1) + (x n−3 − 1) + · · · + (x − 1)] ≥ 0.
This completes the proof. The equality holds for a1 = a2 = · · · = an = 1.

P 1.139. Let a1 , a2 , . . . , an be nonnegative real numbers such that a1 + a2 + · · · + an ≥ n.


If 1 < k ≤ n + 1, then
a1 a2 an
+ + ··· + ≤ 1.
a1k + a2 + · · · + an a1 + a2k + · · · + an a1 + a2 + · · · + ank

(Vasile Cîrtoaje, 2006)

Solution. Using the substitutions


a1 + a2 + · · · + an
s= ,
n
and
a1 a an
x1 = , x2 = 2 , · · · , x n = ,
s s s
the desired inequality becomes
x1 xn
+ ··· + ≤ 1,
s k−1 x 1k + x2 + · · · + x n x 1 + x 2 + · · · + s k−1 x nk

where s ≥ 1 and x 1 + x 2 + · · · + x n = n. Clearly, if this inequality holds for s = 1,


then it holds for any s ≥ 1. Therefore, we need only to consider the case s = 1, when
a1 + a2 + · · · + an = n, and the desired inequality is equivalent to
a1 a2 an
+ + ··· + ≤ 1.
a1k − a1 + n a2k − a2 + n ank − an + n

By Bernoulli’s inequality, we have

a1k − a1 + n ≥ 1 + k(a1 − 1) − a1 + n = n − k + 1 + (k − 1)a1 ≥ 0.

Consequently, it suffices to prove that


n
X ai
≤ 1.
i=1
n − k + 1 + (k − 1)ai
208 Vasile Cîrtoaje

For k = n + 1, this inequality is an equality. Otherwise, for 1 < k < n + 1, we rewrite the
inequality as
n
X 1
≥ 1,
i=1
n − k + 1 + (k − 1)ai

which follows from the AM-HM inequality as follows:


n
X 1 n2
≥ Pn = 1.
i=1
n − k + 1 + (k − 1)ai i=1 [n − k + 1 + (k − 1)ai ]

The equality holds for a1 = a2 = · · · = an = 1.

P 1.140. Let a1 , a2 , . . . , an be nonnegative real numbers such that a1 + a2 + · · · + an ≤ n.


If 0 ≤ k < 1, then

1 1 1
+ + ··· + ≥ 1.
a1k + a2 + · · · + an a1 + a2k + · · · + an a1 + a2 + · · · + ank

Solution. By the AM-HM inequality


X 1 n2 n2
≥ =
a1k + a2 + · · · + an (a1k + a2 + · · · + an ) a1k + (n − 1) a1
P P P

and Jensen’s inequality


 X ‹k
X 1
a1k ≤n a1 ,
n
we get
X 1 n2
≥ ≥ 1.
a1k + a2 + · · · + an 1
k
+ (n − 1)
P P
n n a1 a1
The equality holds for a1 = a2 = · · · = an = 1.

P 1.141. Let a1 , a2 , . . . , an be positive real numbers. If k > 1, then

X ak + ak + · · · + ak n(a1k + a2k + · · · + ank )


2 3 n
≤ .
a2 + a3 + · · · + an a1 + a2 + · · · + an

(Wolfgang Berndt, Vasile Cîrtoaje, 2006)


Symmetric Nonrational Inequalities 209

Solution. Due to homogeneity, we may assume that a1 + a2 + ... + an = 1. Write the


inequality as follows:
X a1
‹
1+ (a2k + a3k + · · · + ank ) ≤ n(a1k + a2k + · · · + ank );
a2 + a3 + · · · + an

X a1 (a k + a k + · · · + a k )
2 3 n
≤ a1k + a2k + · · · + ank ;
a2 + a3 + · · · + an
+ + +
‚ k k k
Œ
X a 2 a3 · · · a n
a1 a1k−1 − ≥ 0;
a2 + a3 + · · · + an
X a1 a2 (a k−1 − a k−1 ) + a1 a3 (a k−1 − a k−1 ) + · · · + a1 an (a k−1 − a k−1 )
1 2 1 3 1 n
≥ 0;
a2 + a3 + · · · + an
!
X aik−1 − a k−1
j a k−1
j − aik−1
ai a j + ≥ 0;
1≤i< j≤n
1 − ai 1 − aj

X ai a j (aik−1 − a k−1
j )(ai − a j )
≥ 0.
1≤i< j≤n
(1 − ai )(1 − a j )

Since the last inequality is true for k > 1, the proof is finished. The equality holds for
a1 = a2 = · · · = an .
210 Vasile Cîrtoaje
Appendix A

Glossary

1. AM-GM (ARITHMETIC MEAN-GEOMETRIC MEAN) INEQUALITY


If a1 , a2 , · · · , an are nonnegative real numbers, then

a1 + a2 + · · · + an ≥ n n a1 a2 · · · an ,
p

with equality if and only if a1 = a2 = · · · = an .

ƒ
2. WEIGHTED AM-GM INEQUALITY
Let p1 , p2 , · · · , pn be positive real numbers satisfying

p1 + p2 + · · · + pn = 1.

If a1 , a2 , · · · , an are nonnegative real numbers, then


p p
p1 a1 + p2 a2 + · · · + pn an ≥ a1 1 a2 2 · · · anpn ,

with equality if and only if a1 = a2 = · · · = an .

ƒ
3. AM-HM (ARITHMETIC MEAN-HARMONIC MEAN) INEQUALITY
If a1 , a2 , · · · , an are positive real numbers, then

1 1 1
 ‹
(a1 + a2 + · · · + an ) + + ··· + ≥ n2 ,
a1 a2 an

with equality if and only if a1 = a2 = · · · = an .

211
212 Vasile Cîrtoaje

4. POWER MEAN INEQUALITY


The power mean of order k of positive real numbers a1 , a2 , · · · , an , that is

1
a1 +a2 +···+ank k
 k k
, k=


 n 6 0
Mk = p ,



n
a a
1 2 · · · a n , k = 0

is an increasing function with respect to k ∈ R. For instant, M2 ≥ M1 ≥ M0 ≥ M−1 is


equivalent to
v
t a1 + a22 + · · · + an2
u 2
a1 + a2 + · · · + an p n
≥ ≥ n a1 a2 · · · an ≥ .
n n 1 1 1
+ + ··· +
a1 a2 an

ƒ
5. BERNOULLI’S INEQUALITY
For any real number x ≥ −1, we have
a) (1 + x) r ≥ 1 + r x for r ≥ 1 and r ≤ 0;
b) (1 + x) r ≤ 1 + r x for 0 ≤ r ≤ 1.
In addition, if a1 , a2 , · · · , an are real numbers such that either a1 , a2 , · · · , an ≥ 0 or −1 ≤
a1 , a2 , · · · , an ≤ 0, then

(1 + a1 )(1 + a2 ) · · · (1 + an ) ≥ 1 + a1 + a2 + · · · + an .

ƒ
6. SCHUR’S INEQUALITY
For any nonnegative real numbers a, b, c and any positive number k, the inequality
holds
a k (a − b)(a − c) + b k (b − c)(b − a) + c k (c − a)(c − b) ≥ 0,
with equality for a = b = c, and for a = 0 and b = c (or any cyclic permutation).
For k = 1, we get the third degree Schur’s inequality, which can be rewritten as follows

a3 + b3 + c 3 + 3a bc ≥ a b(a + b) + bc(b + c) + ca(c + a),

(a + b + c)3 + 9a bc ≥ 4(a + b + c)(a b + bc + ca),


9a bc
a2 + b2 + c 2 + ≥ 2(a b + bc + ca),
a+b+c
(b − c)2 (b + c − a) + (c − a)2 (c + a − b) + (a − b)2 (a + b − c) ≥ 0.
Glossary 213

For k = 2, we get the fourth degree Schur’s inequality, which holds for any real numbers
a, b, c, and can be rewritten as follows

a4 + b4 + c 4 + a bc(a + b + c) ≥ a b(a2 + b2 ) + bc(b2 + c 2 ) + ca(c 2 + a2 ),

(b − c)2 (b + c − a)2 + (c − a)2 (c + a − b)2 + (a − b)2 (a + b − c)2 ≥ 0,


6a bc p ≥ (p2 − q)(4q − p2 ),
where p = a + b + c, q = a b + bc + ca.
A generalization of the fourth degree Schur’s inequality, which holds for any real num-
bers a, b, c and any real number m, is the following (Vasile Cirtoaje, 2004):
X
(a − mb)(a − mc)(a − b)(a − c) ≥ 0,

where the equality holds for a = b = c, and for a/m = b = c (or any cyclic permutation).
This inequality is equivalent to
X X X X
a4 + m(m + 2) a2 b2 + (1 − m2 )a bc a ≥ (m + 1) a b(a2 + b2 ),
X
(b − c)2 (b + c − a − ma)2 ≥ 0.
Another generalization of the fourth degree Schur’s inequality (Vasile Cirtoaje, 2004):
Let α, β, γ be real numbers such that

1 + α + β = 2γ.

The inequality
X X X X
a4 + α a2 b2 + β a bc a≥γ a b(a2 + b2 )

holds for any real numbers a, b, c if and only if

1 + α ≥ γ2 .

ƒ
7. CAUCHY-SCHWARZ INEQUALITY
For any real numbers a1 , a2 , · · · , an and b1 , b2 , · · · , bn we have

(a12 + a22 + · · · + an2 )(b12 + b22 + · · · + bn2 ) ≥ (a1 b1 + a2 b2 + · · · + an bn )2 ,

with equality if and only if ai and bi are proportional for all i.

ƒ
214 Vasile Cîrtoaje

8. HÖLDER’S INEQUALITY
If x i j (i = 1, 2, · · · , m; j = 1, 2, · · · n) are nonnegative real numbers, then
! v !m
m
Y n
X n uY
X m
m
xi j ≥ .
t
xi j
i=1 j=1 j=1 i=1

ƒ
9. CHEBYSHEV’S INEQUALITY
Let a1 ≥ a2 ≥ · · · ≥ an be real numbers.

a) If b1 ≥ b2 ≥ · · · bn , then
n n n
‚ Œ‚ Œ
X X X
n ai bi ≥ ai bi ;
i=1 i=1 i=1

b) If b1 ≤ b2 ≤ · · · ≤ bn , then
n n n
‚ Œ‚ Œ
X X X
n ai bi ≤ ai bi .
i=1 i=1 i=1

ƒ
10. MINKOWSKI’S INEQUALITY
For any real number k ≥ 1 and any positive real numbers a1 , a2 , · · · , an and b1 , b2 , · · · , bn ,
the inequalities hold
 1
Œk Œk k
n n n
‚ ‚
X 1 X X
aik + bi k k
≥ ai + bi  ;
i=1 i=1 i=1

 1
Œk Œk Œk k
n n n n
‚ ‚ ‚
X 1 X X X
aik + bik + ci
k k
≥ ai + bi + ci  .
i=1 i=1 i=1 i=1

ƒ
11. REARRANGEMENT INEQUALITY
(1) If a1 , a2 , · · · , an and b1 , b2 , · · · , bn are two increasing (or decreasing) real se-
quences, and (i1 , i2 , · · · , in ) is an arbitrary permutation of (1, 2, · · · , n), then

a1 b1 + a2 b2 + · · · + an bn ≥ a1 bi1 + a2 bi2 + · · · + an bin


Glossary 215

and
n(a1 b1 + a2 b2 + · · · + an bn ) ≥ (a1 + a2 + · · · + an )(b1 + b2 + · · · + bn ).
(2) If a1 , a2 , · · · , an is decreasing and b1 , b2 , · · · , bn is increasing, then

a1 b1 + a2 b2 + · · · + an bn ≤ a1 bi1 + a2 bi2 + · · · + an bin

and
n(a1 b1 + a2 b2 + · · · + an bn ) ≤ (a1 + a2 + · · · + an )(b1 + b2 + · · · + bn ).
(3) Let b1 , b2 , · · · , bn and c1 , c2 , · · · , cn be two real sequences such that

b1 + · · · + bk ≥ c1 + · · · + ck , k = 1, 2, · · · , n.

If a1 ≥ a2 ≥ · · · ≥ an ≥ 0, then

a1 b1 + a2 b2 + · · · + an bn ≥ a1 c1 + a2 c2 + · · · + an cn .

Notice that all these inequalities follow immediately from the identity
!
X n n
X i
X i
X
ai (bi − ci ) = (ai − ai+1 ) bj − cj ,
i=1 i=1 j=1 j=1

where an+1 = 0.

ƒ
12. MACLAURIN’S INEQUALITY and NEWTON’S INEQUALITY
If a1 , a2 , . . . , an are nonnegative real numbers, then

S1 ≥ S2 ≥ · · · ≥ S n (M acl aur in)

and
Sk2 ≥ Sk−1 Sk+1 , (N ewt on)
where
v X
u
u
u a i1 a i2 · · · a i k
k 1≤i1 <···<ik ≤n
Sk = u
u
 ‹ .
t n
k
ƒ
216 Vasile Cîrtoaje

13. CONVEX FUNCTIONS


A function f defined on a real interval I is said to be convex if

f (αx + β y) ≤ α f (x) + β f ( y)

for all x, y ∈ I and any α, β ≥ 0 with α + β = 1. If the inequality is reversed, then f is


said to be concave.
If f is differentiable on I, then f is (strictly) convex if and only if the derivative f 0 is
(strictly) increasing. If f 00 ≥ 0 on I, then f is convex on I. Also, if f 00 ≥ 0 on (a, b) and
f is continuous on [a, b], then f is convex on [a, b].
A function f : I → R is half convex on a real interval I if there exists a point s ∈ I such
that f is convex on Iu≤s or Iu≥s .
A function f : I → R is right partially convex related to a a point s ∈ I if there exists a
number s0 ∈ I, s0 > s, such that f is convex on Iu∈[s,s0 ] . Also, a function f : I → R is left
partially convex related to a point s ∈ I if there exists a point s0 ∈ I, s0 < s, such that f
is convex on Iu∈[s0 ,s] .
Jensen’s inequality. Let p1 , p2 , . . . , pn be positive real numbers. If f is a convex function
on a real interval I, then for any a1 , a2 , . . . , an ∈ I, the inequality holds

p1 f (a1 ) + p2 f (a2 ) + · · · + pn f (an ) p1 a1 + p2 a2 + · · · + pn an


 ‹
≥f .
p1 + p2 + · · · + p n p1 + p2 + · · · + p n
For p1 = p2 = · · · = pn , Jensen’s inequality becomes
a + a + ··· + a 
1 2 n
f (a1 ) + f (a2 ) + · · · + f (an ) ≥ n f .
n
Based on the following three theorems, we can extend this form of Jensen’s in-
equality to half or partially convex functions.
Half Convex Function-Theorem (Vasile Cirtoaje, 2004). Let f (u) be a function defined
on a real interval I and convex on Iu≤s or Iu≥s , where s ∈ I. The inequality
a + a + ··· + a 
1 2 n
f (a1 ) + f (a2 ) + · · · + f (an ) ≥ n f
n
holds for all a1 , a2 , . . . , an ∈ I satisfying a1 + a2 + . . . + an = ns if and only if

f (x) + (n − 1) f ( y) ≥ n f (s)

for all x, y ∈ I such that x + (n − 1) y = ns.


Right Partially Convex Function-Theorem (Vasile Cirtoaje, 2012). Let f be a function
defined on a real interval I and convex on [s, s0 ], where s, s0 ∈ I, s < s0 . In addition, f is
decreasing on Iu≤s0 and
min f (u) = f (s0 ).
u≥s
Glossary 217

The inequality
a + a + ··· + a 
1 2 n
f (a1 ) + f (a2 ) + · · · + f (an ) ≥ n f
n
holds for all a1 , a2 , · · · , an ∈ I satisfying a1 + a2 + · · · + an = ns if and only if

f (x) + (n − 1) f ( y) ≥ n f (s)

for all x, y ∈ I such that x ≤ s ≤ y and x + (n − 1) y = ns.


Left Partially Convex Function-Theorem (Vasile Cirtoaje, 2012). Let f be a function
defined on a real interval I and convex on [s0 , s], where s0 , s ∈ I, s0 < s. In addition, f is
increasing on Iu≥s0 and satisfies

min f (u) = f (s0 ).


u≤s

The inequality
a + a + ··· + a 
1 2 n
f (a1 ) + f (a2 ) + · · · + f (an ) ≥ n f
n
holds for all x 1 , x 2 , · · · , x n ∈ I satisfying a1 + a2 + · · · + an = ns if and only if

f (x) + (n − 1) f ( y) ≥ n f (s)

for all x, y ∈ I such that x ≥ s ≥ y and x + (n − 1) y = ns.


In all these theorems, we may replace the hypothesis condition

f (x) + (n − 1) f ( y) ≥ n f (s),

by the equivalent condition


h(x, y) ≥ 0 for all x, y ∈ I such that x + (n − 1) y = ns,
where
g(x) − g( y) f (u) − f (s)
h(x, y) = , g(u) = .
x−y u−s
The following theorem is also useful to prove some symmetric inequalities.
Left Convex-Right Concave Function Theorem (Vasile Cirtoaje, 2004). Let a < c be
real numbers, let f be a continuous function on I = [a, ∞), strictly convex on [a, c] and
strictly concave on [c, ∞), and let

E(a1 , a2 , . . . , an ) = f (a1 ) + f (a2 ) + · · · + f (an ).

If a1 , a2 , . . . , an ∈ I such that

a1 + a2 + · · · + an = S = const ant,
218 Vasile Cîrtoaje

then
(a) E is minimum for a1 = a2 = · · · = an−1 ≤ an ;
(b) E is maximum for either a1 = a or a < a1 ≤ a2 = · · · = an .
On the other hand, it is known the following result concerning the best upper bound
of Jensen’s difference.
Best Upper Bound of Jensen’s Difference-Theorem (Vasile Cirtoaje, 1989). Let p1 , p2 , · · · , pn
be fixed positive real numbers, and let f be a convex function on a closed interval I = [a, b].
If a1 , a2 , · · · , an ∈ I, then Jensen’s difference

p1 f (a1 ) + p2 f (a2 ) + · · · + pn f (an ) p1 a1 + p2 a2 + · · · + pn an


 ‹
D= −f
p1 + p2 + · · · + p n p1 + p2 + · · · + pn

is maximum when some of ai are equal to a, and the others ai are equal to b; that is, when
all ai ∈ {a, b}.
ƒ

14. KARAMATA’S MAJORIZATION INEQUALITY




We say that a vector A = (a1 , a2 , . . . , an ) with a1 ≥ a2 ≥ · · · ≥ an majorizes a vector


B = (b1 , b2 , . . . , bn ) with b1 ≥ b2 ≥ · · · ≥ bn , and write it as

→ − →
A ≥ B,

if
a1 ≥ b1 ,
a1 + a2 ≥ b1 + b2 ,
·····················
a1 + a2 + · · · + an−1 ≥ b1 + b2 + · · · + bn−1 ,
a1 + a2 + · · · + an = b1 + b2 + · · · + bn .
Let f be a convex function on a real interval I. If a decreasingly ordered vector


A = (a1 , a2 , . . . , an ), ai ∈ I,

majorizes a decreasingly ordered vector




B = (b1 , b2 , . . . , bn ), bi ∈ I,

then
f (a1 ) + f (a2 ) + · · · + f (an ) ≥ f (b1 ) + f (b2 ) + · · · + f (bn ).

ƒ
Glossary 219

15. POPOVICIU’S INEQUALITY

If f is a convex function on a real interval I and a1 , a2 , . . . , an ∈ I, then


a + a + ··· + a 
1 2 n
f (a1 ) + f (a2 ) + · · · + f (an ) + n(n − 2) f ≥
n
≥ (n − 1)[ f (b1 ) + f (b2 ) + · · · + f (bn )],
where
1 X
bi = aj, i = 1, 2, · · · , n.
n − 1 j6=i
ƒ
16. SQUARE PRODUCT INEQUALITY

Let a, b, c be real numbers, and let

p = a + b + c, q = a b + bc + ca, r = a bc,
Æ p
s = p2 − 3q = a2 + b2 + c 2 − a b − bc − ca.
From the identity

27(a − b)2 (b − c)2 (c − a)2 = 4(p2 − 3q)3 − (2p3 − 9pq + 27r)2 ,

it follows that

−2p3 + 9pq − 2(p2 − 3q) p2 − 3q −2p3 + 9pq + 2(p2 − 3q) p2 − 3q


p p
≤r≤ ,
27 27
which is equivalent to

p3 − 3ps2 − 2s3 p3 − 3ps2 + 2s3


≤r≤ .
27 27
Therefore, for constant p and q, the product r is minimal and maximal when two of
a, b, c are equal.
ƒ
17. VASC’S INEQUALITY
If a, b, c are real numbers, then

(a2 + b2 + c 2 )2 ≥ 3(a3 b + b3 c + c 3 a),

with equality for a = b = c, and also for


a b c
= = π
sin2 4π
7 sin2 2π
7
sin2 7
220 Vasile Cîrtoaje

(or any cyclic permutation) - Vasile Cirtoaje, 1991.


A generalization of this inequality is the following (Vasile Cirtoaje, 2007):
X X X X X
a4 + A a2 b2 + Ba bc a≥C a3 b + D a b3 ,

where A, B, C, D are real numbers such that

1 + A + B = C + D,

3(1 + A) ≥ C 2 + C D + D2 .

18. SYMMETRIC INEQUALITIES OF DEGREE THREE, FOUR OR FIVE


Let f n (a, b, c) be a symmetric homogeneous polynomial of degree n = 3, n = 4 or n = 5.
(a) The inequality f4 (a, b, c) ≥ 0 holds for all real numbers a, b, c if and only if
f4 (a, 1, 1) ≥ 0 for all real a;
(b) The inequality f n (a, b, c) ≥ 0 holds for all a, b, c ≥ 0 if and only if f n (a, 1, 1) ≥ 0
and f n (0, b, c) ≥ 0 for all a, b, c ≥ 0 (Vasile Cirtoaje, 2008).

19. SYMMETRIC INEQUALITIES OF DEGREE SIX


Any sixth degree symmetric homogeneous polynomial f6 (a, b, c) can be written in the
form
f6 (a, b, c) = Ar 2 + B(p, q)r + C(p, q),

where A is called the highest coefficient of f6 , and

p = a + b + c, q = a b + bc + ca, r = a bc.

Case 1: A ≤ 0. The following statement holds.


(a) The inequality f6 (a, b, c) ≥ 0 holds for all real numbers a, b, c if and only if
f6 (a, 1, 1) ≥ 0 for all real a;
(b) The inequality f6 (a, b, c) ≥ 0 holds for all a, b, c ≥ 0 if and only if f6 (a, 1, 1) ≥ 0
and f6 (0, b, c) ≥ 0 for all a, b, c ≥ 0 (Vasile Cirtoaje, 2008).
Case 2: A > 0. We can use the highest coefficient cancellation method (Vasile Cirtoaje,
2008). This method consists in finding some suitable real numbers B, C and D such
that the following sharper inequality holds
2
q2

3
f6 (a, b, c) ≥ A r + Bp + C pq + D .
p
Glossary 221

Because the function g6 defined by


2
q2

3
g6 (a, b, c) = f6 (a, b, c) − A r + Bp + C pq + D
p

has the highest coefficient A1 = 0, we can prove the inequality g6 (a, b, c) ≥ 0 as in the
preceding case 1.
Notice that sometimes it is useful to break the problem into two parts, p2 ≤ ξq and
p2 > ξq, where ξ is a suitable real number.

20. EQUAL VARIABLE METHOD


The Equal Variable Theorem (EV-Theorem) for nonnegative real variables has the fol-
lowing statement (Vasile Cirtoaje, 2005).
EV-Theorem (for nonnegative variables). Let a1 , a2 , · · · , an (n ≥ 3) be fixed nonnegative
real numbers, and let x 1 ≤ x 2 ≤ · · · ≤ x n be nonnegative real variables such that

x 1 + x 2 + · · · + x n = a1 + a2 + · · · + an ,

x 1k + x 2k + · · · + x nk = a1k + a2k + · · · + ank ,


where k is a real number (for k = 0, assume that x 1 x 2 · · · x n = a1 a2 · · · an > 0). Let
f : (0, ∞) → R be a differentiable function such that g : (0, ∞) → R defined by
€ 1 Š
g(x) = f 0 x k−1

is strictly convex, and let

Sn = f (x 1 ) + f (x 2 ) + · · · + f (x n ).

(1) If k ≤ 0, then Sn is maximum for

0 < x 1 = x 2 = · · · = x n−1 ≤ x n ,

and is minimum for


0 < x1 ≤ x2 = x3 = · · · = x n;
(2) If k > 0 and either f is continuous at x = 0 or f (0+ ) = −∞, then Sn is maximum
for
0 ≤ x 1 = x 2 = · · · = x n−1 ≤ x n ,
and is minimum for
x 1 = · · · = x j−1 = 0, x j+1 = · · · = x n ,
222 Vasile Cîrtoaje

where j ∈ {1, 2, · · · , n}.


For f (x) = x m , we get the following corollary.
EV-COROLLARY (for nonnegative variables). Let a1 , a2 , · · · , an (n ≥ 3) be fixed nonneg-
ative real numbers, let x 1 ≤ x 2 ≤ · · · ≤ x n be nonnegative real variables such that

x 1 + x 2 + · · · + x n = a1 + a2 + · · · + an ,

x 1k + x 2k + · · · + x nk = a1k + a2k + · · · + ank ,


and let
Sn = x 1m + x 2m + · · · + x nm .
Case 1 : k ≤ 0 (for k = 0, assume that x 1 x 2 · · · x n = a1 a2 · · · an > 0 ).
(a) If m ∈ (k, 0) ∪ (1, ∞), then Sn is maximum for

0 < x 1 = x 2 = · · · = x n−1 ≤ x n ,

and is minimum for


0 < x1 ≤ x2 = x3 = · · · = x n;
(b) If m ∈ (−∞, k) ∪ (0, 1), then Sn is minimum for

0 < x 1 = x 2 = · · · = x n−1 ≤ x n ,

and is maximum for


0 < x1 ≤ x2 = x3 = · · · = x n.
Case 2 : 0 < k < 1.
(a ) If m ∈ (0, k) ∪ (1, ∞), then Sn is maximum for

0 ≤ x 1 = x 2 = · · · = x n−1 ≤ x n ,

and is minimum for


x 1 = · · · = x j−1 = 0, x j+1 = · · · = x n ,
where j ∈ {1, 2, · · · , n};
(b) If m ∈ (−∞, 0) ∪ (k, 1), then Sn is minimum for

0 ≤ x 1 = x 2 = · · · = x n−1 ≤ x n ,

and is maximum for


x 1 = · · · = x j−1 = 0, x j+1 = · · · = x n ,
where j ∈ {1, 2, · · · , n}.
Case 3 : k > 1.
Glossary 223

(a) If m ∈ (0, 1) ∪ (k, ∞), then Sn is maximum for

0 ≤ x 1 = x 2 = · · · = x n−1 ≤ x n ,

and is minimum for


x 1 = · · · = x j−1 = 0, x j+1 = · · · = x n ,
where j ∈ {1, 2, · · · , n};
(b) If m ∈ (−∞, 0) ∪ (1, k), then Sn is minimum for

0 ≤ x 1 = x 2 = · · · = x n−1 ≤ x n ,

and is maximum for


x 1 = · · · = x j−1 = 0, x j+1 = · · · = x n ,
where j ∈ {1, 2, · · · , n}.

The Equal Variable Theorem (EV-Theorem) for real variables has the following
statement (Vasile Cirtoaje, 2012).
EV-Theorem (for real variables). Let a1 , a2 , · · · , an (n ≥ 3) be fixed real numbers, let
x 1 ≤ x 2 ≤ · · · ≤ x n be real variables such that

x 1 + x 2 + · · · + x n = a1 + a2 + · · · + an ,

x 1k + x 2k + · · · + x nk = a1k + a2k + · · · + ank ,


where k is an even positive integer, and let f be a differentiable function on R such that the
associated function g : R → R defined by
p 
g(x) = f 0
k−1
x

is strictly convex on R. Then, the sum

Sn = f (x 1 ) + f (x 2 ) + · · · + f (x n )

is minimum for x 2 = x 3 = · · · = x n , and is maximum for x 1 = x 2 = · · · = x n−1 .

21. ARITHMETIC COMPENSATION METHOD


The Arithmetic Compensation Theorem (AC-Theorem) has the following statement (Vasile
Cirtoaje, 2002).
AC-THEOREM. Let s > 0 and let F be a symmetric continuous function on the compact set
in Rn

S = {(x 1 , x 2 , · · · , x n ) : x 1 + x 2 + · · · + x n = s, x i ≥ 0, i = 1, 2, · · · , n}.
224 Vasile Cîrtoaje

If
F (x 1 , x 2 , x 3 , · · · , x n ) ≥
n x + x x + x  o
1 2 1 2
≥ min F , , x 3 , · · · , x n , F (0, x 1 + x 2 , x 3 , · · · , x n )
2 2
for all (x 1 , x 2 , · · · , x n ) ∈ S, then F (x 1 , x 2 , x 3 , · · · , x n ) is minimal when
s
x1 = x2 = · · · = x k = , x k+1 = · · · = x n = 0;
k
that is, s s 
F (x 1 , x 2 , x 3 , · · · , x n ) ≥ min F , · · · , , 0, · · · , 0
1≤k≤n k k
for all (x 1 , x 2 , · · · , x n ) ∈ S.
Notice that if
x + x x + x 
1 2 1 2
F (x 1 , x 2 , x 3 , · · · , x n ) < F , , x3, · · · , x n
2 2
involves
F (x 1 , x 2 , x 3 , · · · , x n ) ≥ F (0, x 1 + x 2 , x 3 , · · · , x n ),
then the hypothesis
F (x 1 , x 2 , x 3 , · · · , x n ) ≥
n x + x x + x  o
1 2 1 2
≥ min F , , x 3 , · · · , x n , F (0, x 1 + x 2 , x 3 , · · · , x n )
2 2
is satisfied.

You might also like